UnRegistered

页码,1/1
The Official SAT Online Course
Help | Profile | My Organizer | My Bookmarks | Logout
Answers and Explanations
Back to Score Report
Test Sections
Section 1
Essay
Section 2
Online - Practice Test #5
Section 3
Section 4
Section 5
Section 6
These sample essays were originally handwritten by students but are shown typed here for ease
of reading. The essays are displayed exactly as students wrote them, without any corrections to
spelling, punctuation, or syntax. One handwritten sample essay is provided to illustrate the need
for legible and clear handwriting.
Section 8
Section 9
Section 10
Exemplars:
Essay Prompt
Think carefully about the information presented in the following excerpt and the assignment
below.
!
Many people believe that “closed doors make us creative.” These people argue that
obstacles and restrictions are necessary, for without them we would never be forced to come
up with new solutions. But “closed doors,” either in the form of specific obstacles or a lack
of opportunities, often prevent people from reaching their full creative potential.
er
ed
途
用
Do closed doors make us creative? Plan and write an essay in which you develop your point of
view on this issue. Support your position with reasoning and examples taken from your
reading, studies, experience, or observations.
is
t
业
商
于
eg
Copyright © 2006 The College Board. All rights reserved.
Privacy Policy
Terms of Use
Contact Us
nR
用
Back to Score Report
禁
U
严
file://E:\新建文件夹\e2.htm
2006-11-12
页码,1/23
The Official SAT Online Course
Help | Profile | My Organizer | My Bookmarks | Logout
Answers and Explanations
Back to Score Report
Test Sections
Section 1
View Answers and Explanations
Section 2
Online - Practice Test #5
Section 4
1
The garden that had remained ------- for months was now pleasantly enlivened by
the budding shoots of its perennial flowers.
Section 5
Section 6
Section 8
(A)
redolent
(B)
dormant
Section 9
(C) exuberant
Section 10
(D) compliant
(E)
trenchant
ANSWERS
!
途
AND EXPLANATIONS
ed
Section 3
Explanation for Correct Answer B :
Choice (B) is correct. “Dormant” means inactive or not growing. If one were to
insert this term into the text, the sentence would read “The garden that had
remained dormant for months was now pleasantly enlivened by the budding shoots
of its perennial flowers. ” A “dormant,” or inactive, garden would become active
as its perennial flowers began to grow.
er
用
is
t
业
商
eg
Explanation for Incorrect Answer A :
Choice (A) is incorrect. “Redolent” means aromatic or fragrant. If one were to
insert this term into the text, the sentence would read “The garden that had
remained redolent for months was now pleasantly enlivened by the budding shoots
of its perennial flowers. ” A garden in full bloom can be “redolent,” but the
phrase “now pleasantly enlivened” suggests that the garden was inactive before
the flowers began to grow. An inactive garden is most likely not fragrant.
于
nR
用
禁
Explanation for Incorrect Answer C :
Choice (C) is incorrect. “Exuberant” means plentiful or producing in abundance.
If one were to insert this term into the text, the sentence would read “The garden
that had remained exuberant for months was now pleasantly enlivened by the
budding shoots of its perennial flowers.” The phrase “now pleasantly enlivened”
suggests that the garden was inactive before the flowers began to grow. An inactive
garden would most likely not be described as “exuberant,” or plentiful.
U
严
Explanation for Incorrect Answer D :
Choice (D) is incorrect. “Compliant” means willing to conform. If one were to
insert this term into the text, the sentence would read “The garden that had
remained compliant for months was now pleasantly enlivened by the budding
shoots of its perennial flowers.” It is illogical to suggest that a garden could be
“compliant,” or willing to conform.
Explanation for Incorrect Answer E :
Choice (E) is incorrect. “Trenchant” means keen or sharply perceptive. If one
were to insert this term into the text, the sentence would read “The garden that
had remained trenchant for months was now pleasantly enlivened by the budding
shoots of its perennial flowers.” It is illogical to suggest that a garden could be
“trenchant,” or sharply perceptive.
2
After several months of training, the ------- young spaniel was finally ------- enough
to be walked safely without a leash.
(A)
file://E:\新建文件夹\e3.htm
eager . . unruly
2006-11-12
页码,2/23
The Official SAT Online Course
(B)
placid . . defiant
(C) clever . . helpful
(D) boisterous . . docile
(E)
vigilant . . convinced
ANSWERS
AND EXPLANATIONS
Explanation for Correct Answer D :
Choice (D) is correct. “Boisterous” means rowdy and undisciplined. “Docile”
means easily managed. If one were to insert these terms into the text, the
sentence would read “After several months of training, the boisterous young
spaniel was finally docile enough to be walked safely without a leash.” The word
“finally” indicates that after several months of training, the spaniel’s behavior
changed. It is logical to suggest that a “boisterous” dog could become “docile”
after several months of training. A dog that is docile, or easily managed, could
likely be walked safely without a leash.
Explanation for Incorrect Answer A :
Choice (A) is incorrect. “Eager” means enthusiastic or impatient. “Unruly”
means undisciplined and willful. If one were to insert these terms into the text, the
sentence would read “After several months of training, the eager young spaniel
was finally unruly enough to be walked safely without a leash.” Although a dog
can certainly be “eager,” it is unlikely that an “unruly” dog could be walked
safely without a leash. Furthermore, one would not expect training to cause a dog
to be unruly, or undisciplined.
re
d
!
途
用
te
Explanation for Incorrect Answer B :
Choice (B) is incorrect. “Placid” means calm and undisturbed. “Defiant” means
bold and disobedient. If one were to insert these terms into the text, the sentence
would read “After several months of training, the placid young spaniel was finally
defiant enough to be walked safely without a leash.” Although a dog can certainly
be “placid,” or calm, it is unlikely that a “defiant” dog could be walked safely
without a leash. Furthermore, one would not expect training to cause a dog to be
defiant, or disobedient.
业
eg
is
商
于
Explanation for Incorrect Answer C :
Choice (C) is incorrect. “Clever” means smart and quick-witted. “Helpful”
means providing assistance. If one were to insert these terms into the text, the
sentence would read “After several months of training, the clever young spaniel
was finally helpful enough to be walked safely without a leash.” Although the first
term fits within the context of the sentence, the second term does not. A dog can
certainly be “clever,” but it does not make logical sense to say that a dog is
providing assistance by walking without a leash.
nR
用
禁
U
严
Explanation for Incorrect Answer E :
Choice (E) is incorrect. “Vigilant” means watchful and alert. “Convinced”
means certain or persuaded. If one were to insert these terms into the text, the
sentence would read “After several months of training, the vigilant young spaniel
was finally convinced enough to be walked safely without a leash.” Although a dog
can be “vigilant,” it does not make sense in this context to say that the dog was
“convinced enough” to be walked without a leash. It is not clear what the dog
has been convinced of or why convincing was necessary in order to walk the dog
without a leash—or, indeed, that dogs can be “convinced,” a term usually
applied to humans.
3
------- as Mario’s misdeed was, his grandmother, always blind to his faults,
pretended to be unaware of it.
(A)
Accidental
(B)
Apt
(C) Random
(D) Flagrant
(E)
file://E:\新建文件夹\e3.htm
Covert
2006-11-12
页码,3/23
The Official SAT Online Course
ANSWERS
AND EXPLANATIONS
Explanation for Correct Answer D :
Choice (D) is correct. “Flagrant” means conspicuously or obviously offensive. If
one were to insert this term into the text, the sentence would read “Flagrant as
Mario’s misdeed was, his grandmother, always blind to his faults, pretended to be
unaware of it.” The structure of the sentence indicates that Mario’s grandmother
was certainly aware of Mario’s misdeed, even though she pretended to be
unaware. The missing term should describe a misdeed that is very obvious. The
term “flagrant” accurately describes an action that is obviously offensive.
Explanation for Incorrect Answer A :
Choice (A) is incorrect. “Accidental” means occurring unexpectedly or by chance.
If one were to insert this term into the text, the sentence would read “Accidental
as Mario’s misdeed was, his grandmother, always blind to his faults, pretended to
be unaware of it.” The structure of the sentence indicates that Mario’s
grandmother was certainly aware of Mario’s misdeed, even though she pretended
to be unaware. The missing term should describe a misdeed that is very obvious.
An action that is “accidental” would not necessarily be obvious.
Explanation for Incorrect Answer B :
Choice (B) is incorrect. “Apt” means appropriate or suitable. If one were to insert
this term into the text, the sentence would read “Apt as Mario’s misdeed was,
his grandmother, always blind to his faults, pretended to be unaware of it.” A
“misdeed” is inappropriate behavior and would not likely be described as
“apt.”
ed
!
途
用
er
Explanation for Incorrect Answer C :
Choice (C) is incorrect. “Random” means lacking a definite pattern or plan. If one
were to insert this term into the text, the sentence would read “Random as
Mario’s misdeed was, his grandmother, always blind to his faults, pretended to be
unaware of it.” The structure of the sentence indicates that Mario’s grandmother
was certainly aware of Mario’s misdeed, even though she pretended to be
unaware. The missing term should describe a misdeed that is very obvious. An
action that is “random,” or does not follow a pattern or plan, would not
necessarily be obvious.
is
t
业
nR
eg
商
于
Explanation for Incorrect Answer E :
Choice (E) is incorrect. “Covert” means secret or not openly shown. If one were
to insert this term into the text, the sentence would read “Covert as Mario’s
misdeed was, his grandmother, always blind to his faults, pretended to be unaware
of it.” The structure of the sentence indicates that Mario’s grandmother was
certainly aware of Mario’s misdeed, even though she pretended to be unaware.
The missing term should describe a misdeed that is very obvious. A “covert,” or
secret, action is the opposite of an obvious action. Mario’s grandmother would
most likely not be aware of a “covert” misdeed.
用
禁
U
严
4
Despite his ------- desire to show off, he remained at heart a very ------- person.
(A)
uncharacteristic. . demonstrative
(B)
inexplicable. . hedonistic
(C) occasional. . reticent
(D) continual. . transparent
(E)
blatant. . exhibitionistic
ANSWERS
AND EXPLANATIONS
Explanation for Correct Answer C :
Choice (C) is correct. “Occasional” means happening only sometimes.
“Reticent” means quiet and reserved. If one were to insert these terms into the
text, the sentence would read “Despite his occasional desire to show off, he
remained at heart a very reticent person.” The word “Despite” indicates that
the missing terms will describe contrasting behaviors. Someone who is
“reticent,” or quiet and reserved, would not frequently show off but might do so
from time to time.
file://E:\新建文件夹\e3.htm
2006-11-12
页码,4/23
The Official SAT Online Course
Explanation for Incorrect Answer A :
Choice (A) is incorrect. “Uncharacteristic” means not typical. “Demonstrative”
means displaying feelings openly. If one were to insert these terms into the text,
the sentence would read “Despite his uncharacteristic desire to show off, he
remained at heart a very demonstrative person.” The word “Despite” indicates
that the missing terms will describe contrasting behaviors. There is no inherent
contrast between the terms “uncharacteristic” and “demonstrative,” because
showing off is not necessarily uncharacteristic behavior for someone who displays
his feelings openly.
Explanation for Incorrect Answer B :
Choice (B) is incorrect. “Inexplicable” means incapable of being explained.
“Hedonistic” means concerned primarily with happiness and pleasure. If one
were to insert these terms into the text, the sentence would read “Despite his
inexplicable desire to show off, he remained at heart a very hedonistic person.”
The word “Despite” indicates that the missing terms will describe contrasting
behaviors. Even if the desire to show off is “inexplicable,” or incapable of being
explained, there is no inherent contrast between having a desire to show off and
being “hedonistic.” Showing off is not necessarily related to being primarily
concerned with happiness and pleasure.
Explanation for Incorrect Answer D :
Choice (D) is incorrect. “Continual” means going on without interruption.
“Transparent” means obvious and free from pretense. If one were to insert these
terms into the text, the sentence would read “Despite his continual desire to show
off, he remained at heart a very transparent person.” The word “Despite”
indicates that the missing terms will describe contrasting behaviors. There is no
inherent contrast between having a “continual” desire to show off and being a
“transparent” person, and a person could be obvious in his desire to show off.
re
d
!
途
用
is
te
Explanation for Incorrect Answer E :
Choice (E) is incorrect. “Blatant” means completely obvious. “Exhibitionistic”
means acting in a way that will attract attention. If one were to insert these terms
into the text, the sentence would read “Despite his blatant desire to show off, he
remained at heart a very exhibitionistic person.” The word “Despite” indicates
that the missing terms will describe contrasting behaviors. There is no contrast
between having a “blatant” desire to show off and being an “exhibitionistic”
person. On the contrary, someone who wants to attract attention would probably be
likely to show off.
业
eg
商
用
nR
5
于
The employer blamed the staff member's lack of productivity on ------- rather than
incompetence, claiming that the man knew how to do his job but was too lazy to
apply himself.
U
严
禁
(A)
infatuation
(B)
tension
(C) indigence
(D) indolence
(E)
ineptitude
ANSWERS
AND EXPLANATIONS
Explanation for Correct Answer D :
Choice (D) is correct. “Indolence ” is a tendency to be lazy. If one were to insert
this term into the text, the sentence would read “The employer blamed the staff
member’s lack of productivity on indolence rather than incompetence, claiming
that the man knew how to do his job but was too lazy to apply himself.” The
employer’s claim that the staff member was competent but “too lazy to apply
himself” indicates that the staff member’s lack of productivity was the result of
laziness. The term “indolence” precisely describes the laziness that the employer
claimed was responsible for the staff member’s lack of productivity.
Explanation for Incorrect Answer A :
Choice (A) is incorrect. “Infatuation” is a foolish or extravagant love or
admiration for something. If one were to insert this term into the text, the sentence
would read “The employer blamed the staff member’s lack of productivity on
file://E:\新建文件夹\e3.htm
2006-11-12
页码,5/23
The Official SAT Online Course
infatuation rather than incompetence, claiming that the man knew how to do his job
but was too lazy to apply himself.” The employer’s claim that the staff member
was competent but “too lazy to apply himself” indicates that the staff member’s
lack of productivity was the result of laziness. The missing term should describe this
laziness. “Infatuation,” or a foolish love, does not describe laziness.
Explanation for Incorrect Answer B :
Choice (B) is incorrect. “Tension” is stress. If one were to insert this term into
the text, the sentence would read “The employer blamed the staff member’s lack
of productivity on tension rather than incompetence, claiming that the man knew
how to do his job but was too lazy to apply himself.” The employer’s claim that
the staff member was competent but “too lazy to apply himself” indicates that
the staff member’s lack of productivity was the result of laziness. The missing
term should describe this laziness. “Tension,” or stress, does not describe
laziness.
Explanation for Incorrect Answer C :
Choice (C) is incorrect. “Indigence” is extreme poverty. If one were to insert this
term into the text, the sentence would read “The employer blamed the staff
member’s lack of productivity on indigence rather than incompetence, claiming
that the man knew how to do his job but was too lazy to apply himself.” The
employer’s claim that the staff member was competent but “too lazy to apply
himself” indicates that the staff member’s lack of productivity was the result of
laziness. The missing term should describe this laziness. “Indigence,” or extreme
poverty, does not describe laziness.
!
途
ed
Explanation for Incorrect Answer E :
Choice (E) is incorrect. “Ineptitude” is incompetence and a lack of sense. If one
were to insert this term into the text, the sentence would read “The employer
blamed the staff member’s lack of productivity on ineptitude rather than
incompetence, claiming that the man knew how to do his job but was too lazy to
apply himself.” The term “ineptitude” is not logical in this context.
“Ineptitude” means incompetence. The employer did not blame the staff
member’s lack of productivity on incompetence but, rather, claimed that the staff
member “knew how to do his job.”
业
商
eg
The audience recognized the officer’s characteristic ------- when he attributed his
achievements to ------- rather than bravery.
于
(A)
pedantry. . chance
(B)
gallantry. . whimsy
用
nR
6
is
te
r
用
(C) humility. . fortune
(D) bravado. . accident
禁
(E)
U
严
effrontery. . discretion
ANSWERS
AND EXPLANATIONS
Explanation for Correct Answer C :
Choice (C) is correct. “Humility” is the quality of being humble and not overly
proud. “Fortune” means luck or chance. If one were to insert these terms into
the text, the sentence would read “The audience recognized the officer’s
characteristic humility when he attributed his achievements to fortune rather than
bravery.” The sentence structure indicates that the first missing term will describe
the behavior demonstrated in the officer’s discussion of his achievements.
Attributing achievements to “fortune,” or luck, instead of bravery is an example
of “humility.”
Explanation for Incorrect Answer A :
Choice (A) is incorrect. “Pedantry” means to pay extreme attention to rules.
“Chance” means luck or an unpredictable advantage. If one were to insert these
terms into the text, the sentence would read “The audience recognized the
officer’s characteristic pedantry when he attributed his achievements to chance
rather than bravery.” The sentence structure indicates that the first missing term
will describe the behavior demonstrated in the officer’s discussion of his
achievements. Attributing achievements to “chance,” or luck, is not an example
of “pedantry,” or extreme attention to rules.
Explanation for Incorrect Answer B :
file://E:\新建文件夹\e3.htm
2006-11-12
页码,6/23
The Official SAT Online Course
Choice (B) is incorrect. “Gallantry” is very noble and brave behavior. “Whimsy”
is a whim or a sudden impulse. If one were to insert these terms into the text, the
sentence would read “The audience recognized the officer’s characteristic
gallantry when he attributed his achievements to whimsy rather than bravery.”
The sentence structure indicates that the first missing term will describe the
behavior demonstrated by the officer’s discussion of his achievements. There is
not necessarily anything noble or brave in the officer’s claim that his
achievements are due to “whimsy,” or sudden impulse.
Explanation for Incorrect Answer D :
Choice (D) is incorrect. “Bravado” means pretending to be brave. “Accident” is
an unplanned and unexpected event. If one were to insert these terms into the text,
the sentence would read “The audience recognized the officer’s characteristic
bravado when he attributed his achievements to accident rather than bravery.”
Attributing his achievements to “accident” instead of bravery is not an example
of the officer pretending to be brave. Someone demonstrating “bravado” would
probably attribute his achievements to bravery.
Explanation for Incorrect Answer E :
Choice (E) is incorrect. “Effrontery” means offensive boldness. “Discretion” is
the ability to make responsible decisions. If one were to insert these terms into the
text, the sentence would read “The audience recognized the officer’s
characteristic effrontery when he attributed his achievements to discretion rather
than bravery.” There is no reason to believe that the audience would consider the
officer to be offensively bold when attributing his achievements to “discretion,” or
responsible choices.
ed
The strong ------- the professor was able to establish with his students made him
------- confidant for those on campus seeking advice beyond the purely academic.
(A)
program. . an occasional
(B)
rapport. . a respected
用
er
7
!
途
业
is
t
(C) confidence. . an unappreciated
商
(D) community. . an unusual
ambition. . a valued
eg
(E)
ANSWERS
于
AND EXPLANATIONS
nR
用
Explanation for Correct Answer B :
Choice (B) is correct. “Rapport” means a relationship of trust. “Respected”
means thought highly of and appreciated. If one were to insert these terms into the
text, the sentence would read “The strong rapport the professor was able to
establish with his students made him a respected confidant for those on campus
seeking advice beyond the purely academic.” A “confidant” is someone one
trusts with secrets or private matters. If the professor established a “rapport,” or
relationship of trust, with his students, it makes sense that the students would
respect his advice on personal matters.
禁
U
严
Explanation for Incorrect Answer A :
Choice (A) is incorrect. “Program” means an academic plan or curriculum.
“Occasional” means happening only sometimes. If one were to insert these
terms into the text, the sentence would read “The strong program the professor
was able to establish with his students made him an occasional confidant for those
on campus seeking advice beyond the purely academic.” A “confidant” is
someone one trusts with secrets or private matters. While the professor could
establish a strong “program” with the help of his students, there is not
necessarily any reason to believe that the professor’s program would cause his
students to seek him out to discuss personal matters.
Explanation for Incorrect Answer C :
Choice (C) is incorrect. “Confidence” is trust in a person or thing.
“Unappreciated” means not recognized or not valued. If one were to insert these
terms into the text, the sentence would read “The strong confidence the professor
was able to establish with his students made him an unappreciated confidant for
those on campus seeking advice beyond the purely academic.” A “confidant” is
someone one trusts with secrets or private matters. If the professor was able to
establish “confidence,” or trust, with his students, it is reasonable to suggest
that they would appreciate his advice on personal matters.
file://E:\新建文件夹\e3.htm
2006-11-12
页码,7/23
The Official SAT Online Course
Explanation for Incorrect Answer D :
Choice (D) is incorrect. “Community” means a group of people with a connection
or something in common. “Unusual” means not ordinary. If one were to insert
these terms into the text, the sentence would read “The strong community the
professor was able to establish with his students made him an unusual confidant for
those on campus seeking advice beyond the purely academic.” A “confidant” is
someone one trusts with secrets or private matters. If the professor had established
a strong “community,” it would not be “unusual” for his students to seek his
advice on personal matters.
Explanation for Incorrect Answer E :
Choice (E) is incorrect. “Ambition” is the desire to achieve a specific thing.
“Valued” means appreciated and regarded highly. If one were to insert these
terms into the text, the sentence would read “The strong ambition the professor
was able to establish with his students made him a valued confidant for those on
campus seeking advice beyond the purely academic.” While the second term fits
the meaning of the sentence, the first term does not. A professor could be a
“valued” confidant, or person trusted with private matters, but it does not make
logical sense to say that the professor established “ambition” with his students.
After David left him waiting for the third consecutive time, Kirk realized that the same
behavior he had initially valued as spontaneous and carefree was, in fact, simply
-------.
(A)
capricious
(B)
incontrovertible
!
途
ed
8
(C) extraneous
(D) captivating
inscrutable
ANSWERS
用
is
te
r
(E)
业
AND EXPLANATIONS
商
eg
Explanation for Correct Answer A :
Choice (A) is correct. “Capricious” means irresponsibly unpredictable. If one were
to insert this term into the text, the sentence would read “After David left him
waiting for the third consecutive time, Kirk realized that the same behavior he had
initially valued as spontaneous and carefree was, in fact, simply capricious.” The
phrase “initially valued” suggests that after being left waiting three times in a
row, Kirk changed his opinion of David’s behavior. It would make sense to say
that Kirk would no longer value David’s behavior if he came to see it as
“capricious,” or irresponsibly unpredictable.
于
nR
用
禁
U
严
Explanation for Incorrect Answer B :
Choice (B) is incorrect. “Incontrovertible” means indisputable or not open to
question. If one were to insert this term into the text, the sentence would read
“After David left him waiting for the third consecutive time, Kirk realized that the
same behavior he had initially valued as spontaneous and carefree was, in fact,
simply incontrovertible.” The phrase “initially valued” indicates that after being
left waiting three times in a row, Kirk changed his opinion of David’s behavior.
There is no reason to believe that Kirk would not have initially believed that
David’s behavior was “incontrovertible,” or not open to question.
Explanation for Incorrect Answer C :
Choice (C) is incorrect. “Extraneous” means irrelevant and not essential. If one
were to insert this term into the text, the sentence would read “After David left
him waiting for the third consecutive time, Kirk realized that the same behavior he
had initially valued as spontaneous and carefree was, in fact, simply extraneous.”
The phrase “initially valued” suggests that after being left waiting three times in
a row, Kirk changed his opinion of David’s behavior. If David’s behavior affected
Kirk’s opinion, it is illogical to say that David’s behavior was “extraneous,” or
irrelevant.
Explanation for Incorrect Answer D :
Choice (D) is incorrect. “Captivating” means irresistibly appealing. If one were to
insert this term into the text, the sentence would read “After David had left him
waiting for the third consecutive time, Kirk realized that the same behavior he had
initially valued as spontaneous and carefree was, in fact, simply captivating.” The
phrase “initially valued” indicates that after being left waiting three times in a
file://E:\新建文件夹\e3.htm
2006-11-12
页码,8/23
The Official SAT Online Course
row, Kirk no longer valued David’s behavior. It is illogical to suggest that being
left waiting caused Kirk to consider David’s behavior to be “captivating,” or
irresistibly appealing.
Explanation for Incorrect Answer E :
Choice (E) is incorrect. “Inscrutable” means mysterious and not readily
understood. If one were to insert this term into the text, the sentence would read
“After David had left him waiting for the third consecutive time, Kirk realized that
the same behavior he had initially valued as spontaneous and carefree was, in fact,
simply inscrutable.” The phrase “the same behavior” indicates that while Kirk’
s opinion of David’s behavior changed, David’s behavior was always the same.
Therefore, the missing term should be another way to describe “spontaneous and
carefree” behavior. Behavior that is spontaneous and carefree is not necessarily
“inscrutable,” or mysterious.
What was most likely the original
purpose of the human appendix? Experts
can only theorize on its use. It may have
Line had the same purpose it has in present5
day herbivores, where it harbors colonies
of bacteria that help in the digestion of
cellulose. Another theory suggests that
tonsils and the appendix might
manufacture the antibody-producing white
10 blood cells called B lymphocytes;
however, B lymphocytes could also be
produced by bone marrow. The third
theory is that the appendix may “attract”
body infections in order to localize the
15 infection in one spot that is not critical to
body functioning.
is
te
re
d
!
途
用
业
商
eg
于
The author of the passage uses quotation marks in line 13 in order to indicate that
用
(A)
this theory is the one with which the author most nearly agrees
(B)
this theory is less scientifically valid than the other theories in the passage
nR
9
禁
(C) a common word is being used to describe a unique biological process
(D) a word is being used in a humorous way
U
严
(E)
a direct quotation from another source is being used
ANSWERS
AND EXPLANATIONS
Explanation for Correct Answer C :
Choice (C) is correct. In the passage, three theories are given concerning the
original purpose of the human appendix. In lines 12-16, the third and final theory is
given: “the appendix may ‘attract’ body infections in order to localize the
infection in one spot that is not critical to body functioning.” The use of the
quotation marks around “attract” implies that there are probably technical terms
to describe this “attraction,” but that the author is instead using a term that the
general reader will be likely to understand. The reader knows that a bodily organ
cannot really “attract,” or draw the attention of, an infection, but the use of this
familiar verb helps clarify the concept.
Explanation for Incorrect Answer A :
Choice (A) is incorrect. There is no indication that the author of the passage
supports the third theory any more than the other two theories, nor would the use
of quotation marks necessarily serve this purpose.
Explanation for Incorrect Answer B :
Choice (B) is incorrect. There is no indication that this third theory is any less
file://E:\新建文件夹\e3.htm
2006-11-12
页码,9/23
The Official SAT Online Course
scientifically valid than the other two theories. Using a non-technical term like
“attracts” to clarify a concept does not suggest that that concept is less than
valid.
Explanation for Incorrect Answer D :
Choice (D) is incorrect. While quotation marks might in certain cases connote
humor, that is not the case here. The tone of the sentence—and of the whole
passage—is objective and dispassionate.
Explanation for Incorrect Answer E :
Choice (E) is incorrect. While marking a direct quotation from another source is one
of the main uses of quotation marks, there is no indication here that the author of
the passage is deriving the word “attract” from another source.
10
How does the theory described in lines 3-7 primarily differ from the other two
theories described in the passage?
(A)
It pertains only to plants.
(B)
It concerns a physical process that occurs in more than one area of the
human body.
(C) It is a theory supported by more experts in the field than are the other
two theories.
!
途
(D) It is concerned with the prevention of disease.
It makes reference to a process presently occurring in other animals.
ANSWERS
ed
(E)
AND EXPLANATIONS
用
te
r
Explanation for Correct Answer E :
Choice (E) is correct. The theory described in lines 3-7 states that the human
appendix may originally have harbored colonies of bacteria to aid in digestion,
which is its purpose “in present-day herbivores.” Neither of the other theories in
the passage—in lines 7-12 and 12-16—refers to present-day animals, herbivorous
or carnivorous.
业
is
商
eg
于
Explanation for Incorrect Answer A :
Choice (A) is incorrect. None of the three theories described in the passage pertains
only to plants. The first theory mentions plants, but only in discussing the digestion
of cellulose by herbivores.
nR
用
禁
Explanation for Incorrect Answer B :
Choice (B) is incorrect. It is the second theory (lines 7-12), not the theory in lines
3-7, that deals with a process possibly occurring in two areas of the human body:
the production of lymphocytes in the appendix and the tonsils.
U
严
Explanation for Incorrect Answer C :
Choice (C) is incorrect. There is no indication that any of the three theories
described in the passage enjoys more scientific support than do the others. The
only mention of experts is in line 3 and pertains to all the theories.
Explanation for Incorrect Answer D :
Choice (D) is incorrect. Both the second and third theories could be said to pertain
to disease prevention—the second by aiding in the production of antibodies, the
third by isolating infections—so disease prevention cannot be said to be
represented in the first theory only.
One hazard in historical study is
the necessity of dividing the whole into
segments, since not everything can be
Line examined simultaneously. Common ways
5
of dividing history are by period, country,
topic, artistic or political movement, or
theme. Each of these can be justified, but
file://E:\新建文件夹\e3.htm
2006-11-12
页码,10/23
The Official SAT Online Course
all have their shortcomings. When
divisions are made according to country,
10 the interconnections among events
occurring in two or more countries may go
unnoticed or remain unexplored. Division
into time periods may interrupt or obscure
ongoing developments, or may give undue
15 emphasis to some event or type of activity
(especially war or politics) as crucial in
marking the end or beginning of a period
or movement.
11
Which of the following is most analogous to the “hazard” the author sees in the
“division” of historical study?
(A)
A lawyer accepts cases in too many different areas of legal practice.
(B)
A teacher must cope with large class sizes and is unable to give students
sufficient individual instruction.
(C) A biologist studies large areas of forest but fails to examine in depth the
nesting site of a specific bird species.
!
途
(E)
re
d
(D) An artist produces works in many different media, but does not excel in
any one medium.
A doctor diagnoses one ailment but overlooks elements of the patient’s
overall health.
用
AND EXPLANATIONS
te
ANSWERS
业
Explanation for Correct Answer E :
Choice (E) is correct. The author argues in this passage that dividing history into
segments for instructional purposes is always risky. No matter how history is
divided, distortions of emphasis will occur. Focusing on one part of history—an
international period or a single nation’s history, for instance—leaves larger
connections unexplored. This type of “hazard” is analogous to the challenge
doctors face when assessing a patient’s health. A doctor might make a specific
diagnosis associated with an ailing portion of a patient’s body in order to
recommend treatment, but may inadvertently overlook larger, more global
elements of the patient’s health.
nR
eg
is
商
于
用
禁
Explanation for Incorrect Answer A :
Choice (A) is incorrect. The author argues in this passage that dividing history into
segments for instructional purposes is always risky. No matter how history is
divided, distortions of emphasis will occur. Focusing on one part of history—an
international period or a single nation’s history, for instance—leaves larger
connections unexplored. A lawyer accepting cases in too many different areas of
legal practice would be a reverse of the “hazard” in historical studies: the lawyer
would be spread too thin by too wide a focus, instead of missing the bigger picture
because of a specializing too narrowly.
U
严
Explanation for Incorrect Answer B :
Choice (B) is incorrect. The author argues in this passage that dividing history into
segments for instructional purposes is always risky. No matter how history is
divided, distortions of emphasis will occur. Focusing on one part of history—an
international period or a single nation’s history, for instance—leaves larger
connections unexplored. Although coping with a large class size could be similar to
dealing with large portions of history, the inability to give individual students
attention is not the same type of “hazard” the author mentions in historical
studies. The author’s point is that too much attention is given to individual
sections of history at the expense of the bigger picture.
Explanation for Incorrect Answer C :
Choice (C) is incorrect. The author argues in this passage that dividing history into
segments for instructional purposes is always risky. No matter how history is
divided, distortions of emphasis will occur. Focusing on one part of history—an
international period or a single nation’s history, for instance—leaves larger
connections unexplored. A biologist studying large areas of forest at the expense of
focusing on a specific bird species would be the reverse of the “hazard” in
historical studies: the biologist would overlook details because of too wide a focus,
file://E:\新建文件夹\e3.htm
2006-11-12
页码,11/23
The Official SAT Online Course
instead of overlooking the big picture because of a narrow focus.
Explanation for Incorrect Answer D :
Choice (D) is incorrect. The author argues in this passage that dividing history into
segments for instructional purposes is always risky. No matter how history is
divided, distortions of emphasis will occur. Focusing on one part of history—an
international period or a single nation’s history, for instance—leaves larger
connections unexplored. An artist working in many different media would be the
reverse of the “hazard” in historical studies: the artist would be spread too thin
by too wide a focus, instead of missing the bigger picture because of specializing too
narrowly.
12
The author implies which of the following about “war” and “politics” in historical
studies (line 16)?
(A) They make the study of international movements difficult.
(B)
They serve primarily as a convenience to the reader.
(C) They are more helpful to use in defining periods than in defining
movements.
(D) They are equally important to historians and to readers.
!
途
ANSWERS
AND EXPLANATIONS
d
They are commonly used to define historical periods.
re
(E)
用
te
Explanation for Correct Answer E :
Choice (E) is correct. In this passage, the author states that dividing history into
“time periods” may overemphasize “some event or type of activity” commonly
used to mark “the end or beginning of a period or movement.” The author
includes “war or politics” as examples of types of activities that are used to mark
the border of a historical period, that is, “the end or beginning of a period or
movement.”
业
eg
is
商
于
Explanation for Incorrect Answer A :
Choice (A) is incorrect. In this passage, the author mentions several ways in which
history is divided for instructional purposes. “War” and “politics” are used as
examples of events or activities used to define historical periods. While the author
says that all such divisions have their shortcomings, he or she does not imply that
these particular divisions—war and politics—“make the study of international
movements difficult.” Such a division might, it could be argued, make the study of
international movements easier than would, say, division according to nation.
nR
用
禁
U
严
Explanation for Incorrect Answer B :
Choice (B) is incorrect. In this passage, the author mentions several ways in which
history is divided for instructional purposes. “War” and “politics” are used as
examples of events or activities used to define historical periods. While the author
says that all such divisions have their shortcomings, he or she does not imply that
these particular divisions—war and politics—“serve primarily as a convenience to
the reader.” In fact, the author seems to imply that such division may not be a
convenience, but may actually inconvenience the reader by distorting the
importance of particular events.
Explanation for Incorrect Answer C :
Choice (C) is incorrect. In this passage, the author mentions several ways in which
history is divided for instructional purposes. “War” and “politics” are used as
examples of events or activities commonly used to define historical periods. The
author does not imply that these particular divisions—war and politics—are more
useful in defining periods than in defining movements. Rather, the author states
that all such divisions introduce certain hazards and oversights into historical study.
Explanation for Incorrect Answer D :
Choice (D) is incorrect. In this passage, the author mentions several ways in which
history is divided for instructional purposes. “War” and “politics” are used as
examples of events or activities commonly used to define historical periods. The
author does not discuss the importance of these particular divisions—war and
politics—to either the historian who writes the history or the reader who reads
history; the author says only that such divisions are common and potentially
hazardous.
file://E:\新建文件夹\e3.htm
2006-11-12
页码,12/23
The Official SAT Online Course
The following passages discuss a type of film called film noir, which, according to
most film historians, had its high point around the time of the Second World War
(1939-1945).
Passage 1
Even though films now called film
noir by critics have been made in
Hollywood since 1939, film noir as a genre
Line did not exist until 1946. In that year an
5
exhibition of American movies was held in
Paris, and French film critics got their first
look at what had been going on in
Hollywood since the advent of World War
II. Among the films shown were Laura;
10 The Maltese Falcon; Murder, My Sweet;
Double Indemnity; and The Woman in the
Window. Those five films shared enough
traits that critic Nino Frank gave them a
new classification: film noir, or literally,
15 “black film.” The traits they shared were
both stylistic and thematic. They were
dark in both look and mood. Their
primary action took place at night on rainswept city streets, in narrow ash-can
20 alleys, in claustrophobic diners, and in
dingy, shadowy hotel rooms with neon
signs flashing outside the windows, rooms
in which, as hard-boiled author Nelson
Algren once put it, “every bed you rent
25 makes you an accessory to somebody
else’s shady past.” The characters in
these films were bookies, con men, killers,
cigarette girls, crooked cops, down-andout boxers, and calculating, scheming,
30 and very deadly women. The well-lit,
singing and tap-dancing, happy-ending
world of the 1930’s had in ten short years
become a hostile, orderless place in which
alienation, obsession, and paranoia ruled.
35 The universe seemed to conspire to defeat
and entrap the inhabitants who wandered
blindly through it. They were victims of
fate, their own worst enemies who,
looking for a score, ended by defeating
40 themselves.
The five films mentioned earlier
that were shown at the 1946 exhibition
were the ones the French critiqued. These
high-budget studio productions most
45 commonly come to the public’s mind when
the word noir is mentioned because they
er
e
d
!
途
用
st
业
eg
i
商
于
nR
用
禁
U
严
file://E:\新建文件夹\e3.htm
2006-11-12
页码,13/23
The Official SAT Online Course
are cited most often in the spate of
contemporary books that have recently
been published on the subject. But the
50 noir cycle, although kick-started by the
success of those high-budget productions,
actually had its roots in the B movie, in
particular, in the B crime movie. Film noir
was made to order for the B, or low55 budget, part of the movie double bill1. It
was cheaper to produce because it
required less lighting and smaller casts
and usually entailed story lines that
required limited-scale sets—an attractive
60 quality to film studios operating on
reduced wartime budgets. Film noir was
character-driven, and its story lines, which
were unusual and compact, could often be
told in the 60 to 80 minutes required of B
65 pictures.
d
!
途
re
Passage 2
用
It may be that noir began in a way
of photographing that was as economical
as it was moody (less light meant less
money on decor—an important wartime
consideration when studios faced limits on
construction material).
Where did noir come from? It’s an
intriguing question and one still not
adequately answered, despite the quantity
of writing that wallows in that noir mood.
Don’t rule out the influence of German
film from the twenties, if only because
there were, by the early forties, so many
European refugees (writers, directors,
camera operators, designers, actors)
working in Hollywood. Don’t forget the
impact of French films of the late thirties,
especially those of Marcel Carne. His Le
Jour Se Leve (1939, called Daybreak in
the United States) was such a success
that it was remade in Hollywood in 1947
as The Long Night. Finally, don’t
underestimate the influence Citizen Kane
had on anyone whose art and craft was
cinematography. The film was a box
office flop, but filmmakers were absorbed
by it. A landmark in so much, Kane is a
turning point in the opening up of a noir
sensibility.
Equally, don’t forget that from the
te
业
商
is
70
eg
于
nR
用
80
85
90
95
file://E:\新建文件夹\e3.htm
禁
严
U
75
2006-11-12
页码,14/23
The Official SAT Online Course
forties onward, Los Angeles was much
beset by psychoanalysis, and the growing
intellectual interest in guilt, depression,
and nightmare. Don’t eliminate the
100 impact, the memory, or the mere thought
of a war’s damage.
I want to stress how deeply noir
impulses lay in the common imagination—
that of the audience as well as the
105 filmmakers. Mildred Pierce, for instance—
which appeared in 1945 when many
American women were running businesses
of their own just to survive while the men
were away at war—sighs and seems to
110 say, “It doesn’t make any difference, why
bother, for there is something malign in
human nature or luck that will undermine
enterprise and hope.” Of course, not
every film was so bleak. Look instead at
115 David Selznick’s very beautiful and
touching Since You Went Away, which is
all about Claudette Colbert, Jennifer
re
d
!
途
2
用
Jones, and Shirley Temple coping in the
absence of men. That, too, looks like a
120 film noir. But the mood is entirely that of
innocent, ardent, flawless hope, and
assurance that when the war ends
everything will revert to calm and order.
st
e
业
eg
i
商
于
nR
用
1 In the 1940’s, a film showing would typically include
a longer, more expensively-produced film (an “A
film”) and a shorter, less-expensively produced film (a
“B film”)—in other words, a “double bill.”
禁
U
严
2 Film actresses who were popular during the 1940’s.
13
In lines 1 through 4 of Passage 1, the author suggests that “films now called film
noir by critics”
(A)
were not classified as film noir when first made
(B)
were reminiscent of earlier European films
(C) were uplifting in mood and theme
(D) were intended to contrast with films of the 1930’s
(E)
were disliked by many French film critics
ANSWERS
AND EXPLANATIONS
Explanation for Correct Answer A :
Choice (A) is correct. The passage states that while films now classified as film noir
had been made since 1939, “film noir as a genre did not exist until 1946. ” Thus
the “films now called film noir by critics” were not classified as film noir when
they were first made, since the term was not coined until after the Paris exhibition.
file://E:\新建文件夹\e3.htm
2006-11-12
页码,15/23
The Official SAT Online Course
Explanation for Incorrect Answer B :
Choice (B) is incorrect. The first four lines of the passage state that “films now
called film noir” were made before film noir had been recognized as its own genre.
So while the passage does speak of films “earlier than 1946,” nothing is
suggested here about the relationship between these American films and “earlier
European films.”
Explanation for Incorrect Answer C :
Choice (C) is incorrect. The first four lines of the passage state that “films now
called film noir” were made before film noir had been recognized as its own genre.
It stands to reason, then, that since these films were film noir in everything but
name—since the name had not yet been invented—they were anything but
“uplifting in mood and theme.” Rather, they were almost certainly bleak in mood
and theme.
Explanation for Incorrect Answer D :
Choice (D) is incorrect. The first four lines of the passage state that “films now
called film noir” were made before film noir had been recognized as its own genre.
While it is suggested in lines 30-32 that certain films of the 1930’s (“well-lit,
singing and tap-dancing, happy-ending”) did contrast markedly with the film noir,
there is nothing in the passage to indicate that either type of film was intended to
contrast with the other.
!
途
ed
Explanation for Incorrect Answer E :
Choice (E) is incorrect. The first four lines of the passage state that “films now
called film noir” were made before film noir had been recognized as its own genre.
If these films were later termed film noir, a term coined by French critics, it stands
to reason that French critics liked rather than disliked them.
er
用
业
is
t
It can be inferred that the films listed in lines 9 through 12 were similar in each of the
following ways EXCEPT:
(A)
visual appearance
(B)
emotional effect
商
eg
14
于
(C) characters
(D) theme
用
nR
(E)
music
禁
ANSWERS
AND EXPLANATIONS
U
严
Explanation for Correct Answer E :
Choice (E) is correct. The passage discusses many similarities among the five films
mentioned in lines 9 through 12; they “shared enough traits that critic Nino Frank
gave them a new classification.” The passage states that the films were similar in
“look and mood,” that they shared “thematic” traits, and that many of them
featured the same types of characters. While the music in these films may or may
not be similar, music is not discussed in the passage, and so no inferences can be
drawn from the passage about it.
Explanation for Incorrect Answer A :
Choice (A) is incorrect. The passage states that the five films mentioned in lines 9
through 12 were “dark in both look and mood,” which implies that they were
indeed similar in visual appearance.
Explanation for Incorrect Answer B :
Choice (B) is incorrect. The passage states that the five films mentioned in lines 9
through 12 were “dark in both look and mood”; further, the world of these films
was “a hostile, orderless place in which alienation, obsession, and paranoia
ruled.” Together, these descriptions imply that the films were indeed similar in
emotional effect.
Explanation for Incorrect Answer C :
Choice (C) is incorrect. The passage explicitly states that the films shared similar
types of characters. In lines 26- 29, several character types are listed, including
file://E:\新建文件夹\e3.htm
2006-11-12
页码,16/23
The Official SAT Online Course
“crooked cops, down-and-out boxers, and calculating, scheming, and very deadly
women.”
Explanation for Incorrect Answer D :
Choice (D) is incorrect. The passage states in line 16 that the traits the films shared
“were both stylistic and thematic”—in other words, that they had similarities in
theme.
15
In paragraph 2, the author says that the films discussed in paragraph 1 are NOT
typical of their genre in regard to
(A)
setting
(B)
budget
(C) country of origin
(D) plot
(E)
lighting
ANSWERS
AND EXPLANATIONS
!
途
re
d
Explanation for Correct Answer B :
Choice (B) is correct. Paragraph 2 discusses the budgets of film noir films. The
author argues that although the “high-budget productions” mentioned in the first
paragraph helped kick-start film noir, the roots of film noir were in B movies. B
movies, the passage states, were “low-budget,” “cheaper to produce” films
used as the less expensive half of double features. The higher budget is the only
difference presented here between the films mentioned in paragraph 1 and typical
films of the film noir genre.
te
用
业
is
商
Explanation for Incorrect Answer A :
Choice (A) is incorrect. Although paragraph 2 states that typical film noir “entailed
story lines that required limited-scale sets,” this is not mentioned as a difference
between the typical film noir and the films mentioned in paragraph 1. The author’
s point is not that these settings were different from those of the films mentioned,
but that film noir did not, in general, require a large budget to produce.
nR
eg
于
用
Explanation for Incorrect Answer C :
Choice (C) is incorrect. Paragraph 2 says nothing about film noir’s country of
origin, though the passage as a whole calls film noir an American phenomenon first
named by French film critics. Rather, paragraph 2 discusses the low budgets of the
typical film noir.
禁
U
严
Explanation for Incorrect Answer D :
Choice (D) is incorrect. Although paragraph 2 states that typical film noir plots were
“character-driven” and “unusual and compact,” this is not mentioned as a
difference between the typical film noir and the films mentioned in paragraph 1.
The author’s point is not that these plots were different from those of the films
mentioned, but that film noir did not, in general, require a high budget to produce.
Explanation for Incorrect Answer E :
Choice (E) is incorrect. Paragraph 2 says nothing about film noir’s lighting, though
the passage as a whole indicates that the typical film noir was dimly lighted.
Rather, paragraph 2 discusses the low budgets of typical film noir.
16
The author of Passage 1 uses the quotation in lines 24-26 primarily in order to
(A)
critique a writer
(B)
recount an incident
(C) evoke a place
(D) describe a character
(E)
file://E:\新建文件夹\e3.htm
summarize a plot
2006-11-12
页码,17/23
The Official SAT Online Course
ANSWERS
AND EXPLANATIONS
Explanation for Correct Answer C :
Choice (C) is correct. In the second half of the first paragraph, the author describes
the characteristics of film noir, including setting: the “claustrophobic diners” and
“dingy, shadowy hotel rooms” where film noir is commonly set. The author
quotes Nelson Algren’s description of the hotels in which “every bed you rent
makes you an accessory to somebody else’s shady past” in order to evoke
strongly the typical places where action occurs in film noir.
Explanation for Incorrect Answer A :
Choice (A) is incorrect. In the second half of the first paragraph, the author
describes the characteristics of film noir, including setting. The author uses Nelson
Algren’s quotation to help evoke the types of places typically found in film noir.
There is no critique of Algren’s writing. In fact, the author seems to agree
wholeheartedly with Algren’s description.
Explanation for Incorrect Answer B :
Choice (B) is incorrect. In the second half of the first paragraph, the author
describes the characteristics of film noir, including setting. The author uses Nelson
Algren’s quotation to help evoke the types of places found in film noir. The quote
does not recount a specific incident, but, rather, refers to film noir settings in
general.
d
!
途
re
Explanation for Incorrect Answer D :
Choice (D) is incorrect. In the second half of the first paragraph, the author
describes the characteristics of film noir, including setting. The author uses Nelson
Algren’s quotation to help evoke the type of places generally found in film noir,
not to describe any specific character, though character types are discussed in the
first paragraph of the passage.
te
用
业
Explanation for Incorrect Answer E :
Choice (E) is incorrect. In the second half of the first paragraph, the author
describes the characteristics of film noir, including setting. The author uses Nelson
Algren’s quotation to help evoke the types of places generally found in film noir,
not to summarize the plot of any specific film.
eg
is
商
于
用
nR
17
The author of Passage 1 suggests that the “spate of contemporary books” (lines 4748) on film noir
禁
(A)
discusses only five films
(B)
focuses on non-crime films
U
严
(C) focuses on relatively costly noir films
(D) is inaccurate in their historical data
(E)
is from big-budget publishing companies
ANSWERS
AND EXPLANATIONS
Explanation for Correct Answer C :
Choice (C) is correct. The author states that the “spate of contemporary books”
on film noir cites “most often” the five films “shown at the 1946 exhibition.”
These five films, consequently, have dominated public perception of film noir. The
author argues that this perception of film noir may be inaccurate, however, because
the films most discussed in the books were much more costly to produce than were
most film noir films. According to the author, film noir films were very often “lowbudget” B movies.
Explanation for Incorrect Answer A :
Choice (A) is incorrect. The author states that the “spate of contemporary books”
on film noir cites “most often” the five films “shown at the 1946 exhibition.”
The author does not state that these five films are the only films discussed in
contemporary writing about film noir.
file://E:\新建文件夹\e3.htm
2006-11-12
页码,18/23
The Official SAT Online Course
Explanation for Incorrect Answer B :
Choice (B) is incorrect. The author states that the “spate of contemporary books”
on film noir cites “most often” the five films “shown at the 1946 exhibition.”
Although the author states that the roots of film noir were in “the B crime movie”
and that the five high-budget films are uncharacteristic, this does not imply that the
five films were not crime films themselves.
Explanation for Incorrect Answer D :
Choice (D) is incorrect. The author states that the “spate of contemporary books”
on film noir cites “most often” the five films “shown at the 1946 exhibition.”
Although the author argues that focusing on these five films produces a slightly
inaccurate understanding of film noir, the historical data is not termed inaccurate.
The author agrees that these films were shown at the exhibition and that the
exhibition was vital to the establishment of film noir as a genre.
Explanation for Incorrect Answer E :
Choice (E) is incorrect. Although the author states that the “spate of contemporary
books” focuses on films that were “high-budget studio productions,” nothing is
said in the passage about the companies that published the books.
18
The authors of both passages imply that contemporary writing about film noir
(A)
comes mainly from French film critics
(B)
has failed to describe the origins of film noir accurately
!
途
ed
(C) mischaracterizes the film noir mood
(D) is inferior to earlier writing on film noir
用
has dramatically improved the understanding of film noir
业
AND EXPLANATIONS
eg
is
t
ANSWERS
er
(E)
Explanation for Correct Answer B :
Choice (B) is correct. Both authors argue that contemporary writing on film noir
fails to describe its origins accurately. The author of Passage 1 argues that “the
spate of contemporary books” on film noir inaccurately portrays film noir’s
origins by focusing on five “high-budget” films “shown at the 1946 exhibition.”
According to the author, this focus de-emphasizes the fact that film noir “actually
had its roots in the B movie,” which were much lower-budget productions. The
author of Passage 2 states that “the quantity of writing that wallows in that noir
mood” has “not adequately answered” the question, “Where did noir come
from?” Thus, both authors think that many modern writers ignore or miss the
point about the real origins of film noir.
商
于
nR
用
禁
U
严
Explanation for Incorrect Answer A :
Choice (A) is incorrect. Neither author mentions the national origin of contemporary
writing about film noir. The author of Passage 1 discusses the importance French
film critics played in establishing the film noir genre in 1946, but says nothing in
specific about contemporary writing by French critics.
Explanation for Incorrect Answer C :
Choice (C) is incorrect. Both authors state that contemporary writing about film noir
mischaracterizes the origins of film noir, not the mood of film noir. The authors
seem to agree on the dark “mood” of film noir.
Explanation for Incorrect Answer D :
Choice (D) is incorrect. While both authors criticize contemporary writing as
wrongheaded about the origins of film noir, neither author compares the quality of
this writing with that of earlier writing.
Explanation for Incorrect Answer E :
Choice (E) is incorrect. Neither author states that contemporary writing has
improved the understanding of film noir. On the contrary, they both argue that
contemporary writing has inadequately described the origins of film noir.
19
file://E:\新建文件夹\e3.htm
Both passages imply that the development of film noir can be attributed in part to
2006-11-12
页码,19/23
The Official SAT Online Course
(A)
the presence of European filmmakers in Hollywood
(B)
the influential writing of French film critics
(C) economic restraints resulting from World War II
(D) United States filmmakers’ dissatisfaction with high-paid actors
(E)
the popularity of Citizen Kane
ANSWERS
AND EXPLANATIONS
Explanation for Correct Answer C :
Choice (C) is correct. Both passages highlight the relatively low cost of making
most film noir films. They both state also that this low cost was central to film noir
production because low cost was consistent with economic restraints resulting from
World War II. Passage 1 states that the “limited-scale sets” of film noir were
“an attractive quality to film studios operating on reduced wartime budgets.”
Passage 2 states that the low lighting that contributed to the mood of film noir was
also cheaper, “an important wartime consideration when studios faced limits on
construction material.”
!
途
re
d
Explanation for Incorrect Answer A :
Choice (A) is incorrect. Although Passage 2 claims the presence of European
filmmakers in Hollywood was influential in the development of film noir, Passage 1
speaks of French film critics but makes no mention of European filmmakers in
Hollywood.
用
te
Explanation for Incorrect Answer B :
Choice (B) is incorrect. Although Passage 1 discusses the importance of French film
critics in the establishment of film noir as a genre, Passage 2 speaks of European
film directors but does not mention French film critics at all.
业
商
is
Explanation for Incorrect Answer D :
Choice (D) is incorrect. Neither passage discusses high-paid actors or United States
filmmakers’ attitudes towards them.
nR
eg
于
Explanation for Incorrect Answer E :
Choice (E) is incorrect. Although Passage 2 states that Citizen Kane was influential
in the development of film noir, Passage 1 does not mention Citizen Kane at all.
用
20
禁
The phrase “wallows in” in line 75 is closest in meaning to
U
严
(A)
indulges in
(B)
conforms to
(C) criticizes
(D) explores
(E)
reveals
ANSWERS
AND EXPLANATIONS
Explanation for Correct Answer A :
Choice (A) is correct. In lines 72-75, Passage 2 discusses the abundant material
written about film noir. The author is saying that writers have failed to explain
accurately the origins of film noir, despite the large quantity of writing that
“wallows in,” or is excessively involved with, film noir. In other words, the
writing has been insufficient despite its indulgence in the topic.
Explanation for Incorrect Answer B :
Choice (B) is incorrect. In lines 72-75, Passage 2 discusses the abundant material
written about film noir. The passage is not arguing that the writing “conforms
to,” or shares, the film noir mood, but that it has insufficiently explained film
noir’s origins despite its excessive involvement with the topic.
file://E:\新建文件夹\e3.htm
2006-11-12
页码,20/23
The Official SAT Online Course
Explanation for Incorrect Answer C :
Choice (C) is incorrect. In lines 72-75, Passage 2 discusses the abundant material
written about film noir. The passage is not stating that the writing “criticizes,” or
disapproves of, film noir, but that it has insufficiently explained film noir’s origins
despite its excessive involvement with the topic.
Explanation for Incorrect Answer D :
Choice (D) is incorrect. In lines 72-75, Passage 2 discusses the abundant material
written about film noir. The passage is not simply stating that this material
“explores” film noir, but that is has insufficiently explained film noir’s origins,
despite its excessive involvement with the topic.
Explanation for Incorrect Answer E :
Choice (E) is incorrect. In lines 72-75, Passage 2 discusses the abundant material
written about film noir. The passage is not stating that this material has
“revealed” or exposed, anything about film noir. On the contrary, it states that
the writing has failed to sufficiently explain film noir’s origins despite its excessive
involvement with the topic.
21
The author of Passage 2 mentions Since You Went Away in line 116 primarily to
(A)
indicate that not all films in the 1940’s expressed the same attitude
(B)
illustrate the popularity of film noir actors
!
途
(C) demonstrate that most film noir dealt with WWII
ANSWERS
re
point out David Selznick’s influence in defining film noir
用
AND EXPLANATIONS
is
te
(E)
d
(D) show that the “absence of men” was a major film noir theme
业
Explanation for Correct Answer A :
Choice (A) is correct. The first half of the fifth paragraph discusses the apathetic
and hopeless mood of many film noir films. The author then mentions Since You
Went Away to point out that not every film of the time “was so bleak.” The
author argues that Since You Went Away “looks like a film noir,” even though its
mood is “entirely that of innocent, ardent, flawless hope,” an attitude quite
different from film noir films previously discussed in the passage.
eg
商
于
nR
用
禁
Explanation for Incorrect Answer B :
Choice (B) is incorrect. Although the author mentions that Since You Went Away
starred certain popular film actors, the author’s point is that the innocent and
hopeful attitude of Since You Went Away is different from the “bleak” mood of
other films mentioned in the passage. The film’s cast is not necessarily relevant to
the argument about the film’s different attitude.
U
严
Explanation for Incorrect Answer C :
Choice (C) is incorrect. Although the author states that Since You Went Away deals
with women during World War II, the author’s point is that that the innocent and
hopeful attitude of Since You Went Away is different from the “bleak” mood of
other films mentioned in the passage. The author does not say that any other film
noir films dealt with World War II, though it is likely that many did.
Explanation for Incorrect Answer D :
Choice (D) is incorrect. Although the author states that Since You Went Away
portrayed women dealing with the “absence of men” during World War II, the
author’s point is that the innocent and hopeful attitude of Since You Went Away is
different from the “bleak” mood of other films mentioned in the passage. The
author does not say that any other film noir films dealt with “the absence of
men.”
Explanation for Incorrect Answer E :
Choice (E) is incorrect. The author does not state that David Selznick was influential
in defining film noir, but only that Selznick’s film Since You Went Away had a
different, more hopeful mood, than the “bleak” mood of other films mentioned in
the passage.
file://E:\新建文件夹\e3.htm
2006-11-12
页码,21/23
The Official SAT Online Course
22
The second paragraph of Passage 2 is best described as
(A)
a catalog of possible sources of the film noir mood
(B)
a description of the mood that characterizes film noir
(C) a refutation of several misconceptions about film noir
(D) an argument for redefining the term “film noir”
(E)
a list of the major film noir films
ANSWERS
AND EXPLANATIONS
Explanation for Correct Answer A :
Choice (A) is correct. Paragraph 2 begins with the question, “Where did noir come
from?” The paragraph then suggests different possible sources for film noir by
exhorting the reader not to “rule out” or “forget,” or “underestimate”
various influences. The paragraph catalogs “German film from the twenties,”
“French films of the late thirties,” and “Citizen Kane” as influential in the
formation of the film noir genre.
!
途
ed
Explanation for Incorrect Answer B :
Choice (B) is incorrect. Paragraph 2 does not address the film noir mood
specifically. Instead, it lists possible answers to the question, “Where did noir
come from?” The paragraph presents sources of film noir, not elements of its
mood.
Explanation for Incorrect Answer C :
Choice (C) is incorrect. Paragraph 2 does not address misconceptions about film
noir. The paragraph states that determining the origins of film noir is difficult, and
then lists possible sources for consideration. The sources are presented as
possibilities, not as misconceptions.
te
r
用
业
Explanation for Incorrect Answer D :
Choice (D) is incorrect. Paragraph 2 does not attempt to define the term “film
noir.” The paragraph assumes a general definition of film noir, then lists, or
catalogs, several possible sources of film noir.
eg
is
商
于
Explanation for Incorrect Answer E :
Choice (E) is incorrect. Although paragraph 2 does mention several films, these
films are not presented as major film noir films. Instead, the paragraph lists them
as possible sources for the style that became film noir.
nR
用
禁
严
U
23
The quote “It doesn’t make…enterprise and hope” in lines 110-113 is primarily
meant to
(A)
give an example of dialogue from Mildred Pierce
(B)
summarize what critics thought about Mildred Pierce
(C) show how Mildred Pierce differs from other films made in 1945
(D) characterize the sentiments expressed in Mildred Pierce
(E)
demonstrate the influence of Mildred Pierce on Since You Went Away
ANSWERS
AND EXPLANATIONS
Explanation for Correct Answer D :
Choice (D) is correct. The passage argues that the “noir impulses” reflected the
common sentiments of people in general during the 1940’s. The quotation in lines
110-113 characterizes the attitude expressed by the film Mildred Pierce. This
attitude, the passage argues, was present in the imagination of many people at that
time.
Explanation for Incorrect Answer A :
Choice (A) is incorrect. The quotation in lines 110-113 is not actual dialogue from
Mildred Pierce. The quotation is, according to the author, a characterization of the
file://E:\新建文件夹\e3.htm
2006-11-12
页码,22/23
The Official SAT Online Course
general attitude the film presents. This attitude, the passage argues, was shared by
many people during the 1940’s.
Explanation for Incorrect Answer B :
Choice (B) is incorrect. The quotation in lines 110-113 does not refer to comments
critics made about Mildred Pierce. The quotation is, according to the author, a
characterization of the general attitude the film presents. This attitude, the passage
argues, was shared by many people during the 1940’s.
Explanation for Incorrect Answer C :
Choice (C) is incorrect. The quotation is a characterization of the general attitude
the film presents—an attitude, the passage argues, that was shared by many
people during the 1940’s. Although the passage does state that “not every film
was so bleak,” the paragraph argues that this attitude was the characteristic of
many film noir films. The quotation is used to express a general sentiment in many
film noir films, not a difference between Mildred Pierce and other films.
Explanation for Incorrect Answer E :
Choice (E) is incorrect. Although the paragraph discusses both Mildred Pierce and
Since You Went Away, the quotation is used to help distinguish between the two
films. The quotation in lines 110-113 characterizes the attitude expressed by the
film Mildred Pierce. This attitude, the passage argues, was present in the
imagination of many people at that time. The paragraph explicitly states that this
attitude was NOT shared by Since You Went Away.
ed
!
途
The word “impulses” in line 103 is closest in meaning to
(A)
incentives
(B)
stimulants
用
er
24
(C) fantasies
(E)
feelings
商
AND EXPLANATIONS
eg
ANSWERS
业
is
t
(D) transformations
于
Explanation for Correct Answer E :
Choice (E) is correct. The passage argues that “noir impulses” reflected the
common sentiments of people in the 1940’s. The author implies that these
“impulses,” or feelings, were shared by both filmmakers and filmgoers and that
such feelings were present “in the common imagination” of the time. The
quotation that the author uses to support this point—lines 110-113—is an
expression of a pessimistic feeling about life.
nR
用
禁
U
严
Explanation for Incorrect Answer A :
Choice (A) is incorrect. The passage argues that the “noir impulses” reflected the
common sentiments of people in the 1940’s. These “impulses” were the
feelings people had about their lives, not “incentives,” or motivating factors, to
impel them to do something.
Explanation for Incorrect Answer B :
Choice (B) is incorrect. The passage argues that the “noir impulses” reflected the
common sentiments of people in the 1940’s. These “impulses” were the
feelings people had about their lives, not “stimulants” or factors that would
stimulate them to do something.
Explanation for Incorrect Answer C :
Choice (C) is incorrect. The passage argues that the “noir impulses” reflected the
common sentiments of people in the 1940’s. These “impulses” were the
feelings, not “fantasies,” many people had about their lives. In fact, the grim
feelings expressed in the paragraph are the opposite of what is usually thought of
as a fantasy, or pleasant notion or daydream.
Explanation for Incorrect Answer D :
Choice (D) is incorrect. The passage argues that the “noir impulses” reflected the
common sentiments of people in the 1940’s. These “impulses” were the
feelings people had about their lives, not any type of “transformation,” or
profound change, they might have experienced.
file://E:\新建文件夹\e3.htm
2006-11-12
页码,23/23
The Official SAT Online Course
Back to Score Report
Privacy Policy
Copyright © 2006 The College Board. All rights reserved.
Terms of Use
Contact Us
re
d
!
途
用
te
业
is
商
nR
eg
于
用
禁
U
严
file://E:\新建文件夹\e3.htm
2006-11-12
页码,1/17
The Official SAT Online Course
Help | Profile | My Organizer | My Bookmarks | Logout
Answers and Explanations
Back to Score Report
Test Sections
Section 1
View Answers and Explanations
Section 2
Online - Practice Test #5
Section 3
Section 4
1
does NOT satisfy
Which of the following triples
Section 5
Section 6
(A)
Section 8
(B)
Section 9
(C)
Section 10
(D)
(E)
!
途
AND EXPLANATIONS
ed
ANSWERS
Explanation for Correct Answer C :
用
then
Choice (C) is correct. If
er
Therefore, the triple
satisfy
does not
is
t
业
商
Explanation for Incorrect Answer A :
eg
于
Choice (A) is not correct. If
then
The question asks which triple does NOT satisfy
nR
用
Explanation for Incorrect Answer B :
禁
then
Choice (B) is not correct. If
The question asks which triple does NOT satisfy
U
严
Explanation for Incorrect Answer D :
then
Choice (D) is not correct. If
The question asks which triple does NOT
satisfy
Explanation for Incorrect Answer E :
then
Choice (E) is not correct. If
The question asks which triple does NOT
satisfy
2
Susan had
trading cards. After giving away
cards and receiving
cards, she
cards. What is the value of
had
(A)
(B)
(C)
file://E:\新建文件夹\e4.htm
2006-11-12
页码,2/17
The Official SAT Online Course
(D)
(E)
ANSWERS
AND EXPLANATIONS
Explanation for Correct Answer A :
trading cards. After giving away
Choice (A) is correct. Susan had
Subtracting
cards, it follows that
Susan ended up with
cards, she
cards. Since
cards, she had
cards; then, after receiving
had
from each side of this equation and rearranging gives
Explanation for Incorrect Answer B :
Choice (B) is not correct. The value of
is
not
Explanation for Incorrect Answer C :
Choice (C) is not correct. The value of
is
not
Explanation for Incorrect Answer D :
Choice (D) is not correct. The value of
is
Explanation for Incorrect Answer E :
Choice (E) is not correct. The value of
is
!
途
not
re
d
not
业
is a positive integer divisible by
If
value of
(B)
(C)
what is the greatest possible
商
于
用
nR
(D)
and if
eg
is
(A)
用
te
3
(E)
禁
ANSWERS
U
严
AND EXPLANATIONS
Explanation for Correct Answer B :
Choice (B) is correct. If
In addition,
then
is divisible by
satisfies the condition
that are divisible by
and less than
than
that satisfies both conditions.
possible integer
because
There are no integers greater
Therefore,
Explanation for Incorrect Answer A :
is not divisible by
Choice (A) is not correct. The integer
cannot be the greatest possible integer less than
file://E:\新建文件夹\e4.htm
Therefore,
that is divisible by
Explanation for Incorrect Answer E :
is not divisible by
Choice (E) is not correct. The integer
cannot be the greatest possible integer less than
Therefore,
that is divisible by
Explanation for Incorrect Answer D :
is not divisible by
Choice (D) is not correct. The integer
cannot be the greatest possible integer less than
Therefore,
that is divisible by
Explanation for Incorrect Answer C :
is not divisible by
Choice (C) is not correct. The integer
cannot be the greatest possible integer less than
is the greatest
Therefore,
that is divisible by
2006-11-12
页码,3/17
The Official SAT Online Course
4
!
途
er
ed
for the
and Company
The graph above shows the monthly sales for Company
What was the largest one-month decrease in sales for Company
second half of
during this period of time?
用
(A)
(B)
业
is
t
(C)
(D)
商
(E)
于
eg
ANSWERS
AND EXPLANATIONS
用
nR
Explanation for Correct Answer E :
Choice (E) is correct. The sales for Company
are represented by the dashed line,
so the sales for the six months for that company were
禁
and
month decrease in sales for Company
U
严
Therefore, the largest one-
was
Explanation for Incorrect Answer A :
Choice (A) is not correct. This choice,
was the difference in sales
for the month of August. The question asks
and Company
between Company
for the largest one-month decrease in sales for Company
Explanation for Incorrect Answer B :
Choice (B) is not correct. This choice,
increase in sales for Company
decrease in sales for Company
was the largest one-month
The question asks for the largest one-month
Explanation for Incorrect Answer C :
Choice (C) is not correct. This choice,
decrease in sales for Company
was the largest one-month
The question asks for the largest one-month
decrease in sales for Company
Explanation for Incorrect Answer D :
Choice (D) is not correct. This choice,
increase in sales for Company
was the largest one-month
The question asks for the largest one-month
decrease in sales for Company
file://E:\新建文件夹\e4.htm
2006-11-12
页码,4/17
The Official SAT Online Course
5
For parties, the number of cases of juice a caterer orders is directly proportional to
cases for a party with
the number of people attending. If the caterer orders
people
people attending, how many cases would she order for a party with
attending?
(A)
(B)
(C)
(D)
(E)
ANSWERS
AND EXPLANATIONS
Explanation for Correct Answer E :
represent the number of cases of juice the caterer
Choice (E) is correct. Let
people attending. Then
and
!
途
re
d
would order for a party with
用
Explanation for Incorrect Answer A :
Choice (A) is not correct. This choice is
te
业
but the correct number of cases for
people is
is
商
Explanation for Incorrect Answer B :
represent the number of cases of juice the caterer
Choice (B) is not correct. Let
eg
于
would order for a party with
nR
用
people attending. Then
and
not
禁
Explanation for Incorrect Answer C :
Choice (C) is not correct. This choice is
U
严
for
but the correct number of cases
people is
Explanation for Incorrect Answer D :
represent the number of cases of juice the caterer
Choice (D) is not correct. Let
would order for a party with
people attending. Then
and
not
6
Which of the following is an element of both the set of positive odd integers and the
set of prime numbers?
(A)
(B)
(C)
(D)
(E)
file://E:\新建文件夹\e4.htm
2006-11-12
页码,5/17
The Official SAT Online Course
ANSWERS
AND EXPLANATIONS
Explanation for Correct Answer B :
is odd. In addition,
Choice (B) is correct. The integer
is greater than
is a prime number.
so
has no positive integer factors other than itself and
and
is an element of both the set of positive odd integers and the set of
Therefore,
prime numbers.
Explanation for Incorrect Answer A :
is even, not odd.
Choice (A) is not correct. The number
Explanation for Incorrect Answer C :
Choice (C) is not correct. Since
the number
Explanation for Incorrect Answer D :
Choice (D) is not correct. The number
is even, not odd. In addition, since
the number
is not prime.
is not prime.
!
途
is not prime.
the number
ed
Explanation for Incorrect Answer E :
Choice (E) is not correct. Since
7
用
er
feet more than its width. If the length of the
The length of a rectangular garden is
feet, what is the area of the garden in square feet?
garden is
业
is
t
(A)
商
(B)
(C)
(E)
于
eg
(D)
nR
用
ANSWERS
禁
AND EXPLANATIONS
Explanation for Correct Answer C :
Choice (C) is correct. The length of the garden is
U
严
feet. Since the length is
more than the width, it follows that the width of the garden is
feet
feet. Therefore,
the area of the garden, in square feet, is
Explanation for Incorrect Answer A :
Choice (A) is not correct. The length of the garden is
garden is
garden, in square feet, is
feet. The width of the
feet. Therefore, the area of the
feet less than the length, or
not
Explanation for Incorrect Answer B :
Choice (B) is not correct. The width of the garden is
feet, but the length is
not
feet. Therefore, the area of the garden, in square feet, is
Explanation for Incorrect Answer D :
Choice (D) is not correct. The length of the garden is
feet, but the width is
feet. Therefore, the area of the garden, in square feet, is
not
Explanation for Incorrect Answer E :
Choice (E) is not correct. The length of the garden is
garden is
feet less than the length (
feet), not
Therefore, the area of the garden, in square feet, is
file://E:\新建文件夹\e4.htm
feet. The width of the
feet more (
feet).
not
2006-11-12
页码,6/17
The Official SAT Online Course
8
On the number line above, the tick marks are equally spaced. What is the value of
(A)
(B)
(C)
(D)
(E)
!
途
AND EXPLANATIONS
ed
ANSWERS
Explanation for Correct Answer E :
From
and
is the distance between
Choice (E) is correct. The value of
the figure, we can see that this is the same as the distance between two adjacent
into
to
tick marks. Since the equally spaced tick marks divide the interval from
er
用
equal subintervals, the distance between two adjacent tick marks is
eg
is
t
业
商
Explanation for Incorrect Answer A :
Choice (A) is not correct. The value of
于
is the distance between two adjacent
tick marks. Since the equally spaced tick marks divide the interval from
to
into
用
nR
equal subintervals, the distance between two adjacent tick marks is
禁
not
严
U
Explanation for Incorrect Answer B :
Choice (B) is not correct. The value of
is the distance between two adjacent
tick marks. Since the equally spaced tick marks divide the interval from
to
into
equal subintervals, the distance between two adjacent tick marks is
not
Explanation for Incorrect Answer C :
Choice (C) is not correct. This choice is the value of
the value of
Explanation for Incorrect Answer D :
Choice (D) is not correct. The value of
but the question asks for
is the distance between two adjacent
tick marks. Since the equally spaced tick marks divide the interval from
to
into
equal subintervals, the distance between two adjacent tick marks is
not
This choice could be the result of miscounting the subintervals between
and
9
file://E:\新建文件夹\e4.htm
Jamal has some coins in his pocket. Some of these coins are quarters, and none of
2006-11-12
页码,7/17
The Official SAT Online Course
Which of the following must
the quarters in his pocket are dated earlier than
be true?
(A)
None of the coins in Jamal’s pocket are dated earlier than
(B)
Some of the coins in Jamal’s pocket are dated earlier than
(C) Some of the coins in Jamal’s pocket are dated
or later.
(D) Most of the coins in Jamal’s pocket are either quarters or dated earlier
than
(E)
Most of the coins in Jamal’s pocket are not quarters.
ANSWERS
AND EXPLANATIONS
Explanation for Correct Answer C :
Choice (C) is correct. Jamal has some quarters in his pocket, and none of them are
Therefore, all of the quarters in his pocket must be dated
dated earlier than
or later. This means that the statement “Some of the coins in Jamal’s
or later” MUST be true.
pocket are dated
!
途
ed
Explanation for Incorrect Answer A :
Choice (A) is not correct. It may be true that some of the coins in Jamal’s pocket
are not quarters. We know that none of the quarters in Jamal’s pocket are dated
but we have no information about the other coins. Therefore,
earlier than
the statement “None of the coins in Jamal’s pocket are dated earlier than
” is NOT necessarily true.
er
用
Explanation for Incorrect Answer B :
Choice (B) is not correct. We know that none of the quarters in Jamal’s pocket are
业
st
and we know that Jamal may have some coins in his
dated earlier than
pocket that are not quarters. We know that none of the quarters are dated earlier
and it is possible than none of the other coins are dated earlier than
than
商
eg
i
either. Therefore, the statement “Some of the coins in Jamal’s pocket are
” is NOT necessarily true.
于
dated earlier than
Explanation for Incorrect Answer D :
Choice (D) is not correct. We know that Jamal may have some coins in his pocket
that are not quarters, but we do not know how many coins are quarters and how
many coins are not quarters. We also do not know anything about the dates of the
coins that are not quarters. Therefore, the statement “Most of the coins in
” may or may not
Jamal’s pocket are either quarters or dated earlier than
be true.
nR
用
禁
U
严
Explanation for Incorrect Answer E :
Choice (E) is not correct. Although we know that Jamal has some quarters in his
pocket and possibly some coins that are not quarters, we do not know how many of
the coins are quarters and how many are not quarters. Therefore, the statement
“Most of the coins in Jamal’s pocket are not quarters” is NOT necessarily true.
10
The circumference of the circle with center
of the shaded region?
shown above is
What is the area
(A)
file://E:\新建文件夹\e4.htm
2006-11-12
页码,8/17
The Official SAT Online Course
(B)
(C)
(D)
(E)
ANSWERS
AND EXPLANATIONS
Explanation for Correct Answer B :
Choice (B) is correct. The circumference of a circle is
has diameter
the circle, which has circumference
times its diameter. Thus,
Hence, the radius of the
Since the two diameters
and the area of the entire circle is
circle is
of the circle.
in the figure are perpendicular, the shaded region is
!
途
Choice (A) is not correct.
用
te
r
Explanation for Incorrect Answer A :
ed
Therefore, the area of the shaded region is
is one quarter of the circumference, but the question
业
asks for the area of one quarter of the circle.
Explanation for Incorrect Answer C :
is the radius of the circle, but the question asks for the
Choice (C) is not correct.
area of one quarter of the circle.
eg
is
商
于
Explanation for Incorrect Answer D :
Choice (D) is not correct.
用
is one quarter of the diameter, but the question asks
nR
for the area of one quarter of the circle.
禁
Explanation for Incorrect Answer E :
Choice (E) is not correct. The shaded area is one quarter of the circle, but since the
严
U
circle has area
the area of one quarter of the circle is
11
and
If
be true?
which of the following must
are positive integers, and if
(A)
(B)
(C)
(D)
(E)
ANSWERS
AND EXPLANATIONS
Explanation for Correct Answer D :
Choice (D) is correct. If the product of two numbers is
numbers must be equal to
Thus, if
is a positive integer,
(or both). Since
Therefore, it must be true that
file://E:\新建文件夹\e4.htm
then at least one of the
then either
or
It follows that
2006-11-12
页码,9/17
The Official SAT Online Course
Explanation for Incorrect Answer A :
and
Choice (A) is not correct. If
then
and
are positive
and
are positive
and
are positive
and
are positive
yet it is not true that
integers such that
Explanation for Incorrect Answer B :
and
Choice (B) is not correct. If
then
yet it is not true that
integers such that
Explanation for Incorrect Answer C :
and
Choice (C) is not correct. If
then
yet it is not true that
integers such that
Explanation for Incorrect Answer E :
and
Choice (E) is not correct. If
then
yet it is not true that
integers such that
12
re
d
!
途
用
te
业
is divided into three nonoverlapping
In the figure above, regular pentagon
triangles. Which of the following is true about the three triangles?
is
商
(A)
They have equal areas.
(B)
They have equal perimeters.
eg
于
(C) They are similar.
用
nR
(D) They are isosceles.
(E)
They each have at least one angle of measure
禁
ANSWERS
U
严
AND EXPLANATIONS
Explanation for Correct Answer D :
Choice (D) is correct. A regular pentagon has five sides of equal length, so
have
are each isosceles, and the two sides of equal length in
and
The five angles of a
the same length as the two sides of equal length in
has the same measure as
regular pentagon are of equal measure, so
and
are congruent. Therefore,
and
Thus,
is also isosceles.
Explanation for Incorrect Answer A :
and
Choice (A) is not correct.
Let
on
Explanation for Incorrect Answer B :
and
Choice (B) is not correct.
the longest side of
file://E:\新建文件夹\e4.htm
is greater.
is the longest
has the same length as
be the point such that
a side of the pentagon. It can be shown that
Therefore,
perimeter of
so
is the largest angle of
is greater.
side of
have the same area, but the area of
is congruent to
are of equal perimeter, but the
is the largest angle of
The perimeter of
so
is
is
2006-11-12
页码,10/17
The Official SAT Online Course
is
whereas the perimeter of
Explanation for Incorrect Answer C :
and
Choice (C) is not correct.
are similar; in fact, they are
is not similar to the other two triangles. The angles of
congruent. But
whereas the other two triangles have
and
are of measure
and
angles of measure
Explanation for Incorrect Answer E :
Choice (E) is not correct. None of the three triangles has any angles of measure
13
which of the following must be true?
or
If
I.
II.
!
途
III only
(B)
I and II only
用
业
is
te
(A)
re
d
III.
(C) I and III only
(D) II and III only
商
I, II, and III
于
nR
eg
(E)
ANSWERS
AND EXPLANATIONS
用
Explanation for Correct Answer B :
Choice (B) is correct. If
禁
or
In addition, by the
then
Therefore, statements I
definition of absolute value, it is also true that
and II must both be true.
U
严
However, statement III could be false. The value of
but it could be less. For example, if
then
could be greater than
but
Explanation for Incorrect Answer A :
Choice (A) is not correct. It is not true that
of
could be greater than
must be greater than
The value
but it could be less. For example, if
then
but
Explanation for Incorrect Answer C :
Choice (C) is not correct. Statement I must be true, but statement II must also be
true, and statement III could be false. The value of
but it could be less. For example, if
then
could be greater than
but
Explanation for Incorrect Answer D :
Choice (D) is not correct. Statement II must be true, but statement I must also be
true, and statement III could be false. The value of
but it could be less. For example, if
file://E:\新建文件夹\e4.htm
then
could be greater than
but
2006-11-12
页码,11/17
The Official SAT Online Course
Explanation for Incorrect Answer E :
Choice (E) is not correct. Statements I and II must be true, but statement III could
could be greater than
be false. The value of
but
then
example, if
but it could be less. For
14
If
and
In the figure above,
is an integer, what is the least
possible value of
(A)
(B)
!
途
(C)
d
(D)
用
AND EXPLANATIONS
te
ANSWERS
re
(E)
业
Explanation for Correct Answer E :
Choice (E) is correct. The sum of the measures of the three angles of a triangle is
Substituting
it follows that
Since
so
商
is
gives
for
it follows that
Since
equivalent to
nR
eg
于
which implies that
least possible value of
用
禁
is the least integer greater than
Explanation for Incorrect Answer A :
were equal to
Choice (A) is not correct. If
U
严
The value of
so
then
so
so
then
so
then
would equal
However,
would then be
Therefore,
cannot equal
then
would equal
However,
would then be
cannot be
would equal
cannot equal
Therefore,
cannot be
Explanation for Incorrect Answer D :
were equal to
Choice (D) is not correct. If
The value of
would equal
However,
would then be
cannot be
which is
cannot equal
Therefore,
cannot be
Explanation for Incorrect Answer C :
were equal to
Choice (C) is not correct. If
The value of
is an
However,
would then be
Explanation for Incorrect Answer B :
were equal to
Choice (B) is not correct. If
The value of
Since
is also an integer. Therefore, the
it must be true that
integer and
this equation is
Therefore,
cannot equal
15
For all positive integers
and
let
be defined by
If
and
are positive integers, which of the following CANNOT be the value of
file://E:\新建文件夹\e4.htm
2006-11-12
页码,12/17
The Official SAT Online Course
(A)
(B)
(C)
(D)
(E)
ANSWERS
AND EXPLANATIONS
Explanation for Correct Answer B :
There are no positive integers
then
There are nonnegative integers that work (for
such that
and
), but the question states that
and
example,
integers.
If
cannot be equal to
Choice (B) is correct. The value of
then
Explanation for Incorrect Answer C :
and
The question
re
d
asks which number CANNOT be the value of
Choice (C) is not correct. If
are positive
!
途
Explanation for Incorrect Answer A :
Choice (A) is not correct. If
and
用
then
The
question asks which number CANNOT be the value of
te
业
Explanation for Incorrect Answer D :
Choice (D) is not correct. If
is
商
then
The
question asks which number CANNOT be the value of
nR
eg
于
Explanation for Incorrect Answer E :
Choice (E) is not correct. If
and
then
The
用
question asks which number CANNOT be the value of
16
禁
U
严
The functions
(A)
None
(B)
One
and
are defined above. For how many values of
is it true that
(C) Two
(D) Three
(E)
More than three
ANSWERS
AND EXPLANATIONS
Explanation for Correct Answer A :
Choice (A) is correct. If
that
so that
This implies
then
There are no real numbers
that satisfy
this equation.
file://E:\新建文件夹\e4.htm
2006-11-12
页码,13/17
The Official SAT Online Course
and
has no solutions is to graph
Another way to see that
-plane and observe that these two graphs do not intersect.
in the
for which
Therefore, there are no values
Explanation for Incorrect Answer B :
were
for which
Choice (B) is not correct. If there were one value of
However, this
true, then there would be a real solution of the equation
equation has no real solutions.
Explanation for Incorrect Answer C :
were
for which
Choice (C) is not correct. If there were two values of
However,
true, then there would be two real solutions of the equation
this equation has no real solutions.
Explanation for Incorrect Answer D :
for which
Choice (D) is not correct. If there were three values of
were true, then there would be at least one real solution of the equation
However, this equation has no real solutions. (In any case, the number
!
途
of solutions of a quadratic equation cannot be greater than two.)
d
Explanation for Incorrect Answer E :
Choice (E) is not correct. If there were more than three values of
for which
re
were true, then there would be at least one real solution of the
用
However, this equation has no real solutions. (In any case,
equation
the number of solutions of a quadratic equation cannot be greater than two.)
te
业
商
is
17
eg
于
nR
用
禁
U
严
The function
is graphed above. Which of the
defined for
following gives all values of
for which
is positive?
(A)
(B)
(C)
(D)
(E)
ANSWERS
and
and
AND EXPLANATIONS
Explanation for Correct Answer D :
Choice (D) is correct. The values of
file://E:\新建文件夹\e4.htm
for which
is positive are exactly those
2006-11-12
页码,14/17
The Official SAT Online Course
is positive.
for which
are all the values of
and
-axis. The graph
Therefore,
and
-axis for
lies above the
of
lies above the
-axis for which the graph of
values on the
Explanation for Incorrect Answer A :
is not positive for many values of
(Also,
the interval
which is not in
is positive for
Choice (A) is not correct. For example,
in the
)
interval
Explanation for Incorrect Answer B :
is positive for all values of
Choice (B) is not correct. It is true that
is also positive for all values of
but
that
Explanation for Incorrect Answer C :
Choice (C) is not correct. For example,
such that
yet
is in the interval
is positive for all values of
is not positive. (Also,
such
in the interval
)
!
途
Explanation for Incorrect Answer E :
is negative exactly for those values of
Choice (E) is not correct.
er
ed
is positive.
for which
of
in the
However, the question asks for all the values
and
intervals
用
18
(B)
is
t
商
于
用
nR
(C)
is the reflection of
eg
(A)
If line
is
-axis, what is an equation of line
in the
line
业
-plane, an equation of line
In the
(D)
禁
(E)
U
严
ANSWERS
AND EXPLANATIONS
Explanation for Correct Answer A :
Choice (A) is correct. If
When line
which is on line
to the point
equation of line
file://E:\新建文件夹\e4.htm
and
then it is true that
-axis, the point
Since
satisfy the relationship
point
Lines
is a point on line
is reflected in the
is reflected
the coordinates of
Therefore, an
is
are graphed in the figure below.
2006-11-12
页码,15/17
The Official SAT Online Course
Explanation for Incorrect Answer B :
for
is found by substituting
Choice (B) is not correct. An equation of line
This incorrect answer may be the result of mistakenly
in the equation
by
multiplying the entire right-hand side of
Explanation for Incorrect Answer C :
for
is found by substituting
Choice (C) is not correct. An equation of line
This incorrect answer may be the result of mistakenly
in the equation
instead of
for
substituting
in the equation
for
Explanation for Incorrect Answer D :
is an equation of a line perpendicular to line
Choice (D) is not correct.
-axis.
in the
not of the line that is the reflection of line
Explanation for Incorrect Answer E :
is an equation of a line perpendicular to line
Choice (E) is not correct.
-axis.
in the
not of the line that is the reflection of line
!
途
ed
19
(D)
st
er
于
用
nR
(E)
eg
i
(C)
will the system of equations above have no
商
(A)
(B)
用
业
For which of the following values for
solution?
禁
ANSWERS
AND EXPLANATIONS
U
严
Explanation for Correct Answer A :
then the system of equations becomes
Choice (A) is correct. If
then
This system has no solution: if
be equal to
must
not
Explanation for Incorrect Answer B :
then the system of equations becomes
Choice (B) is not correct. If
This system has the solution
and
Explanation for Incorrect Answer C :
then the system of equations becomes
Choice (C) is not correct. If
file://E:\新建文件夹\e4.htm
2006-11-12
页码,16/17
The Official SAT Online Course
and
This system has the solution
Explanation for Incorrect Answer D :
then the system of equations becomes
Choice (D) is not correct. If
and
This system has the solution
Explanation for Incorrect Answer E :
then the system of equations becomes
Choice (E) is not correct. If
!
途
and
re
d
This system has the solution
用
20
te
业
is
商
eg
于
nR
用
禁
customers purchased from a
The table above shows the number of items
hardware store over a -hour period. Which of the following can be determined from
the information in the table?
U
严
I.
II.
III.
The average (arithmetic mean) number of items purchased per customer
The median number of items purchased per customer
The mode of the number of items purchased per customer
(A)
None
(B)
I and II only
(C) I and III only
(D) II and III only
(E)
I, II, and III
ANSWERS
AND EXPLANATIONS
Explanation for Correct Answer D :
Choice (D) is correct. To determine the average number of items purchased per
customer, it is necessary to know the total number of items purchased. This total
of the
to find the average. However, for
can then be divided by
file://E:\新建文件夹\e4.htm
2006-11-12
页码,17/17
The Official SAT Online Course
customers, the exact number of items purchased is not given in the table. “Fewer
or as few as
customers bought as many as
” could mean these
than
items. Therefore, I cannot be determined.
The median number of items purchased per customer is the average of the number
customers when all customers are rank
and
of items purchased by the
and
ordered by the number of items purchased. The
items, so the median is
rank ordering each purchased
determined.
customers in this
Therefore, II can be
The mode is the number of items purchased by the greatest number of customers.
items each, and no larger
customers purchased
because
The mode is
Therefore, III can be
group purchased a number of items different from
determined.
Summarizing, only II and III can be determined from the information in the table.
Therefore, choice (D) is the correct answer.
Explanation for Incorrect Answer A :
Choice (A) is not correct. The median number of items purchased per customer and
the mode of the number of items purchased per customer can be determined from
the information in the table.
!
途
re
d
Explanation for Incorrect Answer B :
Choice (B) is not correct. The median can be determined from the information in the
table, but the mean cannot.
用
Explanation for Incorrect Answer C :
Choice (C) is not correct. The mode can be determined from the information in the
table, but the mean cannot.
te
业
Explanation for Incorrect Answer E :
Choice (E) is not correct. The median and the mode can be determined from the
information in the table, but the mean cannot.
is
商
eg
于
nR
用
禁
Copyright © 2006 The College Board. All rights reserved.
Back to Score Report
Privacy Policy
Terms of Use
Contact Us
U
严
file://E:\新建文件夹\e4.htm
2006-11-12
页码,1/24
The Official SAT Online Course
Help | Profile | My Organizer | My Bookmarks | Logout
Answers and Explanations
Back to Score Report
Test Sections
Section 1
View Answers and Explanations
Section 2
Online - Practice Test #5
Section 3
Section 4
1
Section 5
Section 6
The border between the two properties, never ------- by legal means, had long been
the subject of ------- between the antagonistic neighbors.
(A) determined . . concord
Section 8
(B)
Section 9
(C) verified . . consonance
Section 10
undermined . . hostility
(D) quantified . . diversion
established . . disputation
ANSWERS
!
途
AND EXPLANATIONS
er
ed
(E)
Explanation for Correct Answer E :
Choice (E) is correct. “To establish” means to agree to institute something
permanently. “Disputation” is the act of arguing. If one were to insert these
terms into the text, the sentence would read “The border between the two
properties, never established by legal means, had long been the subject of
disputation between the antagonistic neighbors.” It is logical to suggest that
antagonistic neighbors, or neighbors who express hostility and opposition, might
argue over a border that had never been "established."
用
is
t
业
eg
商
于
Explanation for Incorrect Answer A :
Choice (A) is incorrect. “To determine” means to settle or decide authoritatively.
“Concord” is a state of agreement. If one were to insert these terms into the
text, the sentence would read “The border between the two properties, never
determined by legal means, had long been the subject of concord between the
antagonistic neighbors.” Antagonistic neighbors, or neighbors who express
hostility and opposition, would probably not have a state of agreement concerning a
border that had not been "determined." On the contrary, an undetermined border
would most likely cause arguments between antagonistic neighbors.
nR
用
禁
U
严
Explanation for Incorrect Answer B :
Choice (B) is incorrect. “To undermine” means to weaken or subvert.
“Hostility” is an attitude of ill will and opposition. If one were to insert these
terms into the text, the sentence would read “The border between the two
properties, never undermined by legal means, had long been the subject of hostility
between the antagonistic neighbors.” Although antagonistic neighbors might be
hostile to one another, it is somewhat illogical to say that the property border had
never been “undermined,” or weakened, by legal means.
Explanation for Incorrect Answer C :
Choice (C) is incorrect. “To verify” means to establish formally. “Consonance”
is a state of harmony and agreement. If one were to insert these terms into the
text, the sentence would read “The border between the two properties, never
verified by legal means, had long been the subject of consonance between the
antagonistic neighbors.” Antagonistic neighbors, or neighbors who express
hostility and opposition, would most likely not have “consonance,” or a state of
harmony, concerning a border that had never been formally established. On the
contrary, a border that has not been “verified,” or established, would probably
cause arguments between antagonistic neighbors.
Explanation for Incorrect Answer D :
Choice (D) is incorrect. “To quantify” means to measure or determine in terms of
amount or magnitude. “Distraction” is something that draws attention away from
something else. If one were to insert these terms into the text, the sentence would
read “The border between the two properties, never quantified by legal means,
had long been the subject of distraction between the antagonistic neighbors.”
file://E:\新建文件夹\e5.htm
2006-11-12
页码,2/24
The Official SAT Online Course
Although a property border might be a “distraction” for antagonistic neighbors, it
is somewhat illogical to suggest that the border could be “quantified, ” or
determined in terms of amount or magnitude.
2
Even though Charlie was in apparently good health, the doctor prescribed for him
some ------- medication due to his familial history of high blood pressure.
(A) presumptive
(B)
predictive
(C) preliminary
(D) premeditated
(E)
preventative
ANSWERS
AND EXPLANATIONS
Explanation for Correct Answer E :
Choice (E) is correct. “Preventative” means preventing something. If one were to
insert this term into the text, the sentence would read “Even though Charlie was
in apparently good health, the doctor prescribed for him some preventative
medication due to his familial history of high blood pressure.” The phrase “Even
though” sets up a contrast between Charlie’s apparent good health and his
family’s history of high blood pressure. The missing term should describe a
medication that the doctor would prescribe for Charlie even in the absence of
symptoms. It would make sense for the doctor to prescribe “preventative”
medication to help Charlie avoid developing the high blood pressure that runs in his
family.
te
re
d
!
途
用
业
Explanation for Incorrect Answer A :
Choice (A) is incorrect. “Presumptive” means based on an assumption. If one
were to insert this term into the text, the sentence would read “Even though
Charlie was in apparently good health, the doctor prescribed for him some
presumptive medication due to his familial history of high blood pressure.” While
the doctor might have presumed that Charlie should be taking medication due to
his family’s history of high blood pressure, it does not make sense to say that the
medication itself was “presumptive,” or based on an assumption that the doctor
made about Charlie’s health.
eg
is
商
于
nR
用
Explanation for Incorrect Answer B :
Choice (B) is incorrect. “Predictive” means making a prediction. If one were to
insert this term into the text, the sentence would read “Even though Charlie was
in apparently good health, the doctor prescribed for him some predictive medication
due to his familial history of high blood pressure.” It does not make sense to
describe medication as “predictive.” Medication cannot predict a patient’s
health.
禁
U
严
Explanation for Incorrect Answer C :
Choice (C) is incorrect. “Preliminary” means coming before something else. If
one were to insert this term into the text, the sentence would read “Even though
Charlie was in apparently good health, the doctor prescribed for him some
preliminary medication due to his familial history of high blood pressure.” The
phrase “Even though” sets up a contrast between Charlie’s apparent good
health and his family’s history of high blood pressure. The missing term should
describe a medication that the doctor would prescribe for Charlie even in the
absence of symptoms. For medication to be “preliminary,” or coming before
something else, the doctor would have to know that Charlie would later need other
medications. Charlie may or may not develop high blood pressure, so the
medication would not necessarily be preliminary.
Explanation for Incorrect Answer D :
Choice (D) is incorrect. “Premeditated” means considered beforehand. If one
were to insert this term into the text, the sentence would read “Even though
Charlie was in apparently good health, the doctor prescribed for him some
premeditated medication due to his familial history of high blood pressure.” While
the doctor’s decision to prescribe medication for Charlie may have been
“premeditated,” or considered beforehand, it does not make sense to say that
the medication itself was premeditated.
file://E:\新建文件夹\e5.htm
2006-11-12
页码,3/24
The Official SAT Online Course
3
Though earlier anatomists had touched on the idea, Paul Broca was the first to
--------- fully the modern notion that specific behaviors are controlled by particular
areas of the human brain.
(A) articulate
(B)
derogate
(C) represent
(D) refute
(E)
iterate
ANSWERS
AND EXPLANATIONS
Explanation for Correct Answer A :
Choice (A) is correct. “To articulate” means to express a concept. If one were to
insert this term into the text, the sentence would read “Though earlier anatomists
had touched on the idea, Paul Broca was the first to articulate fully the modern
notion that specific behaviors are controlled by particular areas of the human
brain.” While earlier anatomists had touched on the idea that certain behaviors
are controlled by particular areas of the brain, Paul Broca was the first to fully
“articulate,” or express, the concept.
er
ed
!
途
Explanation for Incorrect Answer B :
Choice (B) is incorrect. “To derogate” means to belittle or to cause to seem
inferior. If one were to insert this term into the text, the sentence would read
“Though earlier anatomists had touched on the idea, Paul Broca was the first to
derogate fully the modern notion that specific behaviors are controlled by particular
areas of the human brain.” Nothing in the sentence suggests that either the
earlier anatomists or Paul Broca in any way derogated, or belittled, the idea that
certain behaviors are controlled by particular areas of the brain.
用
is
t
业
商
eg
Explanation for Incorrect Answer C :
Choice (C) is incorrect. “To represent” means to serve as a sign or symbol of
something. If one were to insert this term into the text, the sentence would read
“Though earlier anatomists had touched on the idea, Paul Broca was the first to
represent fully the modern notion that specific behaviors are controlled by
particular areas of the human brain.” The fact that earlier anatomists touched on
the notion that certain behaviors are controlled by particular areas of the brain is
not logically connected to the idea that Paul Broca represented, or was a symbol of,
that notion.
于
nR
用
禁
Explanation for Incorrect Answer D :
Choice (D) is incorrect. “To refute” means to prove wrong. If one were to insert
this term into the text, the sentence would read “Though earlier anatomists had
touched on the idea, Paul Broca was the first to refute fully the modern notion that
specific behaviors are controlled by particular areas of the human brain.” Nothing
in the sentence suggests that earlier anatomists had partially “refuted,” or
proved wrong, the idea that certain behaviors are controlled by particular areas of
the brain, so it would not make sense to say that Paul Broca fully refuted the idea.
U
严
Explanation for Incorrect Answer E :
Choice (E) is incorrect. “To iterate” means to say or do something repeatedly. If
one were to insert this term into the text, the sentence would read “Though earlier
anatomists had touched on the idea, Paul Broca was the first to iterate fully the
modern notion that specific behaviors are controlled by particular areas of the
human brain.” Although Paul Broca could have been the first to fully state the idea
that specific areas of the human brain control specific behaviors, nothing in the
sentence suggests that he stated this idea repeatedly.
4
Designed as a gathering place, the new student lounge was appropriately ------- with
tables, chairs, and even sofas where groups could assemble comfortably.
(A)
indicated
(B)
appointed
(C) denuded
file://E:\新建文件夹\e5.htm
2006-11-12
页码,4/24
The Official SAT Online Course
(D) conflated
(E)
venerated
ANSWERS
AND EXPLANATIONS
Explanation for Correct Answer B :
Choice (B) is correct. “To appoint” means to provide with furnishings or
equipment. If one were to insert this term into the text, the sentence would read
“Designed as a gathering place, the new student lounge was appropriately
appointed with tables, chairs, and even sofas where groups could assemble
comfortably.” A student lounge that is intended to be a gathering place would
most likely be “appointed,” or furnished, with tables, chairs, and sofas that
would allow groups to gather together in comfortable surroundings.
Explanation for Incorrect Answer A :
Choice (A) is incorrect. “To indicate” means to show or make known. If one were
to insert this term into the text, the sentence would read “Designed as a gathering
place, the new student lounge was appropriately indicated with tables, chairs, and
even sofas where groups could assemble comfortably.” It does not make sense to
say that the lounge was “indicated,” or shown, by its tables, chairs, and sofas.
!
途
ed
Explanation for Incorrect Answer C :
Choice (C) is incorrect. “To denude” means to make bare. If one were to insert
this term into the text, the sentence would read “Designed as a gathering place,
the new student lounge was appropriately denuded with tables, chairs, and even
sofas where groups could assemble comfortably.” The term “denuded” does not
make sense in this context. If the student lounge was “denuded,” or made bare,
it would not have any furniture.
er
用
业
is
t
Explanation for Incorrect Answer D :
Choice (D) is incorrect. “To conflate” means to join or combine. If one were to
insert this term into the text, the sentence would read “Designed as a gathering
place, the new student lounge was appropriately conflated with tables, chairs, and
even sofas where groups could assemble comfortably.” Although a gathering place
might have tables, chairs, and sofas, it does not make sense to say that the student
lounge was “conflated,” or joined, with these pieces of furniture.
eg
商
于
Explanation for Incorrect Answer E :
Choice (E) is incorrect. “To venerate” means to honor or regard something with
respect. If one were to insert this term into the text, the sentence would read
“Designed as a gathering place, the new student lounge was appropriately
venerated with tables, chairs, and even sofas where groups could assemble
comfortably.” Although a gathering place might have tables, chairs, and sofas, it is
not clear how these pieces of furniture would cause the student lounge to be
“venerated,” or regarded with respect.
nR
用
禁
U
严
5
Named in honor of the school’s founder, the Richard Brownstone Community Service
Fellowship is one of the highest awards ------- Brownstone School graduates.
(A)
conscripted to
(B)
redeemed for
(C) conferred on
(D) relegated to
(E)
deprived of
ANSWERS
AND EXPLANATIONS
Explanation for Correct Answer C :
Choice (C) is correct. “To confer” means to give to or to bestow upon. If one
were to insert this term into the text, the sentence would read “Named in honor of
the school’s founder, the Richard Brownstone Community Service Fellowship is
one of the highest awards conferred on Brownstone School graduates.” It makes
sense to say that an award would be “conferred on,” or given to, graduates of
Brownstone School.
file://E:\新建文件夹\e5.htm
2006-11-12
页码,5/24
The Official SAT Online Course
Explanation for Incorrect Answer A :
Choice (A) is incorrect. “To conscript” means to enroll into service. If one were to
insert this term into the text, the sentence would read “Named in honor of the
school’s founder, the Richard Brownstone Community Service Fellowship is one of
the highest awards conscripted to Brownstone School graduates.” The term
“conscripted” does not make sense in this context. It is illogical to suggest that
an award could be enrolled into service.
Explanation for Incorrect Answer B :
Choice (B) is incorrect. “To redeem” means to exchange for something of value.
If one were to insert this term into the text, the sentence would read “Named in
honor of the school’s founder, the Richard Brownstone Community Service
Fellowship is one of the highest awards redeemed for Brownstone School
graduates.” The term “redeemed” does not make sense in this context. It does
not make sense to say that an award can be “redeemed,” or exchanged, for
graduates of the Brownstone School.
Explanation for Incorrect Answer D :
Choice (D) is incorrect. “To relegate” means to assign to a place of insignificance.
If one were to insert this term into the text, the sentence would read “Named in
honor of the school’s founder, the Richard Brownstone Community Service
Fellowship is one of the highest awards relegated to Brownstone School
graduates.” It does not make sense to suggest that presenting graduates of
Brownstone School with an award is an action of assigning something to a place of
insignificance. If some of the graduates are presented with “one of the highest
awards,” it is illogical to describe either the award or the recipients as insignificant.
d
!
途
te
re
Explanation for Incorrect Answer E :
Choice (E) is incorrect. “To deprive” means to take away or withhold. If one were
to insert this term into the text, the sentence would read “Named in honor of the
school’s founder, the Richard Brownstone Community Service Fellowship is one of
the highest awards deprived of Brownstone School graduates.” It does not make
sense to suggest that the award is “deprived of,” or withheld from, graduates of
Brownstone School.
用
业
is
商
eg
于
The following passage on monarch butterflies was written in 2000 by a naturalist and
insect specialist.
nR
用
The migration and the winter
gathering of monarch butterflies are
among the most spectacular of all natural
Line phenomena, unique in the insect world.
5
Lincoln Brower wrote of his feeling on a
warm March morning as he watched tens
of thousands of these butterflies explode
from their resting places on the trees at
an overwintering site in Mexico: “Flying
10 against the azure sky and past the green
boughs of the oyamels, this myriad of
dancing embers reinforced my earlier
conclusion that this spectacle is a treasure
comparable to the finest works of art that
15 our world culture has produced over the
past 4000 years.” But even as I write this
paragraph, the winter gathering places of
the monarch are being destroyed by
illegal logging—indeed, all of the oyamel
20 forests in Mexico are threatened by legal
and illegal logging. If the logging
禁
U
严
file://E:\新建文件夹\e5.htm
2006-11-12
页码,6/24
The Official SAT Online Course
25
30
35
!
途
ed
40
continues at its present rate, all of the
overwintering sites in Mexico will be gone
by the first decades of the twenty-first
century. So desperate is the situation
that the Union for the Conservation of
Nature and Natural Resources has
recognized the monarch migration as an
endangered biological phenomenon and
has designated it the first priority in their
effort to conserve the butterflies of the
world.
All efforts to preserve the
overwintering sites in Mexico have failed.
In August of 1986, the Mexican
government issued a proclamation
designating these sites as ecological
preserves. Five of the 12 known sites
were to receive complete protection.
Logging and agricultural development
were to be prohibited in their core areas,
a total area of only 17 square miles, and
only limited logging was to be permitted in
buffer zones surrounding the cores, a total
of another 43 miles. The proclamation
was largely ignored. One of the 5
“protected” sites has been clear-cut, some
buffer zones have been more or less
completely destroyed, and trees are being
cut in all of the core areas. As Brower
told me, guards that were appointed to
protect the monarch colonies have not
prevented illegal logging but have barred
tourists, film crews, and scientists from
witnessing logging activities. It is
incomprehensible to me that a way cannot
be found to protect a mere 60 square
miles of land that are home to one of the
world’s most spectacular biological
phenomena.
If the monarchs are to survive, the
oyamel forests in which they spend the
winter must remain intact. Even minor
thinning of the core areas causes high
mortality among the butterflies, because
the canopy of the intact forest serves as a
protective blanket and umbrella for them.
Within a dense stand of trees, the
temperature does not drop as low as it
does elsewhere, enabling the monarchs to
survive freezing weather under the
blanket of trees. Thinning the trees puts
holes in the “umbrella” that protects the
用
te
r
45
业
is
商
于
eg
50
60
65
70
file://E:\新建文件夹\e5.htm
禁
严
U
55
nR
用
2006-11-12
页码,7/24
The Official SAT Online Course
monarchs, letting them get wet during
75 winter storms. A wet butterfly loses its
resistance to freezing and dies. Even a
dry butterfly loses precious calories as its
body heat radiates out to the cold night
sky through holes in the canopy.
6
The quotation in lines 9-16 serves primarily as a
(A)
detailed explanation of the cultural significance of a place
(B)
personal observation about artistic awareness
(C) dramatic portrayal of an impressive event
(D) scientific account of a rare phenomenon
(E)
conclusive argument for the artistic importance of spectacle
ANSWERS
AND EXPLANATIONS
Explanation for Correct Answer C :
Choice (C) is correct. The author begins the passage by stating that “the
migrations and winter gathering of monarch butterflies are among the most
spectacular of all natural phenomena.” To support this statement powerfully, the
author uses Lincoln Brower’s vivid and dramatic description of his personal
viewing of the monarchs. Brower’s portrayal of the monarchs as “this myriad of
dancing embers” “flying against the azure sky” gives the reader the sense that
the winter gathering of monarchs is an impressive event.
re
d
!
途
te
用
业
Explanation for Incorrect Answer A :
Choice (A) is incorrect. The quotation vividly describes the winter gathering of
monarchs in Mexico. Although Lincoln Brower does claim that the event is “a
treasure comparable to the finest works of art that our world culture has
produced,” he does not claim that the winter gathering place itself has any
cultural significance.
is
商
nR
eg
于
用
Explanation for Incorrect Answer B :
Choice (B) is incorrect. The quotation does recount a personal observation of
Lincoln Brower’s, but the observation is about how spectacular the winter
gathering of monarchs is, not about artistic awareness in general.
禁
Explanation for Incorrect Answer D :
Choice (D) is incorrect. The author of the passage does claim that the winter
gathering of monarchs is a rare phenomenon, “unique in the insect world.” But
the quotation is a dramatic portrayal using vivid imagery, not a scientific account of
the event.
U
严
Explanation for Incorrect Answer E :
Choice (E) is incorrect. Although the quotation by Lincoln Brower does state that
the winter gathering of monarchs is a spectacle “comparable to the finest works of
art our world culture has produced,” Brower does not develop a conclusive
argument for its artistic importance. He simply thinks it is an extremely beautiful
natural event.
7
The author views the “efforts” cited in line 33 as
(A)
understandably futile
(B)
necessarily limited
(C) scientifically misguided
(D) largely undesirable
(E)
unjustifiably ineffective
ANSWERS
file://E:\新建文件夹\e5.htm
AND EXPLANATIONS
2006-11-12
页码,8/24
The Official SAT Online Course
Explanation for Correct Answer E :
Choice (E) is correct. The “efforts” in line 33 refer to failed attempts “to
preserve the overwintering sites [of monarchs] in Mexico” from the destruction
brought on by unchecked “legal and illegal logging.” The author clearly believes
that these efforts have been ineffective, reporting that “one of the five
‘protected’ sites has been clear-cut, some buffer zones have been more or less
completely destroyed, and trees are being cut down in all of the core areas.” The
author openly states that such ineffectiveness is unjustifiable: “It is
incomprehensible to me that a way cannot be found to protect a mere 60 square
miles of land that are home to one of the world’s most spectacular biological
phenomena.”
Explanation for Incorrect Answer A :
Choice (A) is incorrect. The “efforts” in line 33 refer to failed attempts “to
preserve the overwintering sites [of monarchs] in Mexico” from the destruction
brought on by unchecked “legal and illegal logging.” While the author clearly
believes that these efforts have been futile, he or she would not agree that this
futility is understandable. The author cannot understand this futility: “It is
incomprehensible to me that a way cannot be found to protect a mere 60 square
miles of land that are home to one of the world’s most spectacular biological
phenomena.”
!
途
re
d
Explanation for Incorrect Answer B :
Choice (B) is incorrect. The “efforts” in line 33 refer to failed attempts “to
preserve the overwintering sites [of monarchs] in Mexico” from the destruction
brought on by unchecked “legal and illegal logging.” The author might agree that
the efforts have been limited, but not that this limitation is necessary. The author
actually believes that it should be rather easy to protect the land: “It is
incomprehensible to me that a way cannot be found to protect a mere 60 square
miles of land that are home to one of the world’s most spectacular biological
phenomena.”
is
te
用
业
Explanation for Incorrect Answer C :
Choice (C) is incorrect. The “efforts” in line 33 refer to failed attempts “to
preserve the overwintering sites [of monarchs] in Mexico” from the destruction
brought on by unchecked “legal and illegal logging.” The author does not
comment on the scientific foundation underlying the efforts to protect the sites.
eg
商
于
Explanation for Incorrect Answer D :
Choice (D) is incorrect. The “efforts” in line 33 refer to failed attempts “to
preserve the overwintering sites [of monarchs] in Mexico” from the destruction
brought on by unchecked “legal and illegal logging.” The author argues that the
efforts have been ineffective, not undesirable. On the contrary, the author clearly
desires stronger efforts to protect the sites: “It is incomprehensible to me that a
way cannot be found to protect a mere 60 square miles of land that are home to
one of the world’s most spectacular biological phenomena.”
nR
用
严
U
8
禁
The third paragraph is best described as
(A)
an account of a natural struggle for survival
(B)
a comparison between two types of environments
(C) a description of a disruption in an ecological system
(D) a demonstration of successful efforts to preserve an environment
(E)
a guideline for opposing the destruction of a crucial habitat
ANSWERS
AND EXPLANATIONS
Explanation for Correct Answer C :
Choice (C) is correct. The third paragraph describes the effects that destroying
trees in the oyamel forest has on monarch butterflies. When trees are cut down,
holes are created in the protective canopy that usually keeps the monarchs warm
and dry. When the monarchs become cold and wet they often freeze to death. Such
events are a disruption in the natural oyamel forest ecological system caused by
the unchecked logging.
file://E:\新建文件夹\e5.htm
2006-11-12
页码,9/24
The Official SAT Online Course
Explanation for Incorrect Answer A :
Choice (A) is incorrect. The description of the monarchs dying in the cold, wet night
is certainly a struggle for survival. The monarch’s struggle, however, is not
natural; it is caused by unchecked human loggers.
Explanation for Incorrect Answer B :
Choice (B) is incorrect. The third paragraph describes only one environment: the
oyamel forest that is being destroyed by loggers.
Explanation for Incorrect Answer D :
Choice (D) is correct. The third paragraph does not describe successful efforts to
preserve an environment. It describes the effects that the destruction of an
environment—the oyamel forest—has on the wintering monarchs.
Explanation for Incorrect Answer E :
Choice (E) is incorrect. The paragraph offers no guideline for opposing the
destruction of the oyamel forest. It simply describes the effects that the destruction
has on the wintering monarchs.
9
The tone of the passage could best be characterized as
(A)
indifferent
(B)
hostile
!
途
(C) concerned
complimentary
ANSWERS
te
re
(E)
d
(D) bewildered
用
AND EXPLANATIONS
业
Explanation for Correct Answer C :
Choice (C) is correct. The passage exhibits a concerned tone. The author begins by
describing the spectacular beauty of wintering monarchs. Then the author states
that “even as I write this paragraph, the winter gathering places of the monarch
are being destroyed by illegal logging.” The passage goes on to describe the
failing attempts being made to preserve the overwintering sites and the dying
monarchs’ struggle to survive in the damaged forests. Overall, the author
demonstrates a deep concern for the survival of the monarchs.
is
商
eg
于
nR
用
禁
Explanation for Incorrect Answer A :
Choice (A) is incorrect. The author is not at all indifferent to the plight of the
monarch butterfly. The author is very concerned with preserving the wintering sites
of the monarch, explicitly stating: “It is incomprehensible to me that a way cannot
be found to protect a mere 60 square miles of land that are home to one of the
world’s most spectacular biological phenomena.”
U
严
Explanation for Incorrect Answer B :
Choice (B) is incorrect. The passage does not exhibit a hostile tone. Mainly the
author is concerned about the plight of the monarch butterfly. The author is critical
of the failed efforts to preserve the overwintering sites, but the tone is troubled, not
hostile.
Explanation for Incorrect Answer D :
Choice (D) is incorrect. The tone of passage is concerned, not bewildered, or
confused. The author fully understands that the monarchs are in danger of losing
their overwintering habitat to logging.
Explanation for Incorrect Answer E :
Choice (E) is incorrect. The tone of the passage is concerned, not complimentary.
The author does not compliment the “efforts” to preserve the monarchs’
overwintering sites, but rather criticizes them for their failure.
In this excerpt from a British novel published in 1938, a woman describes staying with
her employer at a fashionable hotel in the resort city of Monte Carlo.
file://E:\新建文件夹\e5.htm
2006-11-12
页码,10/24
The Official SAT Online Course
I wonder what my life would be today, if Mrs. Van Hopper had not been a
snob.
Line
Funny to think that the course of
5
my existence hung like a thread upon that
quality of hers. Her curiosity was a
disease, almost a mania. At first I had
been shocked, wretchedly embarrassed
when I watched people laugh behind her
10 back, leave a room hurriedly upon her
entrance, or even vanish behind a Service
door on the corridor upstairs. For many
years now she had come to the hotel Cote
d’Azur, and, apart from bridge, her one
15 pastime, which was notorious by now in
Monte Carlo, was to claim visitors of
distinction as her friends had she but seen
them once at the other end of the postoffice. Somehow she would manage to
20 introduce herself, and before her victim
had scented danger she had proffered an
invitation to her suite. Her method of
attack was so downright and sudden that
there was seldom opportunity to escape.
25 At the Cote d’Azur she staked a claim
upon a certain sofa in the lounge, midway
between the reception hall and the
passage to the restaurant, and she would
have her coffee there after luncheon and
30 dinner, and all who came and went must
pass her by. Sometimes she would
employ me as a bait to draw her prey,
and, hating my errand, I would be sent
across the lounge with a verbal message,
35 the loan of a book or paper, the address
of some shop or other, the sudden
discovery of a mutual friend. It seemed
as though notables must be fed to her,
re
d
!
途
用
st
e
业
eg
i
商
于
nR
用
禁
U
严
1
and though titles were preferred by her,
40 any face once seen in a social paper
served as well. Names scattered in a
gossip column, authors, artists, actors and
their kind, even the mediocre ones, as
long as she had learnt of them in print.
45
I can see her as though it were but
yesterday, on that unforgettable
afternoon—never mind how many years
ago—when she sat on her favourite sofa in
the lounge, debating her method of
50 attack. I could tell by her abrupt manner,
and the way she tapped her lorgnette
file://E:\新建文件夹\e5.htm
2
2006-11-12
页码,11/24
The Official SAT Online Course
55
60
65
!
途
ed
70
against her teeth, that she was questing
possibilities. I knew, too, when she had
missed the sweet and rushed through
dessert, and she had wished to finish
luncheon before the new arrival and so
install herself where he must pass.
Suddenly she turned to me, her small
eyes alight.
“Go upstairs quickly and find that
letter from my nephew. You remember,
the one written on his honeymoon, with
the snapshot. Bring it down right away.”
I saw then that her plans were
formed, and the nephew was to be the
means of introduction. Not for the first
time I resented the part that I must play
in her schemes. Like a juggler’s assistant
I produced the props, then silent and
attentive I waited on my cue. This newcomer would not welcome intrusion, I felt
certain of that. In the little I had learnt of
him at luncheon, a smattering of hearsay
garnered by her ten months ago from the
daily papers and stored in her memory for
future use, I could imagine, in spite of my
youth and inexperience of the world, that
he would resent this sudden bursting in
upon his solitude. Why he should have
chosen to come to the Cote d’Azur at
Monte Carlo was not our concern, his
problems were his own, and anyone but
Mrs. Van Hopper would have understood.
Tact was a quality unknown to her,
discretion too, and because gossip was
the breath of life to her this stranger must
be served for her dissection.
用
is
te
r
75
业
商
于
nR
eg
80
用
禁
严
U
85
1 “Titles” here refers to members of the European
nobility.
2 Eyeglasses on the end of a short handle.
10
The passage is narrated from the point of view of
(A)
an employee of the Cote d’Azur hotel
(B)
an observer who is uninvolved in the action
(C) Mrs. Van Hopper
(D) a participant who is remembering the scene at a later time
(E)
a tourist who has just met Mrs. Van Hopper
ANSWERS
file://E:\新建文件夹\e5.htm
AND EXPLANATIONS
2006-11-12
页码,12/24
The Official SAT Online Course
Explanation for Correct Answer D :
Choice (D) is correct. We know that the narrator is a “participant” because she
narrates the action in the first person—“Sometimes she [Mrs. Van Hopper] would
employ me as a bait to draw her prey,” for example. Further, there are clear signs
in the passage that the narrator is recalling the action from the distance of some
years: “I can see her as though it were but yesterday”; “never mind how many
years ago.” Together, these clues tell the reader that the person narrating is
someone who stayed with Mrs. Van Hopper years before at the Cote d’Azur hotel
and is remembering the action.
Explanation for Incorrect Answer A :
Choice (A) is incorrect. It is clear from context that the setting is the Cote d’Azur
hotel, but it is also clear that the narrator works for Mrs. Van Hopper, not for the
hotel: Mrs. Van Hopper might very well send a hotel employee on a personal errand
as she does in paragraph 4, but no hotel employee could know as much about or
spend as much time with Mrs. Van Hopper as does this narrator.
Explanation for Incorrect Answer B :
Choice (B) is incorrect. Certainly the narrator is an “observer” of Mrs. Van
Hopper’s various strategies for “trapping” people, but we know at least as early
as line 31 (“Sometimes she would employ me…”) that the narrator is also
involved in the action.
!
途
ed
Explanation for Incorrect Answer C :
Choice (C) is incorrect. It is unlikely—though not impossible—that Mrs. Van Hopper
would be speaking of herself in the third person. Further, when Mrs. Van Hopper
engages directly with the narrator in paragraph 4, we can be fairly certain that a
separate participant is involved.
is
te
r
用
Explanation for Incorrect Answer E :
Choice (E) is incorrect. The last paragraph of the passage does focus on someone
who, willingly or not, is about to meet Mrs. Van Hopper, but the passage clearly is
not narrated from this person’s point of view.
业
商
eg
11
于
The “disease” mentioned in line 7 is best described as
用
total embarrassment at another person’s behavior
nR
(A)
(B)
a refusal to speak to anyone who is not wealthy
禁
(C) an intense need to avoid public notice
(D) a violent tendency to assault strangers
U
严
(E)
a relentless drive to meet well-known people
ANSWERS
AND EXPLANATIONS
Explanation for Correct Answer E :
Choice (E) is correct. In the second paragraph of the passage, the narrator says
that Mrs. Van Hopper’s “curiosity was a disease.” The narrator then sets about
describing this “curiosity,” which turns out to be an abnormal interest in making
the acquaintance of well-known people. The paragraph describes various of Mrs.
Van Hopper’s techniques for meeting celebrities she does not know and claiming
them as her “friends”—behavior that clearly embarrasses the narrator. The
narrator says that this “disease,” or sickness, verges on mania, or an
unreasonable enthusiasm—a “relentless drive,” in other words, to meet people
who have been written about in gossip columns.
Explanation for Incorrect Answer A :
Choice (A) is incorrect. While the narrator acknowledges being “wretchedly
embarrassed” at Mrs. Van Hopper’s importunate behavior with strangers, the
“disease” referred to in line 7 is clearly attributed by the narrator to Mrs. Van
Hopper, who shows no “embarrassment” at all in preying on celebrities.
Explanation for Incorrect Answer B :
Choice (B) is incorrect. In the first paragraph, the narrator describes Mrs. Van
file://E:\新建文件夹\e5.htm
2006-11-12
页码,13/24
The Official SAT Online Course
Hopper as a "snob," and “a refusal to speak to anyone who is not wealthy” could
be considered a specific kind of snobbery. But it is clear from context that the
“disease” is not Mrs. Van Hopper’s refusal to speak, but rather her insistence
on speaking to the famous. While it can be inferred that most of those who frequent
this expensive hotel are wealthy, it is celebrity rather than money that Mrs. Van
Hopper is drawn to; this is her “disease.”
Explanation for Incorrect Answer C :
Choice (C) is incorrect. Far from “avoiding public notice,” Mrs. Van Hopper seats
herself daily in an area of the hotel where she is almost certain to be noticed, much
to the embarrassment of the narrator.
Explanation for Incorrect Answer D :
Choice (D) is incorrect. While Mrs. Van Hopper certainly does “assault” people in
the figurative sense of approaching them without their permission, there is no
indication that she is “violent” towards them, or even hostile. Rather, Mrs. Van
Hopper’s technique would more likely be directed towards ingratiating herself with
the celebrities she stalks than towards harming them.
12
In context, “employ” (line 32) most nearly means
(A)
service
(B)
use
!
途
(C) attract
hire
ANSWERS
te
re
(E)
d
(D) devote
用
AND EXPLANATIONS
业
Explanation for Correct Answer B :
Choice (B) is correct. In this sentence, the narrator is describing being sent by Mrs.
Van Hopper on useless errands whose sole purpose is to attract the attention of
someone Mrs. Van Hopper wants to befriend. The narrator feels, under these
circumstances, that she is being employed as “bait”—in other words, “used.”
eg
is
商
于
nR
用
Explanation for Incorrect Answer A :
Choice (A) is incorrect. In this sentence, the narrator is describing being sent by
Mrs. Van Hopper on useless errands whose sole purpose is to attract the attention
of someone Mrs. Van Hopper wants to befriend. In this context, it makes little
sense to say that the narrator is being “serviced, ” or provided with a service,
when in fact she is providing a humiliating “service” for Mrs. Van Hopper.
禁
U
严
Explanation for Incorrect Answer C :
Choice (C) is incorrect. In this sentence, the narrator is describing being sent by
Mrs. Van Hopper on useless errands whose sole purpose is to attract the attention
of someone Mrs. Van Hopper wants to befriend. In this context, it makes little
sense to say that the narrator is being “attracted,” or having her attention
solicited, when in fact she is being sent by Mrs. Van Hopper as "bait" to attract the
latter's "prey."
Explanation for Incorrect Answer D :
Choice (D) is incorrect. In this sentence, the narrator is describing being sent by
Mrs. Van Hopper on useless errands whose sole purpose is to attract the attention
of someone Mrs. Van Hopper wants to befriend. In this context, it does not make
sense to say that the narrator is being “devoted,” or dedicated, as “bait” to
attract Mrs. Van Hopper’s “prey.”
Explanation for Incorrect Answer E :
Choice (E) is incorrect. In this sentence, the narrator is describing being sent by
Mrs. Van Hopper on useless errands whose sole purpose is to attract the attention
of someone Mrs. Van Hopper wants to befriend. Although the narrator is clearly
“hired,” or employed, by Mrs. Van Hopper, in this sentence she is speaking of
being sent on useless errands in order to attract the attention of someone Mrs. Van
Hopper wants to befriend; she is being “used.”
file://E:\新建文件夹\e5.htm
2006-11-12
页码,14/24
The Official SAT Online Course
13
It is clear from context that by “debating” (line 49), the narrator means
(A) asking advice about
(B)
considering strategies for
(C) talking aloud to herself about
(D) taking notes on
(E)
arguing about
ANSWERS
AND EXPLANATIONS
Explanation for Correct Answer B :
Choice (B) is correct. In this part of the passage, the narrator is speaking of one
“unforgettable afternoon” when Mrs. Van Hopper is indulging in her usual
predatory behavior towards celebrities; on this occasion, she has one special
“victim” in mind. She has installed herself in her customary seat, and is
“debating,” or deliberating, about which “method of attack” (or way of
entrapping the stranger) to use. It is clear from context that she is reviewing
“methods” in her head and not aloud—the narrator can “tell” by Mrs. Van
Hopper’s actions what the latter is thinking about.
!
途
ed
Explanation for Incorrect Answer A :
Choice (A) is incorrect. In this part of the passage, it is clear from context that Mrs.
Van Hopper is reviewing “methods of attack”—that is, ways of entrapping a
certain stranger—in her head and not aloud. She is clearly thinking silently and not
“asking advice about” ways to proceed. The narrator can “tell” by Mrs. Van
Hopper’s actions what the latter is thinking about.
te
r
用
Explanation for Incorrect Answer C :
Choice (C) is incorrect. In this part of the passage, it is clear from context that Mrs.
Van Hopper is reviewing “methods of attack”—that is, ways of entrapping a
certain stranger—in her head and not aloud. She is clearly thinking silently and not
“talking aloud to herself about” ways to proceed. The narrator can “tell” by
Mrs. Van Hopper’s actions what the latter is thinking about.
业
nR
eg
is
商
于
Explanation for Incorrect Answer D :
Choice (D) is incorrect. In this part of the passage, it is clear from context that Mrs.
Van Hopper is reviewing “methods of attack”—that is, ways of entrapping a
certain stranger—in her head and not aloud. She is clearly thinking silently and not
“taking notes on” ways to proceed. The narrator can “tell” by Mrs. Van
Hopper’s actions what the latter is thinking about.
用
禁
Explanation for Incorrect Answer E :
Choice (E) is incorrect. In this part of the passage, it is clear from context that Mrs.
Van Hopper is reviewing “methods of attack”—that is, ways of entrapping a
certain stranger—in her head and not aloud. She is clearly thinking silently and not
“arguing” with someone else about ways to proceed. The narrator can “tell”
by Mrs. Van Hopper’s actions what the latter is thinking about.
U
严
14
It can be inferred that Mrs. Van Hopper sends her companion to retrieve something
from upstairs primarily with the goal of
(A)
communicating with a relative
(B)
having some time alone
(C) keeping information from the companion
(D) arranging a meeting for the companion
(E)
establishing a connection with a stranger
ANSWERS
AND EXPLANATIONS
Explanation for Correct Answer E :
Choice (E) is correct. We know from context that Mrs. Van Hopper is obsessed with
meeting celebrities, and that she has used the narrator as “bait” to draw the
intended “prey.” In the paragraph directly preceding the quotation, Mrs. Van
Hopper is “debating her method of attack.” When, after this, she speaks, it is
file://E:\新建文件夹\e5.htm
2006-11-12
页码,15/24
The Official SAT Online Course
clearly because she has hit upon an idea for meeting the stranger: she sends the
narrator to get something. In the next paragraph, the narrator says, “I saw then
that…the nephew was to be the means of introduction.” Clearly, the errand is
intended to “establish a connection with a stranger.”
Explanation for Incorrect Answer A :
Choice (A) is incorrect. While it is true that Mrs. Van Hopper sends the narrator to
get a “communication from a relative”—that is, the postcard from the nephew—
the goal of the errand is clearly not “communicating with a relative,” but getting
acquainted with another hotel guest, a “new arrival.”
Explanation for Incorrect Answer B :
Choice (B) is incorrect. While it is true that sending the narrator to get something
from upstairs will leave Mrs. Van Hopper momentarily alone, being alone is clearly
not the latter’s goal; rather, she is sending the narrator to retrieve an object that
will help Mrs. Van Hopper get acquainted with another hotel guest, a “new
arrival.”
Explanation for Incorrect Answer C :
Choice (C) is incorrect. While one might send a companion on an errand in order to
keep him or her from finding something out, that is clearly not the case here. Mrs.
Van Hopper is not interested in what the narrator knows or does not know; rather,
she is sending the narrator to retrieve an object that will help Mrs. Van Hopper get
acquainted with another hotel guest, a “new arrival.”
ed
!
途
Explanation for Incorrect Answer D :
Choice (D) is incorrect. While the long-term goal of the errand may be to “arrange
a meeting,” the meeting is clearly meant to be between Mrs. Van Hopper and
“the new arrival,” not between the companion and someone else.
er
用
is
t
业
In lines 64-70 (“I saw...cue”), the narrator imagines herself as a
(A)
playwright
(B)
actor
于
(C) stagehand
(D) criminal
用
nR
(E)
商
eg
15
magician
禁
ANSWERS
AND EXPLANATIONS
U
严
Explanation for Correct Answer C :
Choice (C) is correct. In the sentences cited, the narrator refers to the “part” she
must play in Mrs. Van Hopper’s “schemes”: “Like a juggler’s assistant I
produced the props, then silent and attentive I waited on my cue.” A stage
“juggler’s assistant,” who must assist the performer by providing props (or
articles to be used on stage) on cue (or when signaled) is a kind of “stagehand.”
Explanation for Incorrect Answer A :
Choice (A) is incorrect. In the sentences cited, the narrator refers to the “part”
she must play in Mrs. Van Hopper’s “schemes”: “Like a juggler’s assistant I
produced the props, then silent and attentive I waited on my cue.” While the
imagery does have to do with the theatre, the narrator is imagining herself not as a
writer, but, rather, as an assistant, or “stagehand.”
Explanation for Incorrect Answer B :
Choice (B) is incorrect. In the sentences cited, the narrator refers to the “part”
she must play in Mrs. Van Hopper’s “schemes”: “Like a juggler’s assistant I
produced the props, then silent and attentive I waited on my cue.” While the
imagery does have to do with the theatre, and while actors do “wait on cues,”
the narrator is imagining herself as someone who assists a performer, “producing
props and waiting on my cue”—in other words, a “stagehand.”
Explanation for Incorrect Answer D :
Choice (D) is incorrect. In the sentences cited, the narrator refers to the “part”
file://E:\新建文件夹\e5.htm
2006-11-12
页码,16/24
The Official SAT Online Course
she must play in Mrs. Van Hopper’s “schemes”: “Like a juggler’s assistant I
produced the props, then silent and attentive I waited on my cue.” There is no
reference here to criminality; rather, the narrator is imagining herself as someone
who assists a performer—in other words, a “stagehand.”
Explanation for Incorrect Answer E :
Choice (E) is incorrect. In the sentences cited, the narrator refers to the “part”
she must play in Mrs. Van Hopper’s “schemes”: “Like a juggler’s assistant I
produced the props, then silent and attentive I waited on my cue.” While the
imagery does have to do with stage juggling, there is no real reference to magic: a
juggler is not really a magician. Further, the narrator is imagining herself as
someone who assists a performer—in other words, a “stagehand.”
16
On the whole, the passage is developed in which of the following ways?
(A)
a single extended episode is narrated
(B)
a dialogue is recounted
(C) a physical description is followed by a summary
(D) a general description is followed by a specific example
(E)
a character description is followed by a monologue
!
途
AND EXPLANATIONS
ed
ANSWERS
Explanation for Correct Answer D :
Choice (D) is correct. The passage begins with a general description of Mrs. Van
Hopper’s customary behavior—deliberately and methodically stalking celebrity
visitors to the Cote d’Azur hotel. The reader is both led to believe and told directly
that this pattern has recurred for years. About halfway through the passage,
however, the narrator begins to focus on a specific instance of this behavior: “on
that unforgettable afternoon…Suddenly she turned to me, her small eyes alight.”
This narrative takes up the rest of the passage. So the method of development of
the passage is that of “a general description followed by a specific example.”
er
用
st
业
eg
i
商
于
Explanation for Incorrect Answer A :
Choice (A) is incorrect. While a single episode is narrated within the passage—the
last half or so of the passage takes place on one “unforgettable afternoon”—the
passage as a whole is not composed of a single extended narration. Rather, the first
half of the passage is taken up with a general description of Mrs. Van Hopper’s
behavior.
nR
用
禁
严
U
Explanation for Incorrect Answer B :
Choice (B) is incorrect. While there is what might be termed “dialogue” in the
passage—the direct quotation from Mrs. Van Hopper in the fourth paragraph—this
speech is only a very small part of the passage as a whole and does not explain the
passage’s development.
Explanation for Incorrect Answer C :
Choice (C) is incorrect. The “description” that makes up the first part of the
passage is about a certain pattern of behavior and cannot properly be called a
“physical” description, as of an object or a place. Further, the latter part of the
passage, since it narrates a single event, cannot properly be called a “summary.”
Explanation for Incorrect Answer E :
Choice (E) is incorrect. While the first part of the passage, in which Mrs. Van
Hopper’s customary behavior is described, might be called a kind of “character
description,” there is only one speech in the passage—Mrs. Van Hopper’s
instructions in the fourth paragraph—so the passage cannot be said to end with a
“monologue,” or long speech.
17
In context, “garnered” (line 74) is closest in meaning to
(A)
gathered
(B)
earned
assumed
file://E:\新建文件夹\e5.htm
2006-11-12
页码,17/24
The Official SAT Online Course
(C)
(D) inferred
(E)
harvested
ANSWERS
AND EXPLANATIONS
Explanation for Correct Answer A :
Choice (A) is correct. In this sentence, the narrator of the passage is speaking of
Mrs. Van Hopper’s having “garnered” information about the "newcomer” from
“daily papers.” In context, it makes sense to say that Mrs. Van Hopper
“gathered,” or acquired, this information from her newspaper reading.
Explanation for Incorrect Answer B :
Choice (B) is incorrect. In this sentence, the narrator of the passage is speaking of
Mrs. Van Hopper’s having “garnered” information about the "newcomer” from
“daily papers.” In context, it makes no sense to say that Mrs. Van Hopper
“earned,” or merited, this information from her newspaper reading.
Explanation for Incorrect Answer C :
Choice (C) is incorrect. In this sentence, the narrator of the passage is speaking of
Mrs. Van Hopper’s having “garnered” information about the "newcomer” from
“daily papers.” In context, it makes little sense to say that Mrs. Van Hopper
“assumed,” or took upon herself, this information from her newspaper reading.
ed
!
途
Explanation for Incorrect Answer D :
Choice (D) is incorrect. In this sentence, the narrator of the passage is speaking of
Mrs. Van Hopper’s having “garnered” information about the "newcomer” from
“daily papers.” In context, it makes little sense to say that Mrs. Van Hopper
“inferred,” or concluded, this information from her newspaper reading.
te
r
用
业
Explanation for Incorrect Answer E :
Choice (E) is incorrect. In this sentence, the narrator of the passage is speaking of
Mrs. Van Hopper’s having “garnered” information about the "newcomer” from
“daily papers.” In context, it makes little sense to say that Mrs. Van Hopper
“harvested,” or gathered a crop from, this information in her newspaper reading.
is
商
eg
于
nR
用
New York in the wake of World War
II was a city on the verge of momentous
changes—economic, social, and political.
Line For almost a century it had been a
5
preeminent manufacturing and port city,
absorbing the unskilled millions who
flocked there from Europe, and had
yielded great fortunes for the astute and
daring. The Depression of the 1930’s had
10 exacted a terrible toll, and leaders
conferred anxiously on how to avoid a
repetition of those doleful days as the war
economy wound down.
Even before the war, experts had
15 been cautioning New York that it was
losing industry and business to other
locales. A 1939 study showed the City’s
percentage of wage earners down in fiftyeight industries.
20
At the close of World War II, the
great port of New York remained the
禁
U
严
file://E:\新建文件夹\e5.htm
2006-11-12
页码,18/24
The Official SAT Online Course
largest general cargo port in the country
and the second largest in the world,
deferring only to Rotterdam. Week in and
25 week out forty thousand longshoremen
worked the ninety-six piers encircling
Manhattan and the eighty lining the
Brooklyn waterfront, manually loading and
*
35
!
途
re
40
d
30
unloading sixteen million tons a year .
Hundreds of tugs and barges plied the
harbor and nearby rivers, guiding the big
ships to their berths and ferrying cargo.
Looking ahead, the new Port Authority of
New York cleared forty old piers in
Brooklyn and replaced them with fourteen
spacious, modern piers, the best in the
world.
In the mid-1950’s Malcolm McLean
of Winston-Salem invented a method of
shipping that was to revolutionize cargo
ports and make the new piers obsolete.
His brainchild was the truck-sized sealed
containers that slashed loading and
unloading time to almost nothing. A small
crew of men could use cranes to load the
gargantuan containers, filling a ship
twenty times faster than the old gangs
grappling with crates, boxes, and bales.
The impregnable containers guaranteed
against waterfront theft or breakage,
eliminating altogether the dockside
pilferage that previously had consumed up
to 15 percent of some cargoes. Ships that
had traditionally spent a week in port
could now finish their loading in a day.
The new containers required huge
storing areas, far larger than were
available in either Manhattan or Brooklyn.
When the City of New York proposed
modernizing its East River piers to handle
the containers, the Port Authority said it
would have to clear all the land from the
river west to Third Avenue to do so. Each
berth for a ship carrying containers of
cargo needed fifty acres of surrounding
land, compared to an old-fashioned berth
of 195,000 square feet. The Port
Authority erected container ports at Port
Newark and Port Elizabeth, New Jersey,
with their vast stretches of empty land.
The old piers of Manhattan and Brooklyn
languished—rotting, deserted white
业
te
45
用
is
商
eg
用
60
65
70
file://E:\新建文件夹\e5.htm
禁
严
U
55
于
nR
50
2006-11-12
页码,19/24
The Official SAT Online Course
elephants. The ranks of the
longshoremen, once forty thousand
75 strong, dwindled to nine thousand. In the
new technical, mechanized world of
container shipping, man-hours fell from
40 million man-hours right after the war
to 13.5 million in 1983.
* The island of Manhattan is bounded by the Hudson
River and the state of New Jersey to the west and by the
East River and the New York City borough of Brooklyn on
the east.
18
In line 5, “preeminent” is closest in meaning to
(A)
knowledgeable
(B)
outstanding
(C) growing
!
途
(D) abnormal
ANSWERS
d
notorious
re
(E)
用
AND EXPLANATIONS
is
te
Explanation for Correct Answer B :
Choice (B) is correct. The first sentence of the paragraph says that post-World War
II New York was poised for important changes; the second sentence goes on to
discuss the city’s longtime importance as a shipping and manufacturing center.
The comparison between the past and present is implicit: New York “had been” a
“preeminent,” or outstanding, hub for industries that were now declining in
importance.
业
商
eg
于
nR
用
Explanation for Incorrect Answer A :
Choice (A) is incorrect. The sentence is clearly saying that New York “had been”
a “preeminent,” or outstanding, hub for industries that were declining in
importance. It makes little sense to say that the city had been “knowledgeable,”
or knowing.
禁
U
严
Explanation for Incorrect Answer C :
Choice (C) is incorrect. The first sentence of the paragraph says that post-World
War II New York was poised for important changes; the second sentence goes on to
discuss the city’s longtime importance as a shipping and manufacturing center.
While it makes a certain amount of sense to call New York a “growing” industrial
center, or one increasing in size or importance, the comparison between past and
present is implied by the context: New York “had been” a “preeminent,” or
outstanding, hub for industries that were now declining in importance.
Explanation for Incorrect Answer D :
Choice (D) is incorrect. The sentence is clearly saying that New York “had been”
a “preeminent,” or outstanding, hub for industries that were declining in
importance. It makes little sense in context to say that the city had been
“abnormal,” or deviating from normal, as an industrial center.
Explanation for Incorrect Answer E :
Choice (E) is incorrect. The first sentence of the paragraph says that post-World
War II New York was poised for important changes; the second sentence goes on to
discuss the city’s longtime importance as a shipping and manufacturing center.
“Notorious” means “widely but unfavorably known”; so while it might make
sense to call attention to New York’s fame as an industrial center, there is no
reason to assume that its fame was accompanied by something negative.
file://E:\新建文件夹\e5.htm
2006-11-12
页码,20/24
The Official SAT Online Course
19
In context, the “unskilled millions” (line 6) apparently refers to people who
(A) made great fortunes
(B)
were unprepared for the Depression of the 1930’s
(C) took jobs in shipping and manufacturing
(D) were uneasy about the U.S. economy
(E)
left New York to find work
ANSWERS
AND EXPLANATIONS
Explanation for Correct Answer C :
Choice (C) is correct. In these lines, the author is speaking of the century-long
importance of New York as a “manufacturing and port city,” a city that had both
absorbed “the unskilled millions who flocked there from Europe” and “yielded
great fortunes for the astute and daring.” It is clear from context that the
overriding topic is the New York City economy, which had been thriving for a
century before World War II but which was expected to falter somewhat in the
post-War years. In this reading, the “astute and daring” are those who made
great sums of money by speculation, and the “unskilled millions” the European
emigrants who had been employed—or absorbed—in manufacturing and seaport
trades during the city’s period of greatest importance.
ed
!
途
Explanation for Incorrect Answer A :
Choice (A) is incorrect. In these lines, the author is speaking of the century-long
importance of New York as a “manufacturing and port city,” a city that had both
absorbed “the unskilled millions who flocked there from Europe” and “yielded
great fortunes for the astute and daring.” It is clear from context that the latter—
those who made great fortunes—are separate from the European emigrants, who
clearly were employed—or absorbed by—the manufacturing and seaport trades.
eg
is
te
r
用
业
Explanation for Incorrect Answer B :
Choice (B) is incorrect. While this paragraph does mention the Depression, it does
so in the context of the Depression’s having “exacted a terrible toll”—or had a
disastrous effect—on New York City’s economy. It is clear from context that the
“unskilled millions” had found rather than lost jobs in the several decades before
the Depression of the 1930’s.
商
于
用
nR
Explanation for Incorrect Answer D :
Choice (D) is incorrect. While this paragraph does imply that various people were
uneasy about the New York economy in the wake of World War II, it is clearly
business “leaders,” not the “unskilled millions” of workers who had found
employment in the preceding century, who are experiencing this anxiety.
禁
U
严
Explanation for Incorrect Answer E :
Choice (E) is incorrect. While there is a suggestion in this paragraph that New York
City was poised to lose jobs in the post-World War II period, there is no discussion
of people who left the area in order to find work.
20
The third paragraph is best described as a description of
(A)
a process that would soon be obsolete
(B)
a blueprint for changing an area
(C) a plan that drew much criticism
(D) a decline in the importance of an industry
(E)
an event that foreshadowed future happenings
ANSWERS
AND EXPLANATIONS
Explanation for Correct Answer A :
Choice (A) is correct. The third paragraph of the passage clearly discusses the state
of New York City’s shipping industry “at the close of World War II.” But as the
rest of the passage makes clear, this was an industry whose circumstances were
about to change drastically—and for the worse—in the decade of the 1950’s;
shipping as it was carried on the 1940’s was about to become “obsolete.”
file://E:\新建文件夹\e5.htm
2006-11-12
页码,21/24
The Official SAT Online Course
Explanation for Incorrect Answer B :
Choice (B) is incorrect. The only real discussion of “changing an area” occurs in
the fifth paragraph of the passage, not in the third; and this change—the decline of
the old New York piers—comes about gradually and unintentionally, not according
to a “blueprint.”
Explanation for Incorrect Answer C :
Choice (C) is incorrect. While it might be inferable that the shipping revolution
described in the fourth paragraph would “draw criticism,” there is no indication
that the description given in the third paragraph did so; further, it is more a
description of a process than it is a “plan.”
Explanation for Incorrect Answer D :
Choice (D) is incorrect. While the third paragraph of the passage does indeed
discuss the shipping industry—an industry, as the rest of the passage tells us, that
would soon decline—this paragraph discusses New York shipping at its height, not
at the time of its “decline in importance.”
Explanation for Incorrect Answer E :
Choice (E) is incorrect. While the last sentence in the third paragraph might be
construed as a “foreshadowing” or indication of future events—the clearing of the
old piers to make room for the new piers, which would themselves become
obsolete—the great majority of this paragraph is devoted to discussing the shipping
industry of New York at its height, not in its decline.
用
The author considers Malcolm McLean to be
(A)
a dreamer
(B)
an opportunist
(C) an eccentric
业
is
商
(D) an obstructionist
an innovator
eg
(E)
te
21
re
d
!
途
ANSWERS
于
AND EXPLANATIONS
nR
用
Explanation for Correct Answer E :
Choice (E) is correct. An “innovator” is someone who introduces important
changes. Clearly the author of the passage sees McLean, who invented a
revolutionary method of shipping that made the old methods (and piers)
“obsolete,” as an innovator.
禁
U
严
Explanation for Incorrect Answer A :
Choice (A) is incorrect. A “dreamer” is someone who lives in a world of unreality.
A dreamer may have great ideas, but they are usually of an impractical nature.
Clearly the author of the passage sees McLean, whose invention was both
revolutionary and practical, as an innovator rather than as a dreamer.
Explanation for Incorrect Answer B :
Choice (B) is incorrect. An “opportunist” is someone who takes advantage of
opportunities, but who does so with little thought for principle or ethics. While the
author of the passage clearly considers McLean someone who seized opportunities,
there is no indication in the passage that McLean was unprincipled or unethical.
Explanation for Incorrect Answer C :
Choice (C) is incorrect. An “eccentric” is someone who behaves in an
unconventional manner. While there is a conception in popular culture of inventors
being eccentric, there is no indication in this passage that the author considers
McLean or his behavior unconventional.
Explanation for Incorrect Answer D :
Choice (D) is incorrect. An “obstructionist” is someone who deliberately
interferes with progress. Far from considering McLean as an obstacle to progress,
the author clearly sees McLean as someone who aided or sped progress—an
innovator.
file://E:\新建文件夹\e5.htm
2006-11-12
页码,22/24
The Official SAT Online Course
22
The passage mentions each of the following as an advantage of container shipping
EXCEPT:
(A)
large storing areas
(B)
reduced loading time
(C) theft deterrence
(D) personnel reduction
(E)
breakage prevention
ANSWERS
AND EXPLANATIONS
Explanation for Correct Answer A :
Choice (A) is correct. While the passage does indeed discuss “large storing
areas” in the last paragraph, this characteristic of container shipping is presented
more as a disadvantage than an advantage. Container shipping needed “huge
storing areas, far larger than were available in either Manhattan or Brooklyn.”
Further, the plan to modernize New York’s piers proved impractical, leaving the
old piers “languishing” and “rotting.”
ed
!
途
Explanation for Incorrect Answer B :
Choice (B) is incorrect. The author of the passage presents reduced loading time as
a decided advantage of container shipping, in that it “slashed loading and
unloading time to almost nothing”—“Ships that had traditionally spent a week in
port could now finish their loading in a day.”
er
用
业
is
t
Explanation for Incorrect Answer C :
Choice (C) is incorrect. The author of the passage presents theft deterrence as an
advantage of container shipping: “The impregnable containers guaranteed against
waterfront theft . . . eliminating the dockside pilferage that previously had
consumed up to 15 percent of some cargoes.”
商
eg
于
Explanation for Incorrect Answer D :
Choice (D) is incorrect. The author of the passage mentions personnel reduction in
a favorable light in paragraph 4: “A small crew of men could use cranes to load
the gargantuan containers, filling a ship twenty times faster than the old gangs
grappling with crates, boxes, and bales.”
nR
用
禁
Explanation for Incorrect Answer E :
Choice (E) is incorrect. The author of the passage presents breakage prevention as
an advantage of container shipping: “The impregnable containers guaranteed
against waterfront theft or breakage,” implying that both theft and breakage were
problems associated with older shipping practices.
U
严
23
According to the passage, the City of New York was unable to carry out its pier
modernization plan because which of the following was lacking?
(A)
Money
(B)
Space
(C) Time
(D) Materials
(E)
Motivation
ANSWERS
AND EXPLANATIONS
Explanation for Correct Answer B :
Choice (B) is correct. The first two sentences of the last paragraph make it clear
that there was not sufficient space available to fully adapt New York's piers to the
new container shipping system: “The new containers required huge storing areas,
far larger than were available in either Manhattan or Brooklyn. When the City of
New York proposed modernizing its East River piers to handle the containers, the
Port Authority said it would have to clear all the land from the river west to Third
file://E:\新建文件夹\e5.htm
2006-11-12
页码,23/24
The Official SAT Online Course
Avenue to do so.”
Explanation for Incorrect Answer A :
Choice (A) is incorrect. The passage does not mention a lack of financial resources
as being behind the failure of the pier modernization plan.
Explanation for Incorrect Answer C :
Choice (C) is incorrect. The only mention of time in connection with container
shipping in the passage is of how much time was saved in comparison to the older
shipping methods; there is no discussion of there not being sufficient time to
modernize the New York piers.
Explanation for Incorrect Answer D :
Choice (D) is incorrect. The passage does not mention a lack of materials as being
behind the failure of the pier modernization plan.
Explanation for Incorrect Answer E :
Choice (E) is incorrect. The passage illustrates that the City of New York was, on the
contrary, highly motivated to modernize its piers, but that it lacked the space
required to do so.
!
途
d
In the final paragraph of the passage, the author presents an example of
(A)
one man’s vision for the future
(B)
the contrasts between two large industries
(C) the rapid growth of a city
re
24
用
(D) a negative aspect of modernization
te
业
the results of political corruption
ANSWERS
商
is
(E)
AND EXPLANATIONS
eg
于
Explanation for Correct Answer D :
Choice (D) is correct. The last few sentences of the passage create an unmistakable
image of the desolation brought about by modernization: the useless piers,
languishing like “white elephants”; the loss of shipping jobs created by
automation. These are bleak images of the down side of technological progress.
nR
用
禁
U
严
Explanation for Incorrect Answer A :
Choice (A) is incorrect. While the proposal to modernize New York’s piers might
conceivably be called a “vision for the future,” the proposal is a failed one, and
there is no attempt to connect it to “one man.”
Explanation for Incorrect Answer B :
Choice (B) is incorrect. While the paragraph might be said to contain certain
contrasts between container shipping and older shipping methods, these are not
“two large [and separate] industries.”
Explanation for Incorrect Answer C :
Choice (C) is incorrect. The paragraph mentions New York and two cities in New
Jersey, but it is not concerned with their “rapid growth.”
Explanation for Incorrect Answer E :
Choice (E) is incorrect. The dilemma presented in the paragraph is not described as
being the result of political corruption, but of modernization and automation.
Back to Score Report
Copyright © 2006 The College Board. All rights reserved.
file://E:\新建文件夹\e5.htm
Privacy Policy
Terms of Use
Contact Us
2006-11-12
页码,24/24
The Official SAT Online Course
ed
!
途
eg
is
te
r
用
业
商
于
nR
用
禁
U
严
file://E:\新建文件夹\e5.htm
2006-11-12
页码,1/12
The Official SAT Online Course
Help | Profile | My Organizer | My Bookmarks | Logout
Answers and Explanations
Back to Score Report
Test Sections
Section 1
View Answers and Explanations
Section 2
Online - Practice Test #5
Section 3
Section 4
1
Section 5
Section 6
books but
bookcases in a house. Each bookcase contains at least
There are
books. Which of the following could be the total number of books
not more than
bookcases?
in all
Section 8
(A)
Section 9
(B)
Section 10
(C)
(D)
!
途
ANSWERS
re
d
(E)
AND EXPLANATIONS
用
Explanation for Correct Answer D :
bookcases contained the maximum number
Choice (D) is correct. If each of the
If each
then the total number of books would be
业
te
of books,
then the total number of
bookcase contained the minimum number of books,
Therefore, the total number of books in all
books would be
is
商
bookcases must be at least
is in this range.
but no more than
Of the choices, only
eg
于
用
nR
Explanation for Incorrect Answer A :
Choice (A) is not correct. Since each bookcase contains at least
禁
total number of books in all
bookcases must be at least
books, the
Therefore, the
total number of books cannot be
U
严
Explanation for Incorrect Answer B :
Choice (B) is not correct. Since each bookcase contains at least
total number of books in all
bookcases must be at least
books, the
Therefore, the
total number of books cannot be
Explanation for Incorrect Answer C :
Choice (C) is not correct. Since each bookcase contains at least
total number of books in all
bookcases must be at least
books, the
Therefore, the
total number of books cannot be
Explanation for Incorrect Answer E :
Choice (E) is not correct. Since each bookcase contains no more than
the total number of books in all
books,
bookcases can be no more than
Therefore, the total number of books cannot be
2
file://E:\新建文件夹\e6.htm
2006-11-12
页码,2/12
The Official SAT Online Course
what is the value of
are parallel. If
and
In the figure above, lines
(A)
(B)
(C)
(D)
(E)
!
途
AND EXPLANATIONS
ed
ANSWERS
Explanation for Correct Answer E :
Choice (E) is correct. Use the properties of parallel lines cut by a transversal.
er
用
is
t
业
商
eg
于
用
nR
Alternate interior angles are supplementary. Thus, in the figure above,
Since
禁
of equal measure. Thus,
supplementary, so
U
严
Since vertical
it follows that
angles are of equal measure, it follows that
Corresponding angles are also
The angles labeled
and
are
Therefore,
Explanation for Incorrect Answer A :
Choice (A) is not correct. Although from the figure, it may appear that
and
the figure is not drawn to scale.
Explanation for Incorrect Answer B :
Choice (B) is not correct. This incorrect answer may be the result of estimating
angle measures from the figure instead of reasoning from geometric principles.
Explanation for Incorrect Answer C :
Choice (C) is not correct. Although from the figure, it may appear that
and
the figure is not drawn to scale.
Explanation for Incorrect Answer D :
Choice (D) is not correct. This incorrect answer may be the result of estimating
angle measures from the figure instead of reasoning from geometric principles.
3
file://E:\新建文件夹\e6.htm
2006-11-12
页码,3/12
The Official SAT Online Course
Graphed above is the amount that a computer shop charges for a repair job as a
function of the number of hours required to do the job. Which of the following is most
consistent with the information in the graph?
(A)
The shop charges the same amount for any job of any length.
(B)
The shop charges at an hourly rate for work with no fixed initial amount.
(C) The shop charges a fixed initial amount plus an hourly rate for work,
starting at the beginning of the first hour.
!
途
The shop charges at an hourly rate for work beginning at the start of the
hours of work.
first hour. This rate decreases after
ANSWERS
用
er
(E)
ed
hours or less of work. The
(D) The shop charges a fixed amount for the first
hours.
shop charges at an hourly rate for work beyond
AND EXPLANATIONS
业
is
t
Explanation for Correct Answer D :
hours, which
Choice (D) is correct. The graph is a horizontal line segment up to
hours or less of work. Then
corresponds to a fixed amount charged for the first
the graph is a line segment with positive slope, corresponding to an hourly rate for
hours.
work beyond
商
eg
于
nR
用
Explanation for Incorrect Answer A :
Choice (A) is not correct. If the shop charged the same amount for any job of any
length, then the graph would be a horizontal line, which it is not.
禁
Explanation for Incorrect Answer B :
Choice (B) is not correct. If the shop charged at an hourly rate for work with no
fixed initial amount, then the graph would be a line through the origin, which it is
not.
U
严
Explanation for Incorrect Answer C :
Choice (C) is not correct. If the shop charged a fixed initial amount plus an hourly
rate for work, starting at the beginning of the first hour, then the graph would show
hour, but these two charges are the same.
a greater charge for 2 hours than for
Explanation for Incorrect Answer E :
Choice (E) is not correct. If the shop charged at an hourly rate for work beginning
hours of work, then
at the start of the first hour, and the rate decreased after
hours, but it does not.
the slope of the graph would decrease after
4
For how many values of
(A)
file://E:\新建文件夹\e6.htm
is the equation above true?
None
2006-11-12
页码,4/12
The Official SAT Online Course
(B)
One
(C) Two
(D) Four
(E)
More than four
ANSWERS
AND EXPLANATIONS
Explanation for Correct Answer C :
The solutions of these last two equations are
these values of
or
means that either
Choice (C) is correct. The equation
and
Both
and they are the only two values that satisfy
satisfy
Explanation for Incorrect Answer A :
both satisfy
and
Choice (A) is not correct. The values
!
途
Explanation for Incorrect Answer B :
and
both satisfy
er
ed
Choice (B) is not correct. The values
Explanation for Incorrect Answer D :
业
st
Explanation for Incorrect Answer E :
商
and
both satisfy
that satisfy
eg
i
but they are the only two values of
both satisfy
that satisfy
but they are the only two values of
Choice (E) is not correct. The values
用
and
Choice (D) is not correct. The values
于
5
nR
用
禁
U
严
An amusement park awards tickets that can be exchanged for prizes to reward high
scores at an arcade game. Some scores and the corresponding number of prize
the
up to
tickets awarded are shown in the table above. For scores from
-point score
number of tickets awarded increases by a constant amount for each
increase. How many tickets would be awarded for a score of
(A)
(B)
(C)
file://E:\新建文件夹\e6.htm
2006-11-12
页码,5/12
The Official SAT Online Course
(D)
(E)
ANSWERS
AND EXPLANATIONS
Explanation for Correct Answer D :
Choice (D) is correct. For a score of
for a score of
points, there are
-point score
tickets awarded. For this
there are
increase, the increase in the number of tickets awarded is
up to
tickets awarded, and
For scores from
the number of tickets awarded increases by a constant amount for each
-point score increase within this interval
-point score increase. Thus, any
tickets are
Therefore, since
increases the number of tickets awarded by
awarded for a score of
points, it follows that
awarded for a score of
points, and
tickets are
tickets are awarded for a
!
途
re
d
score of
用
Explanation for Incorrect Answer A :
Choice (A) is not correct. The number of tickets awarded for a score of
业
but the number of tickets awarded for a score of
te
is
points
is
Explanation for Incorrect Answer B :
Choice (B) is not correct. The number of tickets awarded for a score of
is
商
points
is
but the number of tickets awarded for a score of
is
eg
于
nR
用
Explanation for Incorrect Answer C :
-point score increase results in an additional
Choice (C) is not correct. Since a
禁
ticket awarded, the number of tickets awarded for a score of
U
严
is
not
Explanation for Incorrect Answer E :
Choice (E) is not correct. The number of tickets awarded for a score of
points
but the number of tickets awarded for a score of
is
is
6
students in a class, only one of whom is a junior. If
There are
to the class, how many students in the class will NOT be juniors?
seniors are added
(A)
(B)
(C)
(D)
(E)
file://E:\新建文件夹\e6.htm
2006-11-12
页码,6/12
The Official SAT Online Course
ANSWERS
AND EXPLANATIONS
Explanation for Correct Answer D :
students in the class, only one of
Choice (D) is correct. There are originally
students in the original class are not juniors. Then
whom is a junior. Thus,
seniors are added to the class; since these students are seniors, they are not
seniors are added to the class, there are
juniors. Therefore, after the
students in the class who are not juniors.
Explanation for Incorrect Answer A :
students in the class were not juniors
Choice (A) is not correct. If exactly
seniors had been added to the class, then originally there would have
after the
students in the class who were not juniors. But originally
been
students who were not juniors.
there were
Explanation for Incorrect Answer B :
students in the class were not juniors
Choice (B) is not correct. If exactly
seniors had been added to the class, then originally there would have
after the
students in the class who were not juniors. But originally
been
!
途
Explanation for Incorrect Answer C :
Choice (C) is not correct. If exactly
d
students who were not juniors.
there were
students in the class were not juniors
re
seniors had been added to the class, then originally there would have
after the
用
students in the class who were not juniors. But originally
been
students who were not juniors.
业
st
e
there were
Explanation for Incorrect Answer E :
seniors were added to the class, there were a
Choice (E) is not correct. After the
students in the class. However, the question asks how many of these
total of
商
eg
i
students were not juniors, which is
于
7
nR
用
禁
U
严
Which of the
The quadratic function graphed above has equation
following must be true?
(A)
(B)
(C)
is even.
(D)
is odd.
(E)
is the square of an integer.
ANSWERS
AND EXPLANATIONS
Explanation for Correct Answer A :
the point
Choice (A) is correct. Since
of
Thus, the intersection of the graph of
origin. Since the graph of
file://E:\新建文件夹\e6.htm
and the
intersects the positive
-axis is
is on the graph
units above the
-axis, the value of
must be
2006-11-12
页码,7/12
The Official SAT Online Course
may be even, odd, or the square of
positive. It cannot be negative. The value of
an integer, but it need not be. Therefore, the only one of the five statements that
must be true is
Explanation for Incorrect Answer B :
is the intersection of the graph of
Choice (B) is not correct. The point
-axis. Since the graph of
the
cannot be negative.
intersects the positive
and
-axis, the value of
Explanation for Incorrect Answer C :
Choice (C) is not correct. It may be true that
be true.
is an even integer, but it need not
Explanation for Incorrect Answer D :
Choice (D) is not correct. It may be true that
be true.
is an odd integer, but it need not
Explanation for Incorrect Answer E :
Choice (E) is not correct. It may be true that
need not be true.
is the square of an integer, but it
!
途
re
d
8
用
is
te
业
商
Which of the following inequalities is true about the lengths
the triangle above?
eg
于
of the sides of
用
nR
(A)
and
(B)
禁
(C)
U
严
(D)
(E)
ANSWERS
AND EXPLANATIONS
Explanation for Correct Answer E :
Choice (E) is correct. The length of each side of a triangle is less than the sum of
Squaring both sides of this
the lengths of the other two sides; thus,
inequality yields
Explanation for Incorrect Answer A :
Choice (A) is not correct. The length of each side of a triangle is less than the sum
Squaring both sides of this
of the lengths of the other two sides; thus,
inequality yields
not
Explanation for Incorrect Answer B :
Choice (B) is not correct. The length of each side of a triangle is less than the sum
Squaring both sides of this
of the lengths of the other two sides; thus,
inequality yields
file://E:\新建文件夹\e6.htm
not
2006-11-12
页码,8/12
The Official SAT Online Course
Explanation for Incorrect Answer C :
Choice (C) is not correct. The length of each side of a triangle is less than the sum
Squaring both sides of this
of the lengths of the other two sides; thus,
not
inequality yields
Explanation for Incorrect Answer D :
Choice (D) is not correct. The length of each side of a triangle is less than the sum
Squaring both sides of this
of the lengths of the other two sides; thus,
not
inequality yields
9
what is the radius of the circle?
If a circle has circumference
Your Response:
Correct Response(s): 3.5, 7/2
Explanation:
where
. The circumference of a circle is equal to
or
The correct answer is
!
途
Solving for
is the radius of the circle. Thus,
gives
.
用
are positive integers, what is one possible value of
and
and
If
业
te
10
re
d
The answer may also be gridded as the equivalent decimal
Your Response:
is
商
Correct Response(s): 5 or 10 or 15 or 20
Explanation:
eg
于
nR
用
must be a factor of
Since
and
are
values of
be gridded as the answer.
are
is positive,
and
respectively.) Any one of
(The corresponding
or
may
严
U
11
have no prime factors in common,
and, since
In addition,
Therefore, the possible values of
禁
and
then
. If
and
The possible correct answers are
floors of an
The figure above shows, on the left, the call buttons for the
buttons labeled A through I for the
apartment building, and on the right, the
file://E:\新建文件夹\e6.htm
2006-11-12
页码,9/12
The Official SAT Online Course
apartments on each floor of the building. How many different apartments can be
called by pressing one button on the left and one button on the right?
Your Response:
Correct Response(s): 108
Explanation:
floors in the building, and there are
. There are
The correct answer is
floors. Therefore, by pressing one button on the left,
apartments on each of the
floors, and then one button on the right, for one of the
for one of the
different apartments can be called.
apartments on that floor,
12
If the function
for what value of
is defined by
Your Response:
!
途
ed
Correct Response(s): 66
Explanation:
用
then
. If
therefore,
er
The correct answer is
does
业
is
t
and
商
13
eg
于
nR
用
禁
U
严
above
Students at Northern High School are represented in Figure above. Figure
students
is another way to show which pets these students have. If the same
are represented in both figures, what is the total number of students represented by
the shaded circle in Figure
file://E:\新建文件夹\e6.htm
2006-11-12
页码,10/12
The Official SAT Online Course
Your Response:
Correct Response(s): 275
Explanation:
represents all students
. The shaded circle in Figure
The correct answer is
at Northern High School who have cats. This includes students who have cats only
of the
shows that
and students who have both cats and dogs. Figure
have both cats and
students at Northern High School have cats only and
of the
represents
dogs. Thus, the shaded circle in Figure
students at Northern High School. Since both figures represent the same
of
represents
students, the shaded circle in Figure
students, that is,
students.
14
what is the value of
is a positive integer, and
If
!
途
Your Response:
d
Correct Response(s): 25
用
to both sides of
. Adding
业
te
The correct answer is
re
Explanation:
dividing both sides of this equation by
equation
Therefore, the value of
gives the
gives the equation
is
is
商
于
eg
15
If
what is the value of
nR
用
Your Response:
禁
Correct Response(s): 2
U
严
Explanation:
The correct answer is
Since
Therefore,
. Use the distributive law.
it follows that
and the value of
is
.
16
file://E:\新建文件夹\e6.htm
2006-11-12
页码,11/12
The Official SAT Online Course
inches. What is the volume, in cubic inches, of the
The sphere above has radius
smallest cube that can contain the entire sphere?
Your Response:
Correct Response(s): 1728
Explanation:
!
途
.
re
d
The correct answer is
用
te
业
is
商
nR
eg
于
用
As shown above, when the sphere is placed inside the smallest cube that can
contain it, the greatest distance between two points on the sphere (the diameter of
the sphere) must be equal to the shortest distance between two points on opposite
faces of the cube (which is equal to the edge length of the cube). Thus, the edge
inches.
length of this cube will be equal to the diameter of the sphere, which is
.
The volume of the cube, in cubic inches, will be
禁
U
严
17
is twice the value of
If the value of
what is the value of
Your Response:
Correct Response(s): 4
Explanation:
The correct answer is
. Since the value of
is twice the value of
gives
is true. Cross-multiplying
equation
or
It follows that
the
or
18
file://E:\新建文件夹\e6.htm
2006-11-12
页码,12/12
The Official SAT Online Course
In the
-intercept of line
what is the
are
If the coordinates of point
is perpendicular to
and
-coordinate plane above,
Your Response:
Correct Response(s): 16/3, 5.33
Explanation:
!
途
is perpendicular to
. Since
or
The correct answer is
it
ed
is a right triangle. By the Pythagorean Theorem,
follows that
are
The coordinates of point
are
Thus, the slope of line
are
业
where
is
is the
is
t
of line
用
and the coordinates of
er
so the coordinates of
商
so
Hence the equation
is
-intercept of
Therefore,
The answer can be
or as its rounded decimal equivalent,
于
eg
gridded as
is on
Point
.
nR
用
严
禁
U
Copyright © 2006 The College Board. All rights reserved.
file://E:\新建文件夹\e6.htm
Back to Score Report
Privacy Policy
Terms of Use
Contact Us
2006-11-12
页码,1/24
The Official SAT Online Course
Help | Profile | My Organizer | My Bookmarks | Logout
Answers and Explanations
Back to Score Report
Test Sections
Section 1
View Answers and Explanations
Section 2
Online - Practice Test #5
Section 3
Section 4
1
Section 5
Numismatics, or coin collecting, and becoming popular in the United States around
1857, when the replacement of the large cent by the new flying-eagle cent led
enthusiasts to start collecting the earlier coin.
Section 6
Section 8
Section 9
(A)
and becoming
(B)
becoming
(C) will become
Section 10
(D) became
it became
ANSWERS
!
途
AND EXPLANATIONS
re
d
(E)
Explanation for Correct Answer D :
Choice (D) is correct. It avoids the error of the original by providing a main verb
(“became”) to perform the action of the sentence (“Numismatics … became
…”).
te
用
业
is
商
eg
Explanation for Incorrect Answer A :
Choice (A) involves improper coordination that results in a sentence fragment. The
coordinating conjunction “and” is not needed, and the participle “becoming”
cannot serve as the main verb in this sentence. The other verb, “led,” is the verb
for the subordinating clause beginning with “when” and cannot serve as the main
verb. A main verb is needed to complete the action of the sentence.
于
nR
用
Explanation for Incorrect Answer B :
Choice (B) creates a sentence fragment. The participle “becoming” cannot serve
as a main verb. The other verb, “led,” is the verb for the subordinating clause
beginning with “when” and cannot serve as the main verb. Therefore, a main
verb is needed to complete the action of the sentence.
禁
U
严
Explanation for Incorrect Answer C :
Choice (C) involves an error in verb tense that results in faulty logic. The action of
the sentence takes place in the past (“around 1857”), so a past-tense verb,
“became,” not a future-tense verb, is needed.
Explanation for Incorrect Answer E :
Choice (E) involves an error in pronoun use that results in a sentence fragment.
The pronoun “it” is used unnecessarily, so the subject of the sentence,
“Numismatics,” has no main verb to complete the action of the sentence.
2
Just as his predecessor Mozart, for whom composing music seemed to come easily,
Beethoven always struggled to perfect his work.
(A)
Just as
(B)
Even though
(C) Whereas
(D) Unlike with
(E)
file://E:\新建文件夹\e7.htm
Unlike
2006-11-12
页码,2/24
The Official SAT Online Course
ANSWERS
AND EXPLANATIONS
Explanation for Correct Answer E :
Choice (E) is correct. It avoids the improper word choice of the original by replacing
the adverb “Just” and the preposition “as” with the appropriate preposition
“Unlike” to indicate that the idea expressed in the prepositional phrase
(“Unlike…Mozart”) contrasts with the idea expressed in the independent clause
(“Beethoven…work”).
Explanation for Incorrect Answer A :
Choice (A) involves inappropriate word choice that creates faulty logic. The phrase
“Just as” is inappropriate in the context of this sentence because it expresses
similarity, not contrast. The sentence explains how Beethoven was not like Mozart,
so the preposition “Unlike” is needed to indicate that the idea expressed in the
prepositional phrase (“Unlike…Mozart”) contrasts with the idea expressed in the
independent clause (“Beethoven…work”).
Explanation for Incorrect Answer B :
Choice (B) involves the misuse of a subordinating conjunction. The subordinating
conjunction “even though” does express contrast, but it is used inappropriately
to introduce a prepositional phrase, not a subordinate clause.
!
途
re
d
Explanation for Incorrect Answer C :
Choice (C) involves the misuse of a subordinating conjunction. The subordinating
conjunction “whereas” does express contrast, but it is used inappropriately to
introduce a prepositional phrase, not a subordinate clause.
用
te
Explanation for Incorrect Answer D :
Choice (D) involves an unnecessary preposition. The preposition “Unlike” is
appropriate to introduce a contrasting idea, but the added preposition “with” is
unnecessary in the context of this sentence.
is
业
eg
商
3
于
The Incas kept records with knotted cords called quipus, historians believe they were
used to document crop production and to aid in tax collection.
用
historians believe they were used
(B)
historians believing in its use
nR
(A)
禁
(C) which historians believe they use
U
严
(D) which historians believe were used
(E)
and historians believe it was used
ANSWERS
AND EXPLANATIONS
Explanation for Correct Answer D :
Choice (D) is correct. It avoids the comma-splice error and ambiguous pronoun of
the original by turning the second independent clause (“historians … collection”)
into a subordinate clause introduced by the relative pronoun “which,” and by
making clear that the relative pronoun “which” refers to the noun “quipus.”
Explanation for Incorrect Answer A :
Choice (A) involves a comma-splice error and an ambiguous pronoun. Two
independent clauses (“The … quipus” and “historians … collection”) are
joined only by a comma rather than by a comma and a coordinating conjunction or
by a semicolon. Also, the plural pronoun “they” could refer to “Incas,”
“records,” or “quipus.”
Explanation for Incorrect Answer B :
Choice (B) contains awkward phrasing and an error in pronoun–antecedent
agreement. The singular possessive pronoun “its” does not agree in number with
the plural noun “quipus” to which it refers. Also, in the context of this sentence,
the use of the preposition “in” after the participle “believing” is not idiomatic.
file://E:\新建文件夹\e7.htm
2006-11-12
页码,3/24
The Official SAT Online Course
Explanation for Incorrect Answer C :
Choice (C) involves an error in verb tense and an unclear pronoun reference. The
present tense of the verb “use” is inconsistent with the past tense of the main
verb “kept.” Also, the plural pronoun “they” could refer either to “Incas” or
(illogically) to “historians.”
Explanation for Incorrect Answer E :
Choice (E) creates an error in pronoun–antecedent agreement. Although the
addition of the coordinating conjunction “and” corrects the comma-splice error of
the original, the singular pronoun “its” does not agree with the plural noun
“quipus” to which it refers.
4
While females were not allowed to compete in the Olympic games of ancient Greece;
they were not even allowed to watch them.
(A)
While females were not allowed
(B)
Even though females were not allowed
(C) Females were not allowed
(D) They did not allow females
(E)
With females, they were not allowed
!
途
AND EXPLANATIONS
d
ANSWERS
re
Explanation for Correct Answer C :
Choice (C) is correct. It avoids the improper subordination of the original by
deleting the subordinating conjunction “While” and thereby making the clause an
independent clause, which can stand alone.
st
e
用
业
商
eg
i
Explanation for Incorrect Answer A :
Choice (A) involves improper subordination. The first clause is a subordinate clause
because it is introduced by a subordinate conjunction (“While”), but it cannot
stand alone (as indicated by the semicolon). To make the clause an independent
clause that is equal in importance to the other independent clause, the
subordinating conjunction “While” should be omitted.
于
nR
用
Explanation for Incorrect Answer B :
Choice (B) involves improper subordination. The first clause is a subordinate clause
because it is introduced by a subordinate conjunction (“Even though”), but it
cannot stand alone (as indicated by the semicolon). To make the clause an
independent clause that is equal in importance to the sentence’s other
independent clause, the subordinating conjunction “Even though” should be
omitted.
禁
U
严
Explanation for Incorrect Answer D :
Choice (D) involves a pronoun error. Although the first clause has been turned into
an independent clause, equal in importance to the other independent clause of the
sentence, it now has a pronoun (“They”) without an antecedent to which it
refers.
Explanation for Incorrect Answer E :
Choice (E) involves awkward phrasing. The addition of the preposition “With” and
the pronoun “they” is unnecessary. The noun “females” can stand on its own
as the subject of the first independent clause.
5
In psychological theory, the term social learning is used to describe learning that is
influenced by one’s environment and not the influence of innate or internal forces.
(A)
and not the influence of
(B)
and not, instead,
(C) rather than by
(D) rather than when it is
(E)
file://E:\新建文件夹\e7.htm
rather than being influenced due to
2006-11-12
页码,4/24
The Official SAT Online Course
ANSWERS
AND EXPLANATIONS
Explanation for Correct Answer C :
Choice (C) is correct. It avoids the error in parallelism and faulty logic of the
original by replacing the phrase “and not the influence of” with the conjunction
“rather than” and the preposition “by.” This revision makes the grammatical
structure that precedes the conjunction “rather than” parallel with the
grammatical structure that follows it. This revision also fixes the faulty logic in the
phrase “learning that is … not the influence.”
Explanation for Incorrect Answer A :
Choice (A) involves a lack of parallelism and faulty logic. The grammatical structure
of the phrase after “and” is made up of an adverb (“not”) modifying “is”
and a noun (“the influence”) that functions as a subject complement renaming
“learning.” It does not make sense to say “learning that is not the influence.”
Also, this grammatical structure is not parallel with the first structure (“influenced
by one’s environment”). Replacing this phrase with the “rather than by” fixes
both problems.
Explanation for Incorrect Answer B :
Choice (B) involves redundancy and a lack of parallelism. The adverbs “not” and
“instead” are redundant, so “instead” is not needed. Also, for the structure
following “and” to be parallel with the structure preceding “and,” the
preposition “by” is needed: “is influenced by … and not by ….”
er
ed
!
途
Explanation for Incorrect Answer D :
Choice (D) involves a lack of parallelism. The structure following the conjunction
“rather than” is a subordinate clause introduced by the relative adverb
“when.” It is not parallel with the prepositional phrase (“by one’s
environment”) that precedes the conjunction. Also, the phrase “when it is” is
awkward and can be replaced by the simple preposition “by.”
用
is
t
业
Explanation for Incorrect Answer E :
Choice (E) involves an error in verb tense and awkward phrasing. The progressive
tense of the verb “being” is inconsistent with the simple present tense of the
verb “is.” Also, the phrase “rather than being influenced due to” can be
replaced by the much simpler, and more idiomatic, phrase “rather than by.”
eg
商
用
nR
6
于
Giant stars, or those that weigh 100 times what the Sun weighs, are important to
galaxies and the universe because with furious combustion, it produces many
important elements that form planets and other bodies, including carbon, oxygen,
sodium, and neon.
U
严
禁
(A)
because with furious combustion, it produces
(B)
because their furious combustion produces
(C) due to their furious combustion, producing
(D) as their furious combustion produced
(E)
in that with furious combustion, they produced
ANSWERS
AND EXPLANATIONS
Explanation for Correct Answer B :
Choice (B) is correct. It avoids the pronoun error and imprecise language of the
original by removing the pronoun “it,” which could refer to “the Sun,” “ the
universe,” or “combustion.”
Explanation for Incorrect Answer A :
Choice (A) involves a pronoun error and imprecise language. The antecedent of the
pronoun “it” is unclear; it could be “the Sun,” “the universe,” or
“combustion.”
Explanation for Incorrect Answer C :
Choice (C) involves faulty logic, imprecise language, and an unclear pronoun
file://E:\新建文件夹\e7.htm
2006-11-12
页码,5/24
The Official SAT Online Course
reference. The prepositional phrase introduced by “due to” creates faulty logic. It
illogically suggests that the giant stars are important because of their combustion,
not because of what their combustion produces. Also, the participial phrase
introduced by “producing” describes the subject, “Giant stars,” and imprecisely
suggests that the giant stars, not their combustion, produce many important
elements.
Explanation for Incorrect Answer D :
Choice (D) contains an incorrect verb tense and ambiguous language. The past
tense of “produced” is inconsistent with the present tense of the main verb
“are” and also illogically suggests that the combustion of giant stars no longer
occurs. The subordinating conjunction “as” could mean “when” or
“because.” In the context of this sentence, its meaning is not clear.
Explanation for Incorrect Answer E :
Choice (E) involves imprecise language and an incorrect verb tense. The
subordinate clause introduced by “in that” is imprecise, suggesting that the giant
stars, not their combustion, produce many important elements. Also, the past tense
of “produced” is inconsistent with the present tense of the main verb “are” and
illogically suggests that the combustion of giant stars no longer occurs.
American photographer Annie Leibovitz is renowned for her revealing, eye-catching
portraits of celebrities, and characterized by carefully staged settings and the use of
vivid primary colors.
(A)
and
(B)
and she is
!
途
ed
7
(C) each of them are
用
(E)
which are
st
er
(D) each of which are
ANSWERS
业
AND EXPLANATIONS
商
eg
i
Explanation for Correct Answer E :
Choice (E) is correct. It avoids the errors of the other options by providing a
relative pronoun, “which,” to introduce the subordinate clause that describes
“portraits” and by providing the plural verb “are,” which is the appropriate
helping verb for the passive verb “characterized.”
于
nR
用
禁
Explanation for Incorrect Answer A :
Choice (A) involves an incorrectly used coordinating conjunction and faulty logic.
The coordinating conjunction “and” that precedes the verb “characterized”
forms a compound verb for the subject of the independent clause, “Annie
Leibovitz.” It illogically suggests that Annie Leibovitz, not her portraits, is
“characterized by carefully staged settings and the use of vivid primary colors.”
U
严
Explanation for Incorrect Answer B :
Choice (B) involves faulty logic. The addition of the pronoun “she,” which refers
to the subject of the sentence, “Annie Leibovitz,” and the addition of the singular
helping verb “is” create a second independent clause. This new clause illogically
suggests that Annie Leibovitz, not her portraits, is “characterized by carefully
staged settings and the use of vivid primary colors.”
Explanation for Incorrect Answer C :
Choice (C) creates a comma-splice error and a subject–verb agreement error. The
addition of a subject (“each”) and a helping verb (“are”) creates an
independent clause connected to the other independent clause (“American …
celebrities”) by only a comma. Also, the plural verb “are” does not agree in
number with its subject, the singular pronoun “each.”
Explanation for Incorrect Answer D :
Choice (D) involves an error in subject–verb agreement. Although the addition of
the phrase “each of which are” creates a subordinate clause that modifies
“portraits,” the plural verb “are” does not agree in number with its subject,
the singular pronoun “each.”
file://E:\新建文件夹\e7.htm
2006-11-12
页码,6/24
The Official SAT Online Course
8
Decorative arts, art forms that have a mainly practical or ornamental purpose, and
include ceramics, basketry, furniture making, and glassblowing.
(A) and include
(B)
including
(C) includes
(D) include
(E)
which include
ANSWERS
AND EXPLANATIONS
Explanation for Correct Answer D :
Choice (D) is correct. It avoids the errors of the original by removing the
unnecessary coordinating conjunction “and,” so that the sentence has a main
plural verb, “include,” that agrees with the plural subject, “Decorative arts.”
Explanation for Incorrect Answer A :
Choice (A) involves an unnecessary coordinating conjunction, “and.” The
conjunction “and” connects the verbs “have” and “include” as compound
verbs of the subordinate clause, so the subject of the main clause has no verb to
complete the action of the sentence. To make “include” the verb of the
independent clause, the “and” should be omitted.
ed
!
途
Explanation for Incorrect Answer B :
Choice (B) results in a sentence fragment. The participle “including” introduces a
participial phrase that functions as an adjective describing “arts,” so the
sentence has no main verb. To fix this problem, the verb “include” is needed.
is
te
r
用
业
Explanation for Incorrect Answer C :
Choice (C) involves an error in subject–verb agreement. The singular verb
“includes” does not agree with the plural noun and subject of the sentence,
“arts.”
eg
商
于
Explanation for Incorrect Answer E :
Choice (E) creates a sentence fragment. The relative pronoun “which” makes all
that follows it a subordinate clause that describes “arts,” so the sentence has no
main verb. Without the relative pronoun, the verb “include” can serve as the
main verb of the sentence.
nR
用
禁
严
U
9
About 35 percent of the world's orange juice is produced by Florida, compared with
Brazil, the world’s largest orange producer, which has nearly 50 percent.
(A)
Brazil, the world’s largest orange producer, which has nearly 50 percent
(B)
Brazil, with nearly 50 percent, it is the world’s largest orange producer
(C) nearly 50 percent that are produced by Brazil as the world’s largest
orange producer
(D) the production of Brazil, the world’s largest orange producer, with nearly
50 percent
(E)
nearly 50 percent produced by Brazil, the world’s largest orange
producer
ANSWERS
AND EXPLANATIONS
Explanation for Correct Answer E :
Choice (E) is correct. It avoids the illogical comparison of the original by replacing
“Brazil” with “50 percent” as the object of the multi-word preposition
“compared with.”
Explanation for Incorrect Answer A :
Choice (A) involves an illogical comparison. The prepositional phrase introduced by
the multi-word preposition “compared with” functions as an adjective modifying
file://E:\新建文件夹\e7.htm
2006-11-12
页码,7/24
The Official SAT Online Course
the subject of the sentence, “35 percent,” so the amount, 35 percent, is
illogically compared with the country, “Brazil,” rather than with another amount,
“50 percent.”
Explanation for Incorrect Answer B :
Choice (B) involves an illogical comparison and a comma-splice error. The
prepositional phrase introduced by the multi-word preposition “compared with”
functions as an adjective modifying the subject of the sentence, “35 percent,” so
the amount, “35 percent,” is illogically compared with the country, “Brazil,”
rather than with another amount, “50 percent.” Also, two independent clauses
(“About … percent” and “it … producer”) are joined only by a comma rather
than by a semicolon or by a comma and a subordinating conjunction.
Explanation for Incorrect Answer C :
Choice (C) contains an error in subject–verb agreement and awkward phrasing.
The relative pronoun “that” refers to the singular amount noun “50 percent,”
so it requires the singular verb “is.” Also, the long phrase beginning with
“nearly,” which includes an unnecessary subordinate clause (“that are produced
by Brazil”) and preposition (“as”), can be reduced to the simpler phrase
“nearly 50 percent produced by Brazil, the world’s largest orange producer.”
Explanation for Incorrect Answer D :
Choice (D) contains a non-idiomatic preposition and awkward phrasing. In the
context of this sentence, the prepositional phrase “production of Brazil” is not
idiomatic and creates nonsense. Brazil is not being produced. The idiomatic
preposition in this case is “by” (“production by Brazil”). Also, the long phrase
beginning with “nearly” can be reduced to the simpler phrase “nearly 50
percent produced by Brazil, the world’s largest orange producer.”
用
Many of blues great Bessie Smith’s songs describe the experiences of southern Black
migrants, especially the struggles of Black women to adjust to urban life in the
northern United States.
业
(A)
the struggles of Black women to adjust
(B)
how Black women struggled at adjusting
is
t
10
er
e
d
!
途
商
eg
(C) when it was Black women struggling to adjust
于
(D) Black women, they struggled with adjusting
(E)
for Black women and struggling to adjust
nR
用
ANSWERS
禁
AND EXPLANATIONS
Explanation for Correct Answer A :
Choice (A) is correct. It avoids the errors of the other options by providing an
appositive, “the struggles of Black women to adjust,” to rename the
“experiences” that Bessie Smith sang about.
U
严
Explanation for Incorrect Answer B :
Choice (B) involves awkward, unidiomatic phrasing. The infinitive “to adjust” is
the idiomatic verb form to follow “struggled” in this context. In addition, the
entire awkward phrase “how Black women struggled at adjusting” can be
reduced to the more precise “the struggles of Black women to adjust.”
Explanation for Incorrect Answer C :
Choice (C) involves unnecessary pronoun use and faulty logic. It does not make
sense to use the relative adverb “when” to describe “experiences.” In
addition, there is nothing in the sentence to which the pronoun “it” can logically
refer.
Explanation for Incorrect Answer D :
Choice (D) involves faulty logic. What comes after “especially” should be an
appositive that renames or describes “the experiences” that Bessie Smith sang
about, but people (“Black women”) cannot logically be “experiences.”
Explanation for Incorrect Answer E :
Choice (E) involves awkward and wordy phrasing, resulting in nonsense. What
comes after “especially” should be an appositive that renames or describes “the
experiences” that Bessie Smith sang about, but the prepositional phrase “for
file://E:\新建文件夹\e7.htm
2006-11-12
页码,8/24
The Official SAT Online Course
Black women” cannot logically be “experiences.” In addition, the entire
awkward phrase “for Black women and struggling to adjust” can be reduced to
the more precise phrase “struggles of Black women to adjust.”
11
The Bretons who began emigrating to the United States from the Brittany region of
France in the early twentieth century were distinguished from other French citizens by
their Celtic origin, but about 40 percent of those who emigrated spoke a Celtic
language closely related to Welsh.
(A) but about 40 percent of those who emigrated spoke
(B)
about 40 percent of whom spoke
(C) about 40 percent of which were speaking
(D) with about 40 percent of those who emigrated speaking
(E)
where 40 percent of them were speaking
ANSWERS
AND EXPLANATIONS
Explanation for Correct Answer D :
Choice (D) is correct. It avoids the coordination error of the original by deleting the
coordinating conjunction “but” and turning the independent clause (“but …
Welsh”) into a prepositional phrase that functions as an adverb modifying the verb
“were distinguished.” The coordinating conjunction “but” indicates contrast,
but the idea expressed in the clause following “but” does not contrast with the
idea in the clause preceding “but.” Instead, the clause following “but” explains
how 40 percent of the French immigrants were distinguished from the other French
immigrants.
ed
!
途
er
用
is
t
业
Explanation for Incorrect Answer A :
Choice (A) involves a coordination error. The coordinating conjunction “but”
indicates contrast, but the idea expressed in the clause following “but” does not
contrast with the idea in the clause preceding “but.” Instead, the clause following
“but” explains how 40 percent of the French immigrants were distinguished from
the other French immigrants.
eg
商
于
用
nR
Explanation for Incorrect Answer B :
Choice (B) involves an unclear pronoun. The relative pronoun “whom” has no
clear antecedent; it could refer to “Bretons” or to “French citizens.”
禁
Explanation for Incorrect Answer C :
Choice (C) involves errors in pronoun reference. The relative pronoun “which”
has no clear antecedent; it could refer to “Bretons” or to “French citizens.” In
addition, “which” should be used to refer to things, not to people.
U
严
Explanation for Incorrect Answer E :
Choice (E) involves an inappropriate relative adverb and a verb-tense error. The
relative adverb “where” introduces a subordinate clause that functions as an
adjective, but the noun to which it refers not clear. It could be “United States,”
“Brittany region,” or “France.”
12
Although they are not fast runners, wolves could have maintained a loping run
for many miles, running throughout the night if necessary. No error
ANSWERS
AND EXPLANATIONS
Corrected Sentence: Although they are not fast runners, wolves can maintain a
loping run for many miles, running throughout the night if necessary.
Explanation for Correct Answer B :
file://E:\新建文件夹\e7.htm
2006-11-12
页码,9/24
The Official SAT Online Course
The error in this sentence occurs at (B), where an incorrect verb tense is used. The
conditional present perfect tense of “could have maintained” is not consistent
with the simple present tense of the subordinate clause (“are”).
Explanation for Incorrect Answer A :
There is no error at (A). The plural pronoun “they” agrees in number with the
plural noun to which it refers, “wolves”; the plural verb “are” agrees with the
plural subject of the subordinate clause, “they”; and the adverb “not”
appropriately modifies the verb “are.”
Explanation for Incorrect Answer C :
There is no error at (C). The preposition “for” is an appropriate preposition to
introduce the prepositional phrase (“for many miles”) that functions as an
adjective describing the noun “run.” The adjective “many” appropriately
modifies the noun “miles.”
Explanation for Incorrect Answer D :
There is no error at (D). The preposition “throughout” properly introduces the
prepositional phrase “throughout the night,” which functions as an adverb
modifying the participle “running.”
Explanation for Incorrect Answer E : There is an error in the sentence.
belong
internationally
to his
famous as a writer
用
te
r
The credit for making Franz Kafka
ed
13
!
途
friend, novelist Max Brod, who editedKafka’s unpublished manuscripts and thenhad
业
them published, despite Kafka’s dying wishes to the contrary.
eg
is
商
ANSWERS
No error
于
AND EXPLANATIONS
nR
用
Corrected Sentence: The credit for making Franz Kafka internationally famous
belongs to his friend, novelist Max Brod, who edited Kafka’s unpublished
manuscripts and then had them published, despite Kafka’s dying wishes to the
contrary.
禁
U
严
Explanation for Correct Answer B :
The error in this sentence occurs at (B), where a plural verb (“belong”) does not
agree in number with its singular subject (“credit”).
Explanation for Incorrect Answer A :
There is no error at (A). The adverb “internationally” appropriately modifies the
adjective “famous.”
Explanation for Incorrect Answer C :
There is no error at (C). The relative pronoun “who” properly refers to the proper
noun “Max Brod,” a person, and properly introduces the subordinate clause
(“who … published”) that describes that proper noun.
Explanation for Incorrect Answer D :
There is no error at (D). The coordinating conjunction “and” properly connects
the two verb phrases (“edited … manuscripts” and “then … published”) that
complete the action of the subordinate clause introduced by the relative pronoun
“who.” The adverb “then” appropriately modifies the verb “had.”
Explanation for Incorrect Answer E : There is an error in the sentence.
file://E:\新建文件夹\e7.htm
2006-11-12
页码,10/24
The Official SAT Online Course
14
Unlike
country-and-western bands,
fiddle, rock bands
which
often feature the harmonica, banjo, or
tended to use electronic
equipment, including amplifiers, guitars,
No error
and organs.
ANSWERS
AND EXPLANATIONS
Corrected Sentence: Unlike country-and-western bands, which often feature the
harmonica, banjo, or fiddle, rock bands tend to use electronic equipment, including
amplifiers, guitars, and organs.
Explanation for Correct Answer C :
The error in this sentence occurs at (C), where an incorrect verb tense is used. The
past tense of “tended” is not consistent with the simple present tense of the
subordinate clause, “feature.” The simple present tense (“tend”) is needed
instead.
d
!
途
re
Explanation for Incorrect Answer A :
There is no error at (A). The preposition “unlike” is an appropriate preposition to
introduce the prepositional phrase (“Unlike … bands”) that modifies the subject
of the independent clause (“rock bands”) and to signal that a contrasting idea
will follow.
te
用
业
Explanation for Incorrect Answer B :
There is no error at (B). The relative pronoun “which” properly refers to the noun
“bands” and properly introduces the subordinate clause (“which … fiddle”)
that describes “bands.”
eg
is
商
Explanation for Incorrect Answer D :
There is no error at (D). The adjective “electronic” appropriately modifies the
noun “equipment.”
于
用
禁
严
U
15
nR
Explanation for Incorrect Answer E : There is an error in the sentence.
Like her nonfiction, Jean Craighead George’s fiction draws extensively not only from
published material but also she had firsthand observations of animals and ecological
systems. No error
ANSWERS
AND EXPLANATIONS
Corrected Sentence: Like her nonfiction, Jean Craighead George’s fiction draws
extensively not only from published material but also from firsthand observations of
animals and ecological systems.
Explanation for Correct Answer C :
The error in this sentence occurs at (C), where there is an error in parallel
structure. The correlative conjunctions “not only … but also” require that the
grammatical structure following the first half the pair is the same as the
grammatical structure following the second half. The independent clause “she had
firsthand observations” is not grammatically parallel to the prepositional phrase,
“from published material,” that follows “not only.” The preposition “from” is
needed instead (“but also from…”)
file://E:\新建文件夹\e7.htm
2006-11-12
页码,11/24
The Official SAT Online Course
Explanation for Incorrect Answer A :
There is no error at (A). The preposition “like” is an appropriate preposition to
introduce the prepositional phrase (“like … nonfiction”) that modifies the subject
of the independent clause, “fiction.”
Explanation for Incorrect Answer B :
There is no error at (B). The singular verb “draws” agrees in number with its
singular subject “fiction,” and the adverb “extensively” appropriately modifies
the verb “draws.”
Explanation for Incorrect Answer D :
There is no error at (D). The coordinating conjunction “and” appropriately
connects the two objects of the preposition “of,” “animals” and “systems.”
Explanation for Incorrect Answer E : There is an error in the sentence.
16
Most
through
ships move
assisted by
a tugboat.
so
extremely large
!
途
No error
用
AND EXPLANATIONS
te
ANSWERS
re
d
ships must be
the Suez Canal under their own power,
业
Corrected Sentence: Most ships move through the Suez Canal under their own
power, but extremely large ships must be assisted by a tugboat.
eg
is
商
Explanation for Correct Answer C :
The error in this sentence occurs at (C), where an inappropriate coordinating
conjunction, “so,” connects two independent clauses that express contrasting
ideas. The coordinating conjunction “but” is needed to properly express the
contrast between the ideas in the sentence.
于
nR
用
禁
Explanation for Incorrect Answer A :
There is no error at (A). The word “Most” appropriately functions as an adjective
modifying the noun “ships.”
U
严
Explanation for Incorrect Answer B :
There is no error at (B). The preposition “through” is an idiomatic preposition to
follow the verb “move” and properly introduces the prepositional phrase
(“through the Suez Canal”) that functions as an adverb modifying the verb
“move.”
Explanation for Incorrect Answer D :
There is no error at (D). The past participle “assisted” is the appropriate verb
form to create the present tense of a passive verb, “be assisted.” The preposition
“by” is the idiomatic preposition to follow a passive verb and to introduce the
prepositional phrase that modifies that verb.
Explanation for Incorrect Answer E : There is an error in the sentence.
17
Ralph Ellison’s Invisible Man is the story of a nameless young Black man who
ultimately decides to forge his own identity rather than accept the one assigned to
file://E:\新建文件夹\e7.htm
2006-11-12
页码,12/24
The Official SAT Online Course
No error
him.
ANSWERS
AND EXPLANATIONS
Corrected Sentence:
Explanation for Correct Answer E : There is no error in this sentence.
Explanation for Incorrect Answer A :
There is no error at (A). The singular verb “is” agrees in number with its singular
subject, “Ralph Ellison’s Invisible Man.”
Explanation for Incorrect Answer B :
There is no error at (B). The adverb “ultimately” correctly modifies the verb
“decides,” and the singular “decides” agrees in number with the singular
subject of the subordinate clause, “man.”
!
途
re
d
Explanation for Incorrect Answer C :
There is no error at (C). The phrase “rather than” is an idiomatic phrase that
serves as a conjunction meaning “and not.” It correctly connects the two phrases
“forge his own identity” and “accept the one assigned to him.”
用
te
Explanation for Incorrect Answer D :
There is no error at (D). The definite article “the” properly modifies the pronoun
“one,” which, in turn, properly serves as the direct object of “accept.”
业
eg
is
商
18
The Sun was shining for nearly five billion years and is thoughtto have sufficient
于
thermonuclear fuel in its core to shine for about another five billion.
No error
nR
用
禁
ANSWERS
AND EXPLANATIONS
U
严
Corrected Sentence: The Sun has been shining for nearly five billion years and is
thought to have sufficient thermonuclear fuel in its core to shine for about another
five billion.
Explanation for Correct Answer A :
The error in this sentence occurs at (A), where an incorrect verb tense is used. The
past progressive tense of “was shining” is incorrect for expressing an action that
started in the past and is still occurring. The present perfect progressive, “has
been shining,” is needed instead.
Explanation for Incorrect Answer B :
There is no error at (B). The singular verb “is” agrees in number with the
singular subject of the sentence, “Sun,” and the past participle “thought” is
the appropriate verb form to create the present tense of a passive verb, “is
thought.”
Explanation for Incorrect Answer C :
There is no error at (C). The adjective “sufficient” appropriately describes the
noun “fuel.”
Explanation for Incorrect Answer D :
There is no error at (D). The preposition “in” properly introduces the
prepositional phrase “in its core,” which functions as an adjective describing the
noun “fuel”; the singular possessive pronoun “its” agrees with the singular
file://E:\新建文件夹\e7.htm
2006-11-12
页码,13/24
The Official SAT Online Course
noun, “Sun,” to which it refers; and the noun “core” properly serves as the
object of the preposition “in.”
Explanation for Incorrect Answer E : There is an error in the sentence.
19
The study
showed
that children
who consumed
small amounts of added sugars
on average one serving more of grains per day
amounts of these same sugars.
ANSWERS
eating
than did
children who ate larger
No error
AND EXPLANATIONS
Corrected Sentence: The study showed that children who consumed small
amounts of added sugars ate on average one serving more of grains per day than
did children who ate larger amounts of these same sugars.
!
途
ed
Explanation for Correct Answer C :
The error in this sentence occurs at (C), where an improper verb form is used. The
participle “eating” cannot serve as a main verb for the subject of this subordinate
clause, “children,” and its present tense is inconsistent with the simple past
tense of the other verbs in the sentence (“showed,” “consumed,” “did,”
and “ate”). The past-tense verb “ate” should be used instead.
eg
is
te
r
用
业
商
Explanation for Incorrect Answer A :
There is no error at (A). The past tense of the verb “showed” appropriately
expresses an action completed in the past.
于
Explanation for Incorrect Answer B :
There is no error at (B). The relative pronoun “who” appropriately refers to the
noun “children” and properly introduces the subordinate clause (“who
consumed small amounts of added sugars”) that describes “children.”
nR
用
禁
Explanation for Incorrect Answer D :
There is no error at (D). The conjunction “than” is necessary to complete the
comparison begun with “more.” Also, the past tense of “did” is consistent with
the past tense of the other verbs in the sentence (“showed,” “ consumed,”
and “ate”).
U
严
Explanation for Incorrect Answer E : There is an error in the sentence.
20
In many parts of the world where grasses cover vast expanses of land, periodic,
controlled burning is practiced in order to keep woody brush from gaining a foothold
and it stimulates continuing grass growth. No error
ANSWERS
AND EXPLANATIONS
Corrected Sentence: In many parts of the world where grasses cover vast
expanses of land, periodic, controlled burning is practiced in order to keep woody
brush from gaining a foothold and to stimulate continuing grass growth.
file://E:\新建文件夹\e7.htm
2006-11-12
页码,14/24
The Official SAT Online Course
Explanation for Correct Answer D :
The error in this sentence occurs at (D), where there is an error in parallelism. The
independent clause “it stimulates … growth” is not parallel with the infinitive
phrase “to keep” to which it is connected by the coordinating conjunction
“and.” The infinitive phrase “to stimulate” is needed instead.
Explanation for Incorrect Answer A :
There is no error at (A). The relative adverb “where” correctly introduces the
subordinate clause that modifies the noun “parts.”
Explanation for Incorrect Answer B :
There is no error at (B). The passive verb “is practiced” is appropriately used in
this sentence, and the singular “is” agrees in number with its singular subject,
“burning.”
Explanation for Incorrect Answer C :
There is no error at (C). The preposition “from” is an idiomatic preposition to
follow “keep,” and the gerund “gaining,” which is a noun, properly serves as
the object of that preposition. In addition, the prepositional phrase appropriately
functions as an adverb modifying “keep.”
Explanation for Incorrect Answer E : There is an error in the sentence.
d
The newspaper reported that having the increase in the minimum wage, many people
re
21
!
途
用
No error
te
are still having trouble making ends meet.
业
is
商
AND EXPLANATIONS
eg
ANSWERS
于
Corrected Sentence: The newspaper reported that even with the increase in the
minimum wage, many people are still having trouble making ends meet.
nR
用
Explanation for Correct Answer B :
The error in this sentence occurs at (B), where improper phrasing is used. The
participle “having” introduces a participial phrase that incorrectly modifies the
subject of the sentence, “newspaper.” So that the phrase “the increase in the
minimum wage” describes the intended noun, “people,” the phrase must be
introduced by a preposition, in this case, “with.”
禁
U
严
Explanation for Incorrect Answer A :
There is no error at (A). The past tense of the verb “reported” appropriately
expresses an action that was completed in the past, and the pronoun “that”
properly introduces the subordinate clause “that … meet,” which functions as
the direct object of the independent clause.
Explanation for Incorrect Answer C :
There is no error at (C). The adjective “many” appropriately modifies the noun
“people,” which, in turn, properly functions as the subject of the subordinate
clause introduced by “that.”
Explanation for Incorrect Answer D :
There is no error at (D). The participle “making” properly introduces the
participial phrase “making ends meet” that functions as an adjective describing
the noun “trouble.”
Explanation for Incorrect Answer E : There is an error in the sentence.
22
file://E:\新建文件夹\e7.htm
2006-11-12
页码,15/24
The Official SAT Online Course
Used primarily
in the United States and Canada, American Sign Language,
comprises hand signs, facial expressions, and fingerspelling,
differences and slang.
ANSWERS
which
including many
regional
No error
AND EXPLANATIONS
Corrected Sentence: Used primarily in the United States and Canada, American
Sign Language, which comprises hand signs, facial expressions, and fingerspelling,
includes many regional differences and slang.
Explanation for Correct Answer C :
The error in this sentence occurs at (C), where a participle, “including,” is used
instead of a main verb, “includes,” which is needed to complete the action of the
sentence.
!
途
re
d
Explanation for Incorrect Answer A :
There is no error at (A). The past participle “used” appropriately introduces the
participial phrase that modifies the subject of the independent clause, “American
Sign Language.” The adverb “primarily” properly modifies the participle
“used,” which functions as an adjective.
is
te
用
业
Explanation for Incorrect Answer B :
There is no error at (B). The relative pronoun “which” appropriately refers to the
proper noun “American Sign Language” and introduces the subordinate clause
that describes “American Sign Language.”
商
eg
Explanation for Incorrect Answer D :
There is no error at (D). The plural adjective “many” idiomatically describes the
plural noun “differences.”
于
用
23
nR
Explanation for Incorrect Answer E : There is an error in the sentence.
禁
U
严
The United States is the world’s largest producer of cranberries and cranberry
products, most of which is consumed there and in Canada. No error
ANSWERS
AND EXPLANATIONS
Corrected Sentence: The United States is the world’s largest producer of
cranberries and cranberry products, most of which are consumed there and in
Canada.
Explanation for Correct Answer C :
The error in this sentence occurs at (C), where an incorrect verb is used. The
singular helping verb “is” does not agree in number with the plural subject of the
subordinate clause (“most”) that refers to the compound plural objects of the
preposition, “cranberries and cranberry products.”
Explanation for Incorrect Answer A :
There is no error at (A). The singular noun “producer” appropriately renames the
singular subject of the independent clause, “United States.” The preposition
file://E:\新建文件夹\e7.htm
2006-11-12
页码,16/24
The Official SAT Online Course
“of” is an idiomatic preposition to follow “producer” and to introduce the
prepositional phrase (“of … products”) that functions as an adjective describing
“producer.”
Explanation for Incorrect Answer B :
There is no error at (B). The pronoun “most,” which can be singular or plural,
agrees in number with the compound plural objects of the preposition,
“cranberries and cranberry products,” to which it refers.
Explanation for Incorrect Answer D :
There is no error at (D). The coordinating conjunction “and” appropriately
connects two adverbs, “there,” and the prepositional phrase, “in Canada,” that
modify the verb “consumed.” The preposition “in” is an idiomatic preposition
to follow the verb “consumed.”
Explanation for Incorrect Answer E : There is an error in the sentence.
24
Almost all animals
which they convert
to thick layers of fat.
No error
!
途
ed
amounts of food,
ANSWERS
by eating
it
that hibernate
large
prepare for during summer
AND EXPLANATIONS
用
st
er
Corrected Sentence: Almost all animals that hibernate prepare for hibernation
during summer by eating large amounts of food, which they convert to thick layers
of fat.
业
Explanation for Correct Answer B :
The error in this sentence occurs at (B), where an improper pronoun is used. The
pronoun “it” has no antecedent to which it can logically refer.
eg
i
商
于
用
nR
Explanation for Incorrect Answer A :
There is no error at (A). The relative pronoun “that” correctly refers to
“animals” and properly introduces the subordinate clause that describes
“animals.” The plural verb of this subordinate clause, “hibernate,” agrees in
number with the pronoun “that,” which refers to the plural noun “animals.”
禁
U
严
Explanation for Incorrect Answer C :
There is no error at (C). The preposition “by” is the idiomatic preposition to
follow the word “prepare,” and the gerund “eating” properly serves as the
object of that preposition. The prepositional phrase appropriately serves as an
adverb modifying the verb “prepare.”
Explanation for Incorrect Answer D :
There is no error at (D). The relative pronoun “which” introduces the clause
(“which they convert”) that modifies the noun “food.” The plural pronoun
“they” agrees in number with the plural noun to which it refers, “animals,”
and the plural verb “convert” agrees in number with its plural subject, “they.”
Explanation for Incorrect Answer E : There is an error in the sentence.
25
Among birds that forage in tree trunks, nuthatches are the only ones that
regularly feed with their heads facing downward, when they often find food in bark
crevices overlooked in their upward search. No error
file://E:\新建文件夹\e7.htm
2006-11-12
页码,17/24
The Official SAT Online Course
ANSWERS
AND EXPLANATIONS
Corrected Sentence: Among birds that forage in tree trunks, nuthatches are the
only ones that regularly feed with their heads facing downward, often finding food
in bark crevices overlooked in their upward search.
Explanation for Correct Answer C :
The error in this sentence occurs at (C), where a subordinate clause creates faulty
logic. The use of “when,” an adverb that modifies the verb “feed,” makes the
sentence illogically suggest that nuthatches regularly feed “when they often find
food.”
Explanation for Incorrect Answer A :
There is no error at (A). The relative pronoun “that” correctly refers to “birds”
and properly introduces the subordinate clause that describes “birds.” The plural
verb of this subordinate clause, “forage,” agrees in number with the pronoun
“that,” which refers to the plural noun “birds.”
!
途
d
Explanation for Incorrect Answer B :
There is no error at (B). The adverb “regularly” properly modifies the plural verb
“feed,” and the plural verb agrees in number with the relative pronoun “that,”
which refers to the plural noun “ones.”
re
Explanation for Incorrect Answer D :
There is no error at (D). The past participle “overlooked” properly functions as an
adjective describing the noun “food,” and the preposition “in” appropriately
introduces the prepositional phrase (“in … search”) that serves as an adverb
modifying the participle “overlooked.”
is
te
用
业
Explanation for Incorrect Answer E : There is an error in the sentence.
eg
商
26
于
If you can acquire the necessary calories by drinking gasoline instead of by eating
nR
用
food, you would be able to run 26 miles on about one-twelfth of a gallon of gas.
禁
No error
U
严
ANSWERS
AND EXPLANATIONS
Corrected Sentence: If you could acquire the necessary calories by drinking
gasoline instead of by eating food, you would be able to run 26 miles on about onetwelfth of a gallon of gas.
Explanation for Correct Answer A :
The error in this sentence occurs at (A), where an incorrect verb tense is used. The
present tense of the verb “can acquire” in the subordinate clause is inconsistent
with the past tense of the verb in the independent clause, “would be able.”
Explanation for Incorrect Answer B :
There is no error at (B). The preposition “by” is the idiomatic preposition to
follow “acquire,” and the gerund “drinking,” which is a noun, appropriately
serves as the object of that preposition. The prepositional phrase appropriately
functions as an adverb modifying the verb “acquire.”
Explanation for Incorrect Answer C :
There is no error at (C). The second-person pronoun “you” properly serves as
file://E:\新建文件夹\e7.htm
2006-11-12
页码,18/24
The Official SAT Online Course
the subject of the independent clause and is consistent with the second-person
pronoun “you” in the introductory subordinate clause (“If … food”).
Explanation for Incorrect Answer D :
There is no error at (D). The infinitive “to run” is the idiomatic verb form to follow
the verb “able.”
Explanation for Incorrect Answer E : There is an error in the sentence.
27
In many large cities in the United States, the presence of a
population
has led
ANSWERS
wholly or partly
that
in languages
curricula taught
to repeated calls
other than English.
culturally
diverse
No error
!
途
AND EXPLANATIONS
eg
is
te
re
d
Corrected Sentence: In many large cities in the United States, the presence of a
culturally diverse population has led to repeated calls for curricula taught wholly or
partly in languages other than English.
用
Explanation for Correct Answer C :
The error in this sentence occurs at (C), where the relative pronoun (“that”) is
not idiomatic and is incorrectly used to introduce a prepositional phrase. The
preposition “for” is needed instead (“calls for curricula…”).
业
商
Explanation for Incorrect Answer A :
There is no error at (A). The adverb “culturally” appropriately modifies the
adjective “diverse,” which, in turn, describes the noun “population.”
于
用
nR
Explanation for Incorrect Answer B :
There is no error at (B). The singular helping verb “has” agrees in number with
the subject of the independent clause, “presence,” and the perfect tense (“has
led”) is appropriate for expressing an action completed in the present time.
禁
Explanation for Incorrect Answer D :
There is no error at (D). The compound adverb phrase “wholly or partly”
appropriately modifies the past participle “taught,” which functions as an
adjective describing the noun “curricula.”
U
严
Explanation for Incorrect Answer E : There is an error in the sentence.
28
Of ancient origin, the game of checkers was played in Egypt during the time of the
pharaohs and is mentioned in the writings of Homer and Plato. No error
ANSWERS
AND EXPLANATIONS
Corrected Sentence:
Explanation for Correct Answer E : There is no error in this sentence.
file://E:\新建文件夹\e7.htm
2006-11-12
页码,19/24
The Official SAT Online Course
Explanation for Incorrect Answer A :
There is no error at (A). The preposition “of” is used properly to introduce the
prepositional phrase, “Of ancient origin,” that modifies the subject of the
sentence, “game.”
Explanation for Incorrect Answer B :
There is no error at (B). The singular helping verb “was” agrees in number with
the singular subject of the sentence, “game,” and the past participle “played”
is the appropriate verb form to create the present tense of a passive verb (“was
played”).
Explanation for Incorrect Answer C :
There is no error at (C). The preposition “during” combines with “the time” to
produce an idiomatic phrase that indicates when checkers was played in Egypt.
Explanation for Incorrect Answer D :
There is no error at (D). The singular helping verb “is” agrees in number with the
singular subject “game,” and the past participle “mentioned” is the
appropriate verb form to create the present tense of a passive verb (“is
mentioned”).
By
the early 1920's, Louis Armstrong recorded
!
途
his first
solos as a member of King
te
re
d
29
which
such pieces as
“Chimes Blues” and “Tears,”
Oliver’s Creole Jazz Band in
is
业
商
AND EXPLANATIONS
eg
ANSWERS
用
No error
he composed with pianist Lil Hardin.
于
Corrected Sentence: In the early 1920’s, Louis Armstrong recorded his first
solos as a member of King Oliver’s Creole Jazz Band in such pieces as “Chimes
Blues” and “Tears, ” which he composed with pianist Lil Hardin.
nR
用
Explanation for Correct Answer A :
The error in this sentence occurs at (A), where an inappropriate preposition
(“By”) is used. In the context of this sentence, the preposition “In” is the
appropriate preposition to introduce the prepositional phrase that functions as an
adverb modifying the verb “recorded.”
禁
U
严
Explanation for Incorrect Answer B :
There is no error at (B). The singular possessive pronoun “his” agrees in number
and gender with the singular noun “Louis Armstrong” to which it refers, and it
appropriately combines with the adjective “first” to describe the plural noun
“solos.”
Explanation for Incorrect Answer C :
There is no error at (C). The adjective “such” appropriately modifies the noun
“pieces,” which properly functions as the object of the preposition “in,” and
the preposition “as” appropriately introduces the prepositional phrase that
functions as an adjective modifying the noun “pieces.”
Explanation for Incorrect Answer D :
There is no error at (D). The relative pronoun “which” is used appropriately to
refer to the pieces that Armstrong composed with pianist Lil Hardin.
Explanation for Incorrect Answer E : There is an error in the sentence.
(1) The early history of astronomy was full of misunderstandings. (2) Some of
file://E:\新建文件夹\e7.htm
2006-11-12
页码,20/24
The Official SAT Online Course
them were funny, it's like the controversy of the "canali" on Mars. (3) In the
late 1800’s an Italian astronomer named Giovanni Schiaparelli studied
Mars. (4) He had a high-powered telescope that he used to look at
Mars. (5) Schiaparelli thought he saw channels criss-crossing the planet's
surface. (6) He was intrigued: perhaps these channels were evidence that Mars
had great flowing rivers like the Earth. (7) Schiaparelli made charts of the
surface of Mars and labeled it with the Italian word "canali."
(8) Unfortunately, "canali" can be translated into English as either "channels" or
"canals." (9) Channels and canals are two different things because channels are
formed naturally by water, while canals are constructed by people. (10) Some
people translated "canali" as "canals," word began to spread that the lines
Schiaparelli saw through his telescope were actually canals that had been built
by intelligent beings. (11) One of them was an amateur astronomer named
Percival Lowell. (12) He wrote a series of best-selling books. (13) In these
books Lowell publicized the notion that these "canals" were built by Martian
farmers who understood irrigation.
(14) In 1965 a U.S. spacecraft flying close to the surface of Mars sent back
conclusive pictures. (15) There are no prominent channels anywhere on the
planet. (16) Lowell and Schiaparelli saw what they wanted to see. (17) Lowell
was wrong, of course, but so was Schiaparelli.
30
Which is the best version of the underlined portion of sentence 2 (reproduced below)?
!
途
(as it is now)
(B)
funny; it's like
er
e
(A)
d
Some of them were funny, it's like the controversy of the "canali" on Mars.
用
(C) funny, like
(E)
as funny as
商
AND EXPLANATIONS
eg
i
ANSWERS
业
st
(D) funny, even
于
Explanation for Correct Answer C :
Choice (C) is correct. The sentence properly introduces the controversy that is the
subject of the passage as one of a number of funny misunderstandings.
nR
用
禁
Explanation for Incorrect Answer A :
Choice (A) is unsatisfactory because it joins two independent thoughts with only a
comma.
U
严
Explanation for Incorrect Answer B :
Choice (B) is unsatisfactory because it is illogical; "it's" does not make sense in this
context.
Explanation for Incorrect Answer D :
Choice (D) is unsatisfactory because "even" suggests that others do not find the
"canali" controversy funny. The passage does not indicate this.
Explanation for Incorrect Answer E :
Choice (E) is unsatisfactory because "as funny as" puts the emphasis of the
sentence on the other misunderstandings, with the expectation that they will be
compared to the "canali" controversy. Such a comparison does not happen in the
passage.
31
Which is the best way to combine sentences 3 and 4 (reproduced below)?
In the late 1800’s an Italian astronomer named Giovanni Schiaparelli studied Mars.
He had a high-powered telescope that he used to look at Mars.
In the late 1800’s an Italian astronomer named Giovanni Schiaparelli
file://E:\新建文件夹\e7.htm
2006-11-12
页码,21/24
The Official SAT Online Course
(A)
studied Mars by a high-powered telescope.
In the late 1800’s an Italian astronomer named Giovanni Schiaparelli
studied Mars with a high-powered telescope that he used to look at Mars.
(C) In the late 1800’s an Italian astronomer named Giovanni Schiaparelli
studied Mars, he had a high-powered telescope that he used.
(B)
(D) In the late 1800’s an Italian astronomer named Giovanni Schiaparelli
used a high-powered telescope to study Mars.
(E)
In the late 1800’s an Italian astronomer named Giovanni Schiaparelli,
using a high-powered telescope to look, studied Mars.
ANSWERS
AND EXPLANATIONS
Explanation for Correct Answer D :
Choice (D) is correct. The sentence nicely joins the ideas of sentences 3 and 4
without repeating information unnecessarily.
Explanation for Incorrect Answer A :
Choice (A) is unsatisfactory because it is unidiomatic to write that Schiaparelli
"studied Mars by" a telescope; "with" or "through" would be more appropriate.
!
途
te
re
d
Explanation for Incorrect Answer B :
Choice (B) is unsatisfactory because it repeats information about studying/looking
at Mars unnecessarily.
Explanation for Incorrect Answer C :
Choice (C) is unsatisfactory because it joins two complete thoughts with only a
comma.
用
业
Explanation for Incorrect Answer E :
Choice (E) is unsatisfactory because it unnecessarily separates the act of looking at
Mars from the act of studying Mars. As the passage indicates, the activities are the
same.
is
商
eg
于
32
用
nR
Which word would be best to insert at the beginning of sentence 10 (reproduced
below)?
禁
Some people translated "canali" as "canals," word began to spread that the lines
Schiaparelli saw through his telescope were actually canals that had been built by
intelligent beings.
U
严
(A)
Whereas
(B)
However
(C) If
(D) Although
(E)
Because
ANSWERS
AND EXPLANATIONS
Explanation for Correct Answer E :
Choice (E) is correct. The word "Because" appropriately signifies the relationship
between the translation problem and the misunderstanding about the "canals."
Explanation for Incorrect Answer A :
Choice (A) is unsatisfactory because "Whereas" indicates that the mistranslation of
"canali" and the misunderstanding about the building of the "canals" were
contradictory ideas, when in fact one caused the other.
Explanation for Incorrect Answer B :
Choice (B) is unsatisfactory because "However" makes no sense in this context.
file://E:\新建文件夹\e7.htm
2006-11-12
页码,22/24
The Official SAT Online Course
Explanation for Incorrect Answer C :
Choice (C) is unsatisfactory because "If" suggests that there is some doubt as to
whether or not people mistranslated "canali" in this way. The implication of the
passage is that people did mistranslate the term, resulting in a misconception about
the building of the "canals."
Explanation for Incorrect Answer D :
Choice (D) is unsatisfactory because "Although" implies that one would not expect
word to spread given that the mistranslation occurred, when the passage indicates
the opposite.
33
What is the best version of the underlined portion of sentence 11 (reproduced
below)?
One of them was an amateur astronomer named Percival Lowell.
(A)
(As it is now)
(B)
One of the most intelligent was
(C) This idea was popularized by
(D) It was spread even more by someone else,
!
途
The person who solved the problem was
AND EXPLANATIONS
re
ANSWERS
d
(E)
用
Explanation for Correct Answer C :
Choice (C) is correct. It properly signifies the relationship between the idea
presented in the previous sentence and the astronomer Lowell.
te
业
is
商
Explanation for Incorrect Answer A :
Choice (A) is unsatisfactory because the pronoun "them" seems to refer to
"intelligent beings" who built canals on Mars, and Lowell was clearly not one of
these.
eg
于
用
nR
Explanation for Incorrect Answer B :
Choice (B) is unsatisfactory because Lowell is not described elsewhere in the
passage as intelligent; in fact, his theory is shown to be based on a simple
misunderstanding.
禁
Explanation for Incorrect Answer D :
Choice (D) is unsatisfactory because it results in an awkward and illogical sentence.
U
严
Explanation for Incorrect Answer E :
Choice (E) is unsatisfactory because, according to the passage, Lowell created more
problems than he solved.
34
What is the best way to combine sentences 12 and 13 (reproduced below)?
He wrote a series of bestselling books. In these books Lowell publicized the notion
that these "canals" were built by Martian farmers who understood irrigation.
(A)
In a series of bestselling books, Lowell publicized the notion that these
"canals" were built by Martian farmers who understood irrigation.
(B)
He wrote a series of books that was a bestseller and publicized the notion
that these "canals" were built by Martian farmers who understood
irrigation.
(C) His books that were bestsellers publicized the notion that these "canals"
were built by Martian farmers who understood irrigation.
(D) He wrote a series of bestselling books; Lowell publicized the notion that
these "canals" were built by Martian farmers who understood irrigation.
(E)
file://E:\新建文件夹\e7.htm
In these books, which were bestsellers, Lowell publicized the notion that
these "canals" were built by Martian farmers who understood irrigation.
2006-11-12
页码,23/24
The Official SAT Online Course
ANSWERS
AND EXPLANATIONS
Explanation for Correct Answer A :
Choice (A) is correct. The resulting sentence maintains the effective structure of the
original sentence 13 while adding the important information from sentence 12.
Explanation for Incorrect Answer B :
Choice (B) is unsatisfactory because it is improper to describe a series of books as
"a bestseller."
Explanation for Incorrect Answer C :
Choice (C) is unsatisfactory because, following sentence 11, it is more appropriate
for the subject of the sentence to be "Lowell" than to be "his books."
Explanation for Incorrect Answer D :
Choice (D) is unsatisfactory because it does not indicate the relationship between
the books Lowell wrote and his popularization of the "intelligent beings" idea.
Explanation for Incorrect Answer E :
Choice (E) is unsatisfactory because it suggests that Lowell's books have been
previously mentioned in the passage ("In these books"), when this is the first
mention of them.
ed
用
What would best replace "it" in sentence 15 (reproduced below)?
te
r
35
!
途
According to these images, there are no prominent channels anywhere on it.
the planet
(B)
the spacecraft
(C) the pictures
商
eg
于
(D) these
(E)
业
is
(A)
them
nR
用
ANSWERS
禁
AND EXPLANATIONS
Explanation for Correct Answer A :
Choice (A) is correct. It corrects the error of the original by clearly specifying the
object on which there are no prominent channels: the planet Mars.
U
严
Explanation for Incorrect Answer B :
Choice (B) is unsatisfactory because, in the context of the passage, it makes no
sense to discuss "channels" on a spacecraft.
Explanation for Incorrect Answer C :
Choice (C) is unsatisfactory because the "images" mentioned earlier in the sentence
are the "pictures" taken by the spacecraft. There is no need to refer to them twice.
Explanation for Incorrect Answer D :
Choice (D) is unsatisfactory because the word “these” does not logically refer to
anything before or after sentence 15.
Explanation for Incorrect Answer E :
Choice (E) is unsatisfactory because it is unclear to what "them" refers.
file://E:\新建文件夹\e7.htm
2006-11-12
页码,24/24
The Official SAT Online Course
Back to Score Report
Privacy Policy
Copyright © 2006 The College Board. All rights reserved.
Terms of Use
Contact Us
ed
!
途
er
用
st
业
eg
i
商
于
nR
用
禁
U
严
file://E:\新建文件夹\e7.htm
2006-11-12
页码,1/18
The Official SAT Online Course
Help | Profile | My Organizer | My Bookmarks | Logout
Answers and Explanations
Back to Score Report
Test Sections
Section 1
View Answers and Explanations
Section 2
Online - Practice Test #5
Section 3
Section 4
1
Section 5
Section 6
The anthropology professor hoped that his latest book would appeal to popular as
well as to ------- readers, thereby earning him ------- in both realms.
(A) general. . disdain
Section 8
(B)
Section 9
(C) academic. . anonymity
Section 10
lay. . attention
(D) avid. . remuneration
scholarly. . acclaim
ANSWERS
!
途
AND EXPLANATIONS
ed
(E)
Explanation for Correct Answer E :
Choice (E) is correct. “Scholarly” means learned or academic. “Acclaim” is
praise or applause. If one were to insert this term into the text, the sentence would
read “The anthropology professor hoped that his latest book would appeal to
popular as well as to scholarly readers, thereby earning him acclaim in both
realms.” The phrases “as well as” and “both realms” indicate that the
professor hoped his book would appeal to two separate groups of readers. It makes
sense to say that the professor hoped to earn “acclaim,” or praise, from both
popular readers and “scholarly,” or academic, readers.
is
te
r
用
业
eg
商
于
Explanation for Incorrect Answer A :
Choice (A) is incorrect. “General,” in this context, means not confined by
specialization. “Disdain” is a feeling of contempt or scorn. If one were to insert
these terms into the text, the sentence would read “The anthropology professor
hoped that his latest book would appeal to popular as well as to general readers,
thereby earning him disdain in both realms.” The phrases “as well as” and
“both realms” indicate that the professor hoped his book would appeal to two
separate groups of readers. The term “general” does not make logical sense in
this context because there is not a clear distinction between the terms “general”
and “popular” as they refer to similar groups of people. Additionally, the
anthropology professor wanted his book to appeal to readers, so it is illogical to
suggest that the anthropology professor hoped to earn “disdain,” or contempt.
nR
用
禁
U
严
Explanation for Incorrect Answer B :
Choice (B) is incorrect. “Lay” means not associated with a profession or any
particular knowledge. “Attention” is observation or notice. If one were to insert
these terms into the text, the sentence would read “The anthropology professor
hoped that his latest book would appeal to popular as well as to lay readers,
thereby earning him attention in both realms.” The phrases “as well as” and
“both realms” indicate that the professor hoped his book would appeal to two
separate groups of readers. The term “lay” does not make logical sense in this
context because the terms “lay” and “popular” both refer to readers from one
group, the general public.
Explanation for Incorrect Answer C :
Choice (C) is incorrect. “Academic” means scholarly or related to higher learning.
“Anonymity” is a state of being unknown or unidentified. If one were to insert
these terms into the text, the sentence would read “The anthropology professor
hoped that his latest book would appeal to popular as well as to academic readers,
thereby earning him anonymity in both realms.” The professor likely hoped his
book would appeal to readers in both the “popular” realm and the “academic,”
or scholarly, realm, but it is illogical to suggest that the professor hoped to earn
“anonymity.” Anonymity, or a state of being unknown, is not usually described
as something one earns, and there is no reason to believe that the professor would
not want to be known for his work.
file://E:\新建文件夹\e8.htm
2006-11-12
页码,2/18
The Official SAT Online Course
Explanation for Incorrect Answer D :
Choice (D) is incorrect. “Avid” means enthusiastic and vigorous.
“Remuneration” is payment for a service. If one were to insert these terms into
the text, the sentence would read “The anthropology professor hoped that his
latest book would appeal to popular as well as to avid readers, thereby earning him
remuneration in both realms.” The phrases “as well as” and “both realms”
indicate that the professor hoped his book would appeal to two separate groups of
readers. The terms “avid” and “popular” do not necessarily describe separate
groups. Popular readers, or readers from the general public, can also be “avid,”
or enthusiastic, readers.
2
As if intended to squelch rumors of cutbacks, the company’s annual celebration was
as ------- as ever.
(A) sparing
(B)
tawdry
(C) belated
(D) lavish
(E)
sated
!
途
AND EXPLANATIONS
d
ANSWERS
re
Explanation for Correct Answer D :
Choice (D) is correct. “Lavish” means extravagant and elaborate. If one were to
insert this term into the text, the sentence would read “As if intended to squelch
rumors of cutbacks, the company’s annual celebration was as lavish as ever.”
The word “squelch” indicates that the missing term will describe a celebration
that might suppress rumors of cutbacks, or reductions in spending. A “lavish,” or
extravagant, celebration could certainly suppress such rumors.
is
te
用
业
商
eg
Explanation for Incorrect Answer A :
Choice (A) is incorrect. “Sparing” means restrained, modest, or thrifty. If one
were to insert this term into the text, the sentence would read “As if intended to
squelch rumors of cutbacks, the company’s annual celebration was as sparing as
ever.” The word “squelch” indicates that the missing term will describe a
celebration that might suppress rumors of cutbacks, or reductions in spending. A
"sparing," or thrifty, celebration would more likely encourage rumors of cutbacks
than squelch them.
于
nR
用
禁
Explanation for Incorrect Answer B :
Choice (B) is incorrect. “Tawdry” means cheap. If one were to insert this term
into the text, the sentence would read “As if intended to squelch rumors of
cutbacks, the company’s annual celebration was as tawdry as ever.” A
“tawdry,” or cheap, celebration would most likely strengthen rumors of cutbacks
rather than squelch, or suppress, them.
U
严
Explanation for Incorrect Answer C :
Choice (C) is incorrect. “Belated” means delayed or late. If one were to insert
this term into the text, the sentence would read “As if intended to squelch rumors
of cutbacks, the company’s annual celebration was as belated as ever.” The
word "squelch" indicates that the missing term will describe a celebration that
might suppress rumors of cutbacks, or reductions in spending. Although a
celebration could be “belated,” it is not logical to suggest that a late celebration
might suppress such rumors.
Explanation for Incorrect Answer E :
Choice (E) is incorrect. “Sated” means satisfied almost to the point of excess. If
one were to insert this term into the text, the sentence would read “As if intended
to squelch rumors of cutbacks, the company’s annual celebration was as sated as
ever.” While a person can be described as “sated,” it does not make logical
sense to describe a celebration as “sated,” or satisfied.
3
file://E:\新建文件夹\e8.htm
The professor’s ------- lecture on American history served more to confuse the
students than to clarify the difficult material.
2006-11-12
页码,3/18
The Official SAT Online Course
(A)
perspicacious
(B)
exhaustive
(C) cogent
(D) scintillating
(E)
disjointed
ANSWERS
AND EXPLANATIONS
Explanation for Correct Answer E :
Choice (E) is correct. “Disjointed” means out of order and incoherent. If one
were to insert this term into the text, the sentence would read “The professor’s
disjointed lecture on American history served more to confuse the students than to
clarify the difficult material.” A “disjointed,” or incoherent, lecture would most
likely confuse the students more than it would help them to understand the difficult
material.
Explanation for Incorrect Answer A :
Choice (A) is incorrect. “Perspicacious” means keen or having acute mental
abilities. If one were to insert this term into the text, the sentence would read
“The professor’s perspicacious lecture on American history served more to
confuse the students than to clarify the difficult material.” It is unlikely that a
lecture could be both “perspicacious” and confusing to the students.
re
d
!
途
用
eg
is
te
Explanation for Incorrect Answer B :
Choice (B) is incorrect. “Exhaustive” means thorough. If one were to insert this
term into the text, the sentence would read “The professor’s exhaustive lecture
on American history served more to confuse the students than to clarify the difficult
material.” There is no reason to believe that an “exhaustive,” or thorough,
lecture would confuse the students more than it would help them to understand the
difficult material. On the contrary, an exhaustive lecture would be likely to clarify
the material.
业
商
Explanation for Incorrect Answer C :
Choice (C) is incorrect. “Cogent” means relevant. If one were to insert this term
into the text, the sentence would read “The professor’s cogent lecture on
American history served more to confuse the students than to clarify the difficult
material.” There is no reason to believe that a “cogent,” or relevant, lecture
would confuse the students more than it would help them to understand the difficult
material.
于
nR
用
禁
Explanation for Incorrect Answer D :
Choice (D) is incorrect. “Scintillating” means lively and witty. If one were to
insert this term into the text, the sentence would read “The professor’s
scintillating lecture on American history served more to confuse the students than
to clarify the difficult material.” There is no reason to believe that a
“scintillating,” or lively, lecture would confuse the students more than it would
help them to understand the difficult material.
U
严
4
Since the foreign correspondent was accustomed to completing his assignments
under ------- conditions, the commotion of the subway at rush hour presented no
------- his creativity.
(A)
squalid . . boon for
(B)
tranquil . . obstruction to
(C) tumultuous . . impediment to
(D) destructive . . demonstration of
(E)
flagrant . . benefit to
ANSWERS
AND EXPLANATIONS
Explanation for Correct Answer C :
Choice (C) is correct. “Tumultuous” means chaotic and disorderly. An
“impediment” means something that slows down or gets in the way of something
file://E:\新建文件夹\e8.htm
2006-11-12
页码,4/18
The Official SAT Online Course
else. If one were to insert these terms into the text, the sentence would read
“Since the foreign correspondent was accustomed to completing his assignments
under tumultuous conditions, the commotion of the subway at rush hour presented
no impediment to his creativity.” The term “Since” indicates that there is a
relationship between the conditions to which the correspondent was accustomed
and his ability to work during the “commotion,” or noisy confusion, of the subway
at rush hour. The noisy confusion of the subway was similar to the “tumultuous,”
or chaotic and disorderly, conditions the correspondent was used to, so the
commotion was not an “impediment” to his creativity.
Explanation for Incorrect Answer A :
Choice (A) is incorrect. “Squalid” means dirty and neglected. A “boon” is a
benefit or favor. If one were to insert these terms into the text, the sentence would
read “Since the foreign correspondent was accustomed to completing his
assignments under squalid conditions, the commotion of the subway at rush hour
presented no boon for his creativity.” The term “Since” indicates that there is a
relationship between the conditions to which the correspondent was accustomed
and his ability to work during the “commotion,” or noisy confusion, of the subway
at rush hour. Because there is not necessarily any connection between “squalid,”
or dirty and neglected, conditions and noisily confusing conditions, there is no
reason to believe that the commotion of the subway would or would not have been
a “boon,” or benefit, to the correspondent’s creativity.
!
途
te
re
d
Explanation for Incorrect Answer B :
Choice (B) is incorrect. “Tranquil” means calm and free from disturbance. An
“obstruction” is something that blocks or gets in the way of something else. If
one were to insert these terms into the text, the sentence would read “Since the
foreign correspondent was accustomed to completing his assignments under
tranquil conditions, the commotion of the subway at rush hour presented no
obstruction to his creativity.” A correspondent who is used to working under
“tranquil,” or calm, conditions would likely find the “commotion,” or noisy
confusion, of the subway to be an “obstruction” to his creativity. Therefore, it is
illogical to suggest that the commotion presented “no obstruction to,” or did not
get in the way of, his creativity.
用
is
业
商
eg
Explanation for Incorrect Answer D :
Choice (D) is incorrect. “Destructive” means causing harm and destroying. A
“demonstration” is a public display. If one were to insert these terms into the
text, the sentence would read “Since the foreign correspondent was accustomed to
completing his assignments under destructive conditions, the commotion of the
subway at rush hour presented no demonstration of his creativity.” Although the
first term makes sense in this context, the second term does not. While the
“commotion,” or noisy confusion, of the subway could affect the correspondent’
s ability to be creative, it is illogical to suggest that a commotion could
“demonstrate” his creativity.
于
nR
用
禁
Explanation for Incorrect Answer E :
Choice (E) is incorrect. “Flagrant” means obviously offensive. A “benefit”
means an advantage or something that is useful. If one were to insert these terms
into the text, the sentence would read “Since the foreign correspondent was
accustomed to completing his assignments under flagrant conditions, the
commotion of the subway at rush hour presented no benefit to his creativity.”
Although the second term makes sense in this context, the first does not. It is
illogical to describe a condition as “flagrant,” or obviously offensive.
U
严
5
Unlike the ------- presentations of the other students in class, Mary Catherine’s
presentation demonstrated a thorough and mature grasp of the material.
(A)
astute
(B)
sophomoric
(C) incredulous
(D) cloying
(E)
scintillating
ANSWERS
AND EXPLANATIONS
Explanation for Correct Answer B :
Choice (B) is correct. “Sophomoric” means very immature and poorly informed.
file://E:\新建文件夹\e8.htm
2006-11-12
页码,5/18
The Official SAT Online Course
If one were to insert this term into the text, the sentence would read “Unlike the
sophomoric presentations of the other students in class, Mary Catherine’s
presentation demonstrated a thorough and mature grasp of the material.” The
term “Unlike” indicates that the presentations of the other students differed from
Mary Catherine’s in that they did not demonstrate a thorough, or complete, and
mature grasp of the material. Presentations that did not demonstrate a mature and
complete grasp of the material would most likely be described as “sophomoric,”
or immature and poorly informed.
Explanation for Incorrect Answer A :
Choice (A) is incorrect. “Astute” means shrewd or clever. If one were to insert
this term into the text, the sentence would read “Unlike the astute presentations
of the other students in class, Mary Catherine’s presentation demonstrated a
thorough and mature grasp of the material.” The term “Unlike” indicates that
the presentations of the other students differed from Mary Catherine’s in that they
did not demonstrate a thorough and mature grasp of the material. It is unlikely that
presentations that did not demonstrate a thorough, or complete, grasp of the
material would be described as “astute,” or shrewd and clever.
Explanation for Incorrect Answer C :
Choice (C) is incorrect. “Incredulous” means skeptical or expressing disbelief. If
one were to insert this term into the text, the sentence would read “Unlike the
incredulous presentations of the other students in class, Mary Catherine’s
presentation demonstrated a thorough and mature grasp of the material.”
Although the students could be “incredulous” themselves, it is illogical to say that
the presentations were “incredulous” or skeptical.
d
!
途
re
Explanation for Incorrect Answer D :
Choice (D) is incorrect. “Cloying” means excessively sweet and sentimental. If
one were to insert this term into the text, the sentence would read “Unlike the
cloying presentations of the other students in class, Mary Catherine’s presentation
demonstrated a thorough and mature grasp of the material.” The term “Unlike”
indicates that the presentations of the other students differed from Mary
Catherine’s in that they did not demonstrate a mature and thorough grasp of the
material. The term “cloying” is somewhat illogical in this context, because an
excessively sweet and sentimental presentation could still demonstrate a mature
and complete understanding of the material.
is
te
用
业
eg
商
于
Explanation for Incorrect Answer E :
Choice (E) is incorrect. “Scintillating” means brilliantly lively and witty. If one
were to insert this term into the text, the sentence would read “Unlike the
sophomoric presentations of the other students in class, Mary Catherine’s
presentation demonstrated a thorough and mature grasp of the material.” The
term “Unlike” indicates that the presentations of the other students differed from
Mary Catherine’s in that they did not demonstrate a mature and thorough grasp of
the material. The term “scintillating” is somewhat illogical in this context,
because a brilliantly lively presentation could certainly demonstrate a mature and
complete understanding of the material.
nR
用
禁
U
严
6
Sadly, the author never ------- the rewards of literary success during her lifetime;
public recognition and appreciation of her talent were completely -------.
(A)
predicted . . conclusive
(B)
reaped . . posthumous
(C) acknowledged . . fulsome
(D) appreciated . . gratuitous
(E)
pursued . . discredited
ANSWERS
AND EXPLANATIONS
Explanation for Correct Answer B :
Choice (B) is correct. “To reap” means to obtain. “Posthumous” means
following or occurring after death. If one were to insert these terms into the text,
the sentence would read “Sadly, the author never reaped the rewards of literary
success during her lifetime; public recognition and appreciation of her talent were
completely posthumous.” The semicolon indicates that the idea presented in the
second part of the sentence will provide evidence in support of the idea presented
in the first part of the sentence. If public recognition and appreciation of the
file://E:\新建文件夹\e8.htm
2006-11-12
页码,6/18
The Official SAT Online Course
author’s talent were completely “posthumous,” or following her death, it makes
sense to say that the author never “reaped,” or obtained, the rewards of literary
success during her lifetime.
Explanation for Incorrect Answer A :
Choice (A) is incorrect. “To predict” means to foresee something. “Conclusive”
means related to an outcome or conclusion. If one were to insert these terms into
the text, the sentence would read “Sadly, the author never predicted the rewards
of literary success during her lifetime; public recognition and appreciation of her
talent were completely conclusive.” Although it is plausible to suggest that the
author never “predicted,” or foresaw, that she would achieve literary success in
her lifetime, it does not make sense to say that the recognition and appreciation she
received were “conclusive,” or related to a conclusion.
Explanation for Incorrect Answer C :
Choice (C) is incorrect. “To acknowledge” means to take notice of something.
“Affected” means a quality of behavior that is not natural to oneself. If one were
to insert these terms into the text, the sentence would read “Sadly, the author
never acknowledged the rewards of literary success during her lifetime; public
recognition and appreciation of her talent were completely affected.” The
semicolon indicates that the idea presented in the second part of the sentence will
provide evidence in support of the idea presented in the first part of the sentence. It
is reasonable to suggest that the author never “acknowledged,” or took notice of,
the rewards of literacy success, but this idea is not logically connected to the idea
that the recognition she received was unnatural to those who recognized her talent.
d
!
途
te
re
Explanation for Incorrect Answer D :
Choice (D) is incorrect. “To appreciate” means to recognize with gratitude.
“Gratuitous” means unearned or unwarranted. If one were to insert these terms
into the text, the sentence would read “Sadly, the author never appreciated the
rewards of literary success during her lifetime; public recognition and appreciation
of her talent were completely gratuitous.” The semicolon indicates that the idea
presented in the second part of the sentence will provide evidence in support of the
idea presented in the first part of the sentence. It is reasonable to suggest that the
author never “appreciated,” or recognized with gratitude, the rewards of literary
success and that the recognition and appreciation she received were “gratuitous,”
or unearned, but there is no logical connection between these ideas.
用
业
eg
is
商
于
Explanation for Incorrect Answer E :
Choice (E) is incorrect. “To pursue” means to seek. “Discredited” means not
accepted as true or accurate. If one were to insert these terms into the text, the
sentence would read “Sadly, the author never pursued the rewards of literary
success during her lifetime; public recognition and appreciation of her talent were
completely discredited.” The semicolon indicates that the idea presented in the
second part of the sentence will provide evidence in support of the idea presented in
the first part of the sentence. Although it is plausible to suggest that the author
never “pursued,” or sought, the rewards of literary success, this idea is not
logically connected to the idea that the public recognition and appreciation she
received was “discredited.” Additionally, it is illogical to suggest that recognition
and appreciation of an author’s talent would not be accepted as true.
nR
用
禁
U
严
These two passages discuss the same scholarly book, Strangers from a Different Shore
by Asian American historian Ronald Takaki. The first passage is a critique of the book
written by another Asian American historian, Li Ling-chi Wang; the second is a defense
of the book by its author.
Passage 1
In terms of its approach, Takaki’s
book is similar to Victor and Bret Nee’s
Longtime Californ’ (1972) because both
Line are media through which diverse Asian
5
American voices surface. The major
difference is in how they collected and use
the voices. The Nees identified and
file://E:\新建文件夹\e8.htm
2006-11-12
页码,7/18
The Official SAT Online Course
10
15
20
!
途
35
用
业
商
于
eg
30
is
te
re
d
25
selected representative personalities from
different segments of the Chinese
American community in San Francisco,
whom they laboriously interviewed
themselves over a two-year period, and
placed these lengthy, in-depth interviews
largely verbatim in their book with
minimal narrative and analysis. From the
analytical standpoint, the book broke no
new ground. But it succeeded in giving us
the sights, sounds, flavors, perspectives,
and feelings of the community that had
never before been permitted to surface.
For their labor, the book received lavish
reviews by the mainstream press.
Takaki’s approach is virtually
identical with that of the Nees except the
voices used tend to be fragmentary and
mostly collected from indirect sources. To
begin with, no attempt was made to
distinguish voices and quotes from literary
works from voices of oral histories,
documents, and newspapers. Unlike in
the Nees’ book, only a very tiny
percentage of the voices, probably less
than 15 percent, are based on personal
interviews by Takaki and of these, most
are interviews of the author’s relatives
and colleagues. In this respect, the
author depends almost entirely on
available primary and secondary sources,
leaving readers wondering how
representative they are.
Also, unlike the Nees, Takaki’s
book offers no analytical breakthrough for
our understanding of either Asian
American history or the newly emerging
communities. Particularly disappointing is
Takaki’s silence on the historic emergence
of “Asian America” in the late 1960s and
early 1970s and his ahistorical approach
to the ensuing decades in Asian American
history. The rise of ethnic and political
consciousness, the emergence of new
community organizations, and the rise of
Asian American studies are among the
most important developments in the
history of Asians in the United States, in
my view.
Unfortunately both Takaki and
Roger Daniels [in the latter’s Asian
America: Chinese and Japanese in the
40
nR
用
禁
U
严
45
50
55
file://E:\新建文件夹\e8.htm
2006-11-12
页码,8/18
The Official SAT Online Course
60 U.S. since 1850] failed to adequately treat
this period of “Asian America” that gave
them the titles for their books, a term that
captures the richness of the newly
emerged Asian American political and
65 cultural expressions.
In this respect, book reviewers are
wrong in characterizing Takaki’s book as a
“comprehensive history of 150 years of
Asian experience in the U.S.” It is more
70 accurate to describe the book as a history
of Asians in the U.S. up to World War II,
with the final two chapters of anecdotal
events since then tacked on as an
epilogue.
Passage 2
!
途
Comparing Strangers from a
Different Shore to Victor and Brett Nee’s
Longtime Californ’, Wang points out that I
did not collect the voices myself. However
my study is a very different one from the
80 Nees’. Theirs studied only one ethnic
group and only one geographical
community—San Francisco Chinatown.
Mine is comparative and also far more
comprehensive: it studies the Chinese,
85 Japanese, Koreans, Filipinos, Asian
Indians, Vietnamese…Thus, given the
tremendous scope of my study, I had to
collect most of the voices from indirect
sources such as the Ethnic Studies Oral
90 History Project, and I acknowledged them
through the study….
But I also directly interviewed
people. One of them, Touly Xiong, a
Hmong refugee now living in Wisconsin,
95 told me: “We hope you will include our
stories in your book. Americans need to
understand us and what we have gone
through.” He said that he had fought for
the US in Laos and that his brother had
100 been killed by North Vietnamese soldiers.
I also interviewed many of my relatives.
One of the meaningful things I discovered
and I hope other Asian American scholars
will learn is that our family histories are
105 tied to the histories of our communities.
We, as scholars, are members of
communities, and our aunts, uncles, and
cousins have stories and voices that
ed
75
er
用
st
业
eg
i
商
于
nR
用
禁
U
严
file://E:\新建文件夹\e8.htm
2006-11-12
页码,9/18
The Official SAT Online Course
belong to Asian American history….
110
But what about Wang’s claim
that I “failed” to “adequately” treat the
post-World War II period? I wish I had
given more attention to the 1950s, but I
felt that the book was already an
115 enormously long one and also that I
wanted to focus more on the post-1965
period. And here it is not accurate to
describe my book as “a history of Asians
in the US up to World War II.” The last
120 two chapters (out of a total of twelve
chapters) are devoted to exactly this time
period. As any reader will see, they are
not “chapters of anecdotal events. . .
tacked on as an epilogue,” as Wang
125 claims. Rather they are designed to relate
the early period to the more recent history
in a substantive way. They present an
analysis of post-World War II
developments….These certainly are not
130 “anecdotal events.”
re
d
!
途
用
业
is
te
7
As used in line 1, “approach” most nearly means
(A)
beginning
(B)
technique
(C) hypothesis
eg
style
用
nR
(E)
商
于
(D) advance
ANSWERS
禁
AND EXPLANATIONS
Explanation for Correct Answer B :
Choice (B) is correct. In context, “approach” most nearly means “technique.”
The author of Passage 1 is comparing two books, saying that “both are media
through which diverse Asian American voices surface”—clearly a reference to both
writers’ technique, or methodology for treating their similar subject matter.
U
严
Explanation for Incorrect Answer A :
Choice (A) is incorrect. The author of Passage 1 is clearly comparing two books in
their entirety in terms of how they handle their subject matter—in other words,
their “technique,” or procedure. The author does not address whether or not the
two books begin in the same manner. Their “beginnings” may or may not be
similar.
Explanation for Incorrect Answer C :
Choice (C) is incorrect. In comparing two books, the author of Passage 1 is
addressing their similar ways of handling their subject matter—in other words, their
“technique,” or procedure. The author does not address whether or not the two
books share a hypothesis, which in this context would mean a philosophical
assumption. The paragraph goes on to compare the books in terms of what they
do, not in terms of the assumptions they may or may not share.
Explanation for Incorrect Answer D :
Choice (D) is incorrect. While “approach” as a verb may have some connection to
“advance” as a verb, it does not make sense to say that the two books have
similar “advances.” In this section, the author of Passage 1 is clearly referring to
methods or procedures that the authors of both books share—in other words, their
file://E:\新建文件夹\e8.htm
2006-11-12
页码,10/18
The Official SAT Online Course
“techniques.”
Explanation for Incorrect Answer E :
Choice (E) is incorrect. The author of Passage 1 is clearly speaking here of the two
books’ similar “technique,” or methods of handling their subject matter. They
may or may not be similar in style, which would more likely refer to literary
elements than to scholarly methodology, or “approach.”
8
In context, “surface” (line 20) is closest in meaning to
(A)
arrange
(B)
level
(C) float
(D) emerge
(E)
smooth
ANSWERS
AND EXPLANATIONS
Explanation for Correct Answer D :
Choice (D) is correct. The author is saying here that the Nees’ book provided
information that had never before been disclosed about a certain community. It is
clear from context (“never before been permitted to”) that this information has
been willfully or deliberately kept obscure; in revealing this information, the Nees
have allowed it to “emerge,” or come forth from obscurity.
re
d
!
途
te
用
业
Explanation for Incorrect Answer A :
Choice (A) is incorrect. The author of the passage is speaking here about the
information that the Nees’ book has brought to light, or has allowed to
“emerge.” In this context, it makes little sense to say that the voices in the book
have been permitted to “arrange,” or “come to an agreement.”
eg
is
商
于
Explanation for Incorrect Answer B :
Choice (B) is incorrect. The author of the passage is speaking here about the
information that the Nees’ book has brought to light, or has allowed to
“emerge.” In this context, it makes little sense to say that the voices in the book
have been permitted to “level,” either in the sense of “equalize” or “tear
down.”
nR
用
禁
Explanation for Incorrect Answer C :
Choice (C) is incorrect. While an object that has been submerged might be said to
“float” to the surface, the author clearly is not speaking so literally here. Rather,
he is speaking of previously obscure voices that have been allowed to “emerge,”
or become evident.
U
严
Explanation for Incorrect Answer E :
Choice (E) is incorrect. The author of the passage is speaking here about the
information that the Nees’ book has brought to light, or has allowed to
“emerge.” In this context, it makes little sense to say that the voices in the book
have been permitted to “smooth,” or to be become level or flat.
9
In paragraph 2, the author of Passage 1 implies that a major difference between the
Nees’ book and Takaki’s book is that
(A)
the Nees’ book draws extensively on interviews conducted by the authors
(B)
Takaki’s study focuses on U.S. citizens with roots in only one Asian
nation
(C) the Nees make no significant analytical advances
(D) Takaki places great emphasis on the rise of Asian American studies
(E)
the Nees’ book uses quotations from literary works and oral histories
ANSWERS
file://E:\新建文件夹\e8.htm
AND EXPLANATIONS
2006-11-12
页码,11/18
The Official SAT Online Course
Explanation for Correct Answer A :
Choice (A) is correct. In the second paragraph, the author of Passage 1 contrasts
Takaki’s book with the Nees’ book, saying that the former relies on voices that
are “mostly collected from indirect sources” and, further, that Takaki, “unlike
the Nees,” acquired “only a very tiny percentage” of his sources from direct
interviews. Together, these comparisons imply that, in the view of the author, the
Nees rely heavily on interviews that they themselves conducted, an area in which
the author considers Takaki somewhat deficient.
Explanation for Incorrect Answer B :
Choice (B) is incorrect. In this paragraph, the author does not address the Asian
roots of the people interviewed by either Takaki or the Nees; rather, he focuses on
what he perceives as the shortage of direct interviews undertaken by Takaki in the
preparation of the book.
Explanation for Incorrect Answer C :
Choice (C) is incorrect. Though in the first paragraph the author says that the
Nees’ book “broke no new analytical ground,” he does not address this subject
in paragraph 2. Further, the author nowhere implies that analytical advances are
made by Takaki and not by the Nees, an assumption that underlies option C.
Explanation for Incorrect Answer D :
Choice (D) is incorrect. Elsewhere in the passage, the author implies that Takaki has
placed insufficient emphasis on the rise of the field of Asian American studies. The
author does not address this topic at all in the second paragraph of the passage.
d
!
途
re
Explanation for Incorrect Answer E :
Choice (E) is incorrect. While the author does mention “literary works and oral
histories” in the second paragraph, he attributes their use to Takaki, not to the
Nees. This paragraph is devoted to what the author sees as Takaki’s book’s
deficiencies, among them a reluctance to differentiate among quotations from
literary works and quotations from oral histories.
is
te
用
业
eg
商
于
In context, “distinguish” (line 28) most nearly means
(A)
mark
(B)
differentiate
用
nR
10
(C) analyze
禁
(D) judge
(E)
U
严
discover
ANSWERS
AND EXPLANATIONS
Explanation for Correct Answer B :
Choice (B) is correct. In this part of the passage, the author is accusing Takaki of
neglecting to “distinguish” one kind of literary voice from another kind in his
book; he means that Takaki does not “differentiate,” or make a distinction,
among quotations from literary works and quotations from documentary sources.
Explanation for Incorrect Answer A :
Choice (A) is incorrect. Although “mark” can be said to mean “distinguish,” in
the context of this passage the author is not speaking of physically marking
different kinds of quotations in order to keep them distinct; rather, he is speaking
of making a distinction among different kinds of quotations in the text of a book.
Explanation for Incorrect Answer C :
Choice (C) is incorrect. In this part of the passage, the author is accusing Takaki of
neglecting to “distinguish,” or “differentiate,” among various types of
quotations in the text of his book. It does not make logical sense or syntactic sense
to say that Takaki fails to “analyze,” or methodically examine, one type of
quotation or source “from” another type.
Explanation for Incorrect Answer D :
file://E:\新建文件夹\e8.htm
2006-11-12
页码,12/18
The Official SAT Online Course
Choice (D) is incorrect. In this part of the passage, the author is accusing Takaki of
neglecting to “distinguish,” or “differentiate,” among various types of
quotations in the text of his book. Although “judge” can be said to mean
“distinguish,” it is clear from context that the author would have preferred that
Takaki make some sort of distinction among various types of quotations, not that
Takaki “judge,” or evaluate, such types.
Explanation for Incorrect Answer E :
Choice (E) is incorrect. In this part of the passage, the author is accusing Takaki of
neglecting to “distinguish,” or “differentiate,” among various types of
quotations in the text of his book. It does not make logical sense or syntactic sense
to say that Takaki fails to “discover,” or learn about for the first time, one type of
quotation or source “from” another type.
11
The author of Passage 1 mentions Roger Daniels primarily as an example of
(A)
an historian who wrote an essay critical of Takaki’s study
(B)
a scholar who was frequently consulted by Takaki
(C) a writer whose research methods the author admires
(D) an author whose book shares a weakness with Takaki’s
someone who assisted Takaki in writing his book
ANSWERS
AND EXPLANATIONS
!
途
d
(E)
re
Explanation for Correct Answer D :
Choice (D) is correct. When the author of Passage 1 mentions Roger Daniels in lines
57-65 (“Unfortunately . . . expressions”), he states that Daniels, along with
Takaki, “failed to adequately treat this period,” that is, a period during which, in
the author’s view, Asian Americans organized themselves into political and
cultural groups. The author considers this omission a shortcoming of both Takaki’s
and Daniels’ books; in other words, both works “share a weakness.”
eg
is
te
用
业
商
于
Explanation for Incorrect Answer A :
Choice (A) is incorrect. Daniels is mentioned as a scholar whose work, in the
opinion of the author, has a shortcoming similar to one found in Takaki’s work.
Nowhere in the passage is he identified as writing anything about Takaki’s work.
nR
用
禁
Explanation for Incorrect Answer B :
Choice (B) is incorrect. Daniels is mentioned as a scholar whose work, in the
opinion of the author, has a shortcoming similar to one found in Takaki’s work.
Nowhere in the passage is he identified as either someone Takaki consulted or
someone whose work Takaki consulted in preparing his own book—the latter is
likely, of course, but it is not mentioned in the passage.
U
严
Explanation for Incorrect Answer C :
Choice (C) is incorrect. Daniels is mentioned as a scholar whose work, in the
opinion of the author, has a shortcoming similar to one found in Takaki’s work. If
the author of the passage expresses any opinion at all of Daniels’ work, it is
disapproval: “[Both books] failed to adequately treat this period….”
Explanation for Incorrect Answer E :
Choice (E) is incorrect. Daniels is mentioned as a scholar whose work, in the
opinion of the author, has a shortcoming similar to one found in Takaki’s work.
The two authors’ sharing what the author of the passage considers a shortcoming
does not imply that the two writers ever worked together, much less “assisted”
one another.
12
In context, “respect” (line 66) is closest in meaning to
(A)
regard
(B)
appreciation
(C) reference
(D) esteem
file://E:\新建文件夹\e8.htm
2006-11-12
页码,13/18
The Official SAT Online Course
(E)
recognition
ANSWERS
AND EXPLANATIONS
Explanation for Correct Answer A :
Choice (A) is correct. In lines 66 and following, the author of Passage 1 is referring
back to the preceding paragraph, where two books are compared in terms of a
common flaw. “This respect,” then, at the beginning of a new paragraph, refers
to something like “Takaki’s neglect of Asian American history after World War
II,” with “respect” meaning “regard,” or aspect.
Explanation for Incorrect Answer B :
Choice (B) is incorrect. In lines 66 and following, the author of Passage 1 is
referring back to the preceding paragraph, where two books are compared in terms
of a common flaw. “This respect,” then, at the beginning of a new paragraph,
refers to something like “Takaki’s neglect of Asian American history after World
War II.” It makes no sense, then, to say “In this appreciation”—or recognition
of value—in this particular context.
Explanation for Incorrect Answer C :
Choice (C) is incorrect. In lines 66 and following, the author of Passage 1 is
referring back to the preceding paragraph, where two books are compared in terms
of a common flaw. “This respect,” then, at the beginning of a new paragraph,
refers to something like “Takaki’s neglect of Asian American history after World
War II.” While it might make sense in some contexts to say “In this
reference”—or denotation—“critics are wrong,” the author makes it clear here
that he means “respect” in the sense of “regard,” or “aspect.”
er
e
d
!
途
用
Explanation for Incorrect Answer D :
Choice (D) is incorrect. In lines 66 and following, the author of Passage 1 is
referring back to the preceding paragraph, where two books are compared in terms
of a common flaw. “This respect,” then, at the beginning of a new paragraph,
refers to something like “Takaki’s neglect of Asian American history after World
War II.” It makes no sense, then, to say “In this esteem”—or favorable
regard—in this particular context.
st
业
eg
i
商
于
Explanation for Incorrect Answer E :
Choice (E) is incorrect. In lines 66 and following, the author of Passage 1 is
referring back to the preceding paragraph, where two books are compared in terms
of a common flaw. “This respect,” then, at the beginning of a new paragraph,
refers to something like “Takaki’s neglect of Asian American history after World
War II.” It makes no sense, then, to say “In this recognition”—or favorable
notice—in this particular context.
nR
用
严
U
13
禁
Which of the following best describes the attitude of the author of Passage 1 toward
Takaki’s book?
(A)
disbelieving
(B)
confrontational
(C) exasperated
(D) outraged
(E)
dissatisfied
ANSWERS
AND EXPLANATIONS
Explanation for Correct Answer E :
Choice (E) is correct. The author of Passage 1 takes issue with two major elements
of Takaki’s book: its research methodology (paragraph 2); and its treatment of
Asian Americans in the U.S. in the 1950’s and 1960’s (paragraphs 3-5). In using
such words as “disappointing,” “unfortunately,” “failed to treat,” and
“tacked on as an epilogue,” the author betrays dissatisfaction, or discontent,
with the book. The passage, however, maintains a scholarly detachment in its
appraisal of the book, avoiding harsh language and outright attacks.
file://E:\新建文件夹\e8.htm
2006-11-12
页码,14/18
The Official SAT Online Course
Explanation for Incorrect Answer A :
Choice (A) is incorrect. The author of Passage 1 is clearly not completely satisfied
with Takaki’s book, and finds fault with two of its major elements. But nowhere
does the author say or imply that Takaki is less than truthful in his reporting or that
he is withholding the truth, so the author’s attitude cannot be called
“disbelieving.”
Explanation for Incorrect Answer B :
Choice (B) is incorrect. The author of Passage 1 is clearly not completely satisfied
with Takaki’s book, and finds fault with two of its major elements. But nowhere
does the author sound “confrontational,” or hostile—the author does not, for
example, challenge Takaki to defend his sources or his methods, which would be
confrontational.
Explanation for Incorrect Answer C :
Choice (C) is incorrect. While the author of the passage is clearly not completely
satisfied with Takaki’s book, and finds fault with two of its major elements, he
nowhere sounds “exasperated,” or angrily impatient. Throughout the passage,
the tone is one of scholarly detachment, avoiding harsh language and outright
attacks.
Explanation for Incorrect Answer D :
Choice (D) is incorrect. While the author of the passage is clearly not completely
satisfied with Takaki’s book, and finds fault with two of its major elements, he
nowhere sounds “outraged,” or grossly offended. Throughout the passage, the
tone is one of scholarly detachment, avoiding harsh language and outright attacks.
用
The author of Passage 2 asserts that the Nees’ book is inferior to Strangers from a
Different Shore in terms of its
(A)
purpose
(B)
readability
业
商
eg
(D) accuracy
is
(C) documentation
(E)
te
14
re
d
!
途
于
range
用
AND EXPLANATIONS
nR
ANSWERS
Explanation for Correct Answer E :
Choice (E) is correct. In the first paragraph of Passage 2, Takaki contrasts
Strangers from a Different Shore with Longtime Californ’, pointing out that the
former “studied only one ethnic group and only one geographical community,”
while the latter is “comparative and also far more comprehensive”; three lines
later, he refers to “the tremendous scope of my study.” Thus, Takaki is
comparing his book to the Nees’ book in terms of its scope, or range of material
covered, and considers his superior.
禁
U
严
Explanation for Incorrect Answer A :
Choice (A) is incorrect. The author of Passage 2 does not really mention
“purpose” in regard to either book. He does say that Strangers from a Different
Shore is “different,” and “more comprehensive,” but he is basically addressing
range or scope rather than purpose with these descriptions.
Explanation for Incorrect Answer B :
Choice (B) is incorrect. The passage does not compare the two books in terms of
their relative “readability,” or ease or pleasure with which they can be read.
Explanation for Incorrect Answer C :
Choice (C) is incorrect. While the author of Passage 2 does discuss matters of
“documentation”—in, for example, lines 86-91—nowhere in the passage does he
assert or imply that his methods are superior to those of the Nees, merely that the
scope of his work necessitated a different kind of documentation.
Explanation for Incorrect Answer D :
Choice (D) is incorrect. The author of Passage 2 does mention “accuracy” in the
last paragraph of the passage, but it is in connection with Li Ling-chi Wang, the
file://E:\新建文件夹\e8.htm
2006-11-12
页码,15/18
The Official SAT Online Course
author of Passage 1, and not with the Nees’ work.
15
The author of Passage 2 mentions the Ethnic Studies Oral History Project (lines 8990) primarily in order to
(A)
cite an organization that praised his study
(B)
highlight a valuable resource overlooked by the Nees
(C) defend the validity of his use of secondary sources
(D) acknowledge the assistance he received from earlier scholars
(E)
refer to a study he conducted earlier in his career
ANSWERS
AND EXPLANATIONS
Explanation for Correct Answer C :
Choice (C) is correct. In this part of the passage, the author is speaking of the
“tremendous scope” of his book, which necessitated his using “indirect
sources” such as the Ethnic Studies Oral History Project rather than relying solely
on direct interviews. He is also defending himself against the charge made by the
author of Passage 1: that he did not conduct enough personal interviews.
Therefore, he is citing the Ethnic Studies Oral History Project by way of defending
his use of it and of other “secondary sources.”
re
d
!
途
用
te
Explanation for Incorrect Answer A :
Choice (A) is incorrect. There is no indication in the passage that the Ethnic Studies
Oral History Project either praised or criticized Takaki’s study, only that Takaki
himself drew on the Project in his work.
业
Explanation for Incorrect Answer B :
Choice (B) is incorrect. While it is clear from context that Takaki considers the
Ethnic Studies Oral History Project “a valuable resource”—having used it
himself—there is no indication that the Nees either used it or did not use it in
preparing their study.
eg
is
商
于
Explanation for Incorrect Answer D :
Choice (D) is incorrect. While it can be inferred that Takaki and other scholars draw
upon the work of their predecessors, there is no direct mention in the passage of
Takaki’s “receiving assistance” nor any acknowledgement of the Ethnic Studies
Oral History Project as the work of “earlier scholars” who might have been in a
position to provide “assistance.”
nR
用
禁
U
严
Explanation for Incorrect Answer E :
Choice (E) is incorrect. There is no indication in the passage that Takaki was in any
way involved with the authorship of the Ethnic Studies Oral History Project, only
that he drew on it in documenting Strangers from a Different Shore.
16
The author of Passage 2 uses the direct quotation in lines 95-98 most probably in
order to
(A)
discredit an assertion made in the Nees’ book
(B)
demonstrate that his book has gained a wide readership
(C) provide evidence that indirect sources are as valid as direct sources
(D) introduce claims about an ethnic group not discussed in his book
(E)
counter a criticism made by the author of Passage 1
ANSWERS
AND EXPLANATIONS
Explanation for Correct Answer E :
Choice (E) is correct. The quotation, from a Hmong refugee, Touly Xiong, is
provided in part to support the claim, “But I also directly interviewed people.”
This assertion is, in turn, made to refute a claim made by the author of Passage 1,
paraphrased by Takaki as “Wang points that I did not collect the voices myself”
(lines 77-78). So including the direct quotation is intended to “counter,” or
file://E:\新建文件夹\e8.htm
2006-11-12
页码,16/18
The Official SAT Online Course
refute, “a criticism made by the author of Passage 1” (that Takaki did not
conduct interviews).
Explanation for Incorrect Answer A :
Choice (A) is incorrect. While Takaki states in Passage 2 that the Nees’ book is
less ambitious than his own (lines 80-84), he nowhere discredits any of the Nees’
assertions. The quotation in lines 95-98 can be seen as an attempt “to discredit an
assertion,” but it is an assertion made in the passage, not the Nees’ book.
Explanation for Incorrect Answer B :
Choice (B) is incorrect. The quotation in lines 95-98 is from a person interviewed by
Takaki in the preparation of the latter’s book; there is no indication that the
person quoted has read the book, nor is there any indication anywhere in the
passage about the scope of Takaki’s audience.
Explanation for Incorrect Answer C :
Choice (C) is incorrect. While in the paragraph preceding this one Takaki defends
the use of “indirect sources,” in lines 95-98 he is quoting a person he actually
interviewed—in other words, addressing the validity of direct sources.
Explanation for Incorrect Answer D :
Choice (D) is incorrect. While Touly Xiong, the person quoted in lines 95-98, is
identified as a Hmong refugee who has left Asia and now lives in the United States,
it is obvious from context that Takaki is using the quotation to refer to an ethnic
group that is discussed in Strangers from a Different Shore.
用
With the parenthetical information in lines 120-121 Takaki intends primarily to
业
(A)
imply that his book is a relatively lengthy one
(B)
illustrate the significance the post-World War II era has in his book
商
eg
is
17
te
re
d
!
途
(C) tell readers where his treatment of the post-World War II era can be found
(D) compare his book to a previous book about the post-World War II era
(E)
于
describe the arrangement of chapters in his book
用
AND EXPLANATIONS
nR
ANSWERS
禁
Explanation for Correct Answer B :
Choice (B) is correct. In this paragraph, the author is defending his book against
Wang’s charge that he “‘failed’ to ‘adequately’ treat the post-World War
II period.” In lines 118-119, he quotes Wang’s description of Takaki’s book as
“a history of Asians in the US up to World War II,” then insists that “the last
two chapters (out of a total of twelve chapters) are devoted to exactly this time
period”—that is, the years after World War II. Thus, by saying exactly how much
of his book is devoted to this period, Takaki is using the parenthetical information
to express how significant this period is to his book as a whole.
U
严
Explanation for Incorrect Answer A :
Choice (A) is incorrect. The parenthetical information tells how much of Takaki’s
book—two out of twelve chapters—is devoted to a certain topic. Though he does
indicate in lines 114-115 that his book is “enormously long,” he is not using the
information in parenthesis to illustrate the overall length of the book.
Explanation for Incorrect Answer C :
Choice (C) is incorrect. While the paragraph as a whole suggests that Takaki’s
treatment of post-World War II Asian American life comes near the end of his book,
the parenthetical information is clearly intended to illustrate how great a proportion
of the entire book is taken up with this topic, not where in the book the topic can be
found.
Explanation for Incorrect Answer D :
Choice (D) is incorrect. The parenthetical information in no way refers to any other
book; rather, Takaki is here referring to how much of his own book—two out of
twelve chapters—is concerned with a single topic, Asian American history after
World War II.
file://E:\新建文件夹\e8.htm
2006-11-12
页码,17/18
The Official SAT Online Course
Explanation for Incorrect Answer E :
Choice (E) is incorrect. While Takaki does say in this paragraph that the chapters of
his book that deal with the post-World War II period come at the end of the book,
he is clearly using the parenthetical information to demonstrate what proportion of
his book is devoted to the topic.
18
The author of Passage 2 uses quotation marks in line 130 most likely in order to
(A)
indicate that he is using a common expression
(B)
highlight an important concept
(C) cite a judgment with which he disagrees
(D) emphasize a humorous statement
(E)
call attention to an unusual phrase
ANSWERS
AND EXPLANATIONS
Explanation for Correct Answer C :
Choice (C) is correct. In this paragraph, Takaki is refuting Li Lin-chi Wang’s
contention that Takaki’s book underemphasizes an important period of Asian
American history. In lines 122-124, he quotes Wang's description of the book's last
two chapters as "Chapters of anecdotal events...tacked on as an epilogue." Takaki,
in lines 127-130, insists that the chapters are more substantive than Wang claims,
directly quoting Wang’s judgment in order to disagree with it.
is
te
re
d
!
途
用
业
Explanation for Incorrect Answer A :
Choice (A) is incorrect. Quotation marks are more often used to indicate an
uncommon expression than a common one; but in this context, it is clear that
Takaki is quoting another author’s work, and that he takes issue with it.
商
Explanation for Incorrect Answer B :
Choice (B) is incorrect. While quotations marks might feasibly be used to highlight
an important concept, it is clear in this context that Takaki is quoting another
author’s work, and that he takes issue with it.
eg
于
用
nR
Explanation for Incorrect Answer D :
Choice (D) is incorrect. Since Takaki is quoting a negative assessment of his work
here, it is unlikely that he is doing so in a humorous way. The tone of the
paragraph as a whole does not support this interpretation.
禁
严
U
Explanation for Incorrect Answer E :
Choice (E) is incorrect. While quotation marks are often used to call attention to an
unusual phrase, it is clear from context that Takaki is refuting a judgment with
which he disagrees; he does, after all, quote the judgment just above, in lines 122125.
19
The authors of Passage 1 and Passage 2 would most probably agree with one another
about which of the following statements concerning scholarly studies of Asian
American communities?
(A)
Asian American scholars should focus on earlier immigrant groups rather
than on those who immigrated later.
(B)
Oral histories are less credible than is written documentation of Asian
American history.
(C) Interviewing one’s own family members is an effective way to represent
the voices of one’s community.
(D) Post-World War II Asian American history is just as important as the
history of earlier periods of Asian American history.
(E)
A study that focuses solely on San Francisco can treat the history of Asian
American immigrants in a sufficiently thorough way.
ANSWERS
AND EXPLANATIONS
Explanation for Correct Answer D :
file://E:\新建文件夹\e8.htm
2006-11-12
页码,18/18
The Official SAT Online Course
Choice (D) is correct. In Passage 1, Wang criticizes Takaki for underplaying the
post-World War II era in Strangers from a Different Shore, terming the events of
the late 1960’s and early 1970's “among the most important developments in
the history of Asians in the United States” (lines 53-55). While Takaki obviously
disagrees with Wang’s assessment of the book, he does acknowledge the
importance of the period, insisting in the last paragraph of Passage 2 that two out of
twelve chapters of his book are dedicated to the period. Thus, it can be inferred that
both authors agree about the importance of the post-World War II period in the
study of Asian American history.
Explanation for Incorrect Answer A :
Choice (A) is incorrect. As it happens, both passages spend more time discussing
relatively recent immigrants, but there is no indication in either passage that the
authors consider earlier immigrants more worthy of study then later immigrants, or
vice versa.
Explanation for Incorrect Answer B :
Choice (B) is incorrect. Far from considering oral histories less credible than written
sources, both authors seem convinced of the importance of oral histories: Wang
chides Takaki for failing to distinguish written histories from oral histories and for
underusing oral interviews (Passage 1, paragraph 2), and Takaki defends at some
length his use of oral histories (Passage 2, paragraph 2).
!
途
ed
Explanation for Incorrect Answer C :
Choice (C) is incorrect. While Takaki defends the scholarly usefulness of
interviewing family members (“our aunts, uncles, and cousins have stories and
voices that belong to Asian American history”), Wang seems unenthusiastic about
the practice, critically stating that “most [of Takaki’s interviews] are of the
author’s relatives and colleagues.”
st
er
用
Explanation for Incorrect Answer E :
Choice (E) is incorrect. Both passages mention Longtime Californ’ by Victor and
Bret Nee as a study that focuses solely on Chinese immigrants in San Francisco, but
neither Wang nor Takaki says or implies that the book is thoroughly satisfactory:
Wang says that it “broke no new [analytical] ground” (lines 16-17), while Takaki
compares his book favorably to the Nees’ in terms of scope (lines 80-86).
业
eg
i
商
于
nR
用
禁
Copyright © 2006 The College Board. All rights reserved.
Back to Score Report
Privacy Policy
Terms of Use
Contact Us
U
严
file://E:\新建文件夹\e8.htm
2006-11-12
页码,1/14
The Official SAT Online Course
Help | Profile | My Organizer | My Bookmarks | Logout
Answers and Explanations
Back to Score Report
Test Sections
Section 1
View Answers and Explanations
Section 2
Online - Practice Test #5
Section 3
Section 4
1
Section 5
Section 6
Section 8
Section 9
Section 10
ed
!
途
Of the labeled points on the cube shown above, which is farthest from point
用
st
er
(A)
业
(B)
(C)
商
(E)
eg
i
(D)
ANSWERS
于
AND EXPLANATIONS
nR
用
Explanation for Correct Answer C :
is the length of each edge of the cube, then points
Choice (C) is correct. If
禁
and
are each distance
from point
the hypotenuses of right triangles with both legs of length
and
and that between points
and
between points
U
严
greater than
points
and
so the distance
must each be
and
Furthermore, the line segment between points
hypotenuse of the right triangle with legs
and
is the
so the distance between
must be greater than the distance between points
Therefore, of the labeled points, point
are
and
The line segments
and
is farthest from point
Explanation for Incorrect Answer A :
is one of the labeled points that is closest to
Choice (A) is not correct. Point
The question asks for the labeled point that is farthest from
point
Explanation for Incorrect Answer B :
Choice (B) is not correct. The line segment between points
hypotenuse of the right triangle with legs
points
and
and
must be greater than the distance between points
Explanation for Incorrect Answer D :
Choice (D) is not correct. The line segment between points
hypotenuse of the right triangle with legs
points
and
and
and
is the
so the distance between
and
and
is the
so the distance between
must be greater than the distance between points
and
Explanation for Incorrect Answer E :
is one of the labeled points that is closest to
Choice (E) is not correct. Point
file://E:\新建文件夹\e9.htm
2006-11-12
页码,2/14
The Official SAT Online Course
point
The question asks for the labeled point that is farthest from
2
inch represents
In a scale model of a statue,
inches. If the statue is
inches tall, what is the height of the scale model?
(A)
inches
(C)
inches
(D)
inches
(E)
inches
ANSWERS
!
途
AND EXPLANATIONS
d
(B)
inches
Choice (A) is correct. If
te
re
Explanation for Correct Answer A :
用
业
inches tall, then
and if the statue is
inches on the statue,
inch on the model represents
where
商
is the height, in
is
inches, of the scale model of the statue. Solving this proportion for
gives
nR
eg
于
用
禁
Explanation for Incorrect Answer B :
Choice (B) is not correct. If the scale model were
U
严
would hold. Solving this proportion
inches tall, then the proportion
But
gives
inches tall and the statue were
is
not
Explanation for Incorrect Answer C :
Choice (C) is not correct. If the scale model were
were
inches tall, then the proportion
proportion gives
inches tall and the statue
would hold. Solving this
But
is
not
Explanation for Incorrect Answer D :
Choice (D) is not correct. If the scale model were
were
file://E:\新建文件夹\e9.htm
inches tall, then the proportion
inches tall and the statue
would hold. Solving this
2006-11-12
页码,3/14
The Official SAT Online Course
Explanation for Incorrect Answer E :
Choice (E) is not correct. If the scale model were
were
inches tall and the statue
would hold. Solving this
inches tall, then the proportion
not
is
But
proportion gives
not
is
But
proportion gives
3
In the figure above, lines
业
what is the value of
eg
于
用
nR
(E)
用
intersect. If
is
(B)
(D)
and
商
(A)
(C)
te
re
d
!
途
禁
ANSWERS
AND EXPLANATIONS
Explanation for Correct Answer D :
Choice (D) is correct. The angled labeled
U
严
angles labeled
and the angle consisting of all three
are vertical angles, so
Explanation for Incorrect Answer A :
Choice (A) is not correct. This choice,
is too small. It could be the result of
visually estimating the answer, but the question does not ask for an estimate.
Explanation for Incorrect Answer B :
is too small. It could be the result of
Choice (B) is not correct. This choice,
visually estimating the answer, but the question does not ask for an estimate.
Explanation for Incorrect Answer C :
is too small. It could be the result of
Choice (C) is not correct. This choice,
visually estimating the answer, but the question does not ask for an estimate.
Explanation for Incorrect Answer E :
is too large. It could be the result of
Choice (E) is not correct. This choice,
visually estimating the answer, but the question does not ask for an estimate.
file://E:\新建文件夹\e9.htm
2006-11-12
页码,4/14
The Official SAT Online Course
4
is
If
in terms of
what is
more than twice
(A)
(B)
(C)
(D)
(E)
ANSWERS
AND EXPLANATIONS
Explanation for Correct Answer E :
Choice (E) is correct. The statement “
” can be written
from both sides of this equation gives
Subtracting
as
more than twice
is
yields
now dividing both sides of the resulting equation by
!
途
ed
Therefore,
用
then
st
er
Explanation for Incorrect Answer A :
were equal to
Choice (A) is not correct. If
which is not equivalent to “
more than twice
业
Explanation for Incorrect Answer B :
were equal to
Choice (B) is not correct. If
商
more than twice
eg
i
which is not equivalent to “
于
then
would be equal to
”
would be equal to
”
Explanation for Incorrect Answer C :
Choice (C) is not correct. If
用
were equal to
nR
which is not equivalent to “
禁
then
more than twice
would be equal to
”
Explanation for Incorrect Answer D :
Choice (D) is not correct. If
U
严
were equal to
which is not equivalent to “
then
more than twice
would be equal to
”
5
students, the amount of time the
The scatterplot above shows, for each of
student spent on homework plotted against the amount of time the student spent on
file://E:\新建文件夹\e9.htm
2006-11-12
页码,5/14
The Official SAT Online Course
after-school activities last week. According to the scatterplot, which of the following
statements is true?
(A)
Each of the students spent the same amount of time on homework as on
after-school activities.
(B)
Each of the students spent more time on homework than on after-school
activities.
(C) Each of the students spent less time on homework than on after-school
activities.
(D) Exactly
students spent
hours on homework.
(E)
students spent
hours on after-school activities.
Exactly
ANSWERS
AND EXPLANATIONS
Explanation for Correct Answer D :
hours on homework
Choice (D) is correct. The number of students who spent
can be found by counting the number of dots that fall on the vertical line
hours of homework. There are three such dots, corresponding to
representing
students spent
hours of after-school activities. Therefore, exactly
and
hours on homework.
!
途
re
d
Explanation for Incorrect Answer A :
Choice (A) is not correct. It is not true that each of the students spent the same
amount of time on homework as on after-school activities. For example, the lowest
hour on
hours on homework and
leftmost dot represents a student who spent
after-school activities.
is
te
用
业
Explanation for Incorrect Answer B :
Choice (B) is not correct. It is not true that each of the students spent more time
on homework than on after-school activities. For example, the highest rightmost
hours on afterhours on homework and
dot represents a student who spent
school activities.
eg
商
于
Explanation for Incorrect Answer C :
Choice (C) is not correct. It is not true that each of the students spent less time on
homework than on after-school activities. For example, the lowest leftmost dot
hour on after-school
hours on homework and
represents a student who spent
activities.
nR
用
禁
Explanation for Incorrect Answer E :
Choice (E) is not correct. It is not true that exactly
hours on
students spent
hours on after-school
after-school activities. The number of students who spent
activities can be found by counting the number of dots on the horizontal line that
hours of after-school activities. There are only two such dots, one
represents
U
严
corresponding to
homework.
6
If the function
hours of homework and one corresponding to
hours of
what is the value of
is defined by
(A)
(B)
(C)
(D)
(E)
ANSWERS
AND EXPLANATIONS
Explanation for Correct Answer B :
Choice (B) is correct. If
file://E:\新建文件夹\e9.htm
then
2006-11-12
页码,6/14
The Official SAT Online Course
Explanation for Incorrect Answer A :
Therefore,
not
Choice (A) is not correct.
not
Explanation for Incorrect Answer C :
because
cannot be
Choice (C) is not correct. The value of
Explanation for Incorrect Answer D :
Choice (D) is not correct. This incorrect answer could be the result of incorrectly
computing the sign of
Explanation for Incorrect Answer E :
Therefore,
not
Choice (E) is not correct.
not
!
途
re
d
7
用
is
te
业
商
eg
于
nR
用
If the areas of the two rectangles in the figure above are equal, which of the following
could be the coordinates of point
禁
(A)
U
严
(B)
(C)
(D)
(E)
ANSWERS
AND EXPLANATIONS
Explanation for Correct Answer D :
is in Quadrant IV and the two rectangles in the
Choice (D) is correct. Since
is positive, the
-coordinate of
figure are of equal area, it follows that the
is negative, and the absolute value of the product of the
coordinate of
Of the given choices, only
is
-coordinate of
and the
coordinate of
satisfies these three conditions.
Explanation for Incorrect Answer A :
is in Quadrant IV, its
Choice (A) is not correct. Since
positive. Therefore,
-coordinate must be
cannot be the coordinates of
Explanation for Incorrect Answer B :
file://E:\新建文件夹\e9.htm
2006-11-12
页码,7/14
The Official SAT Online Course
is in Quadrant IV, its
Choice (B) is not correct. Since
-coordinate must be
-coordinate must be negative. Therefore,
positive and its
cannot be the
coordinates of
Explanation for Incorrect Answer C :
Choice (C) is not correct. The upper rectangle in the figure has area
then the lower rectangle would be of area
were
coordinates of point
If the
However, the question states that the two rectangles must be of equal area.
Explanation for Incorrect Answer E :
is in Quadrant IV, its
Choice (E) is not correct. Since
8
The
The
The
The
The
l
l
l
l
l
-coordinate must be
cannot be the coordinates of
negative. Therefore,
first person ordered a salad.
second person did not order a salad.
third person ordered a hamburger.
fourth person ordered the same thing as the first person.
fifth person ordered the same thing as the second person.
!
途
must be
(C)
must be
(D)
must be
(E)
must be
用
or
te
(B)
业
商
is
must be
or
eg
(A)
re
d
A lunch stand has three choices: hamburger, hot dog, or salad. Five people from an
office ordered one choice each from the lunch stand. The statements above are about
is the number of people who ordered a
what these five people ordered. If
hamburger, which of the following statements is true?
ANSWERS
于
AND EXPLANATIONS
Explanation for Correct Answer E :
Choice (E) is correct. The first person ordered a salad. The second ordered either a
hamburger or a hot dog. The third ordered a hamburger, and the fourth ordered a
salad. The fifth person ordered either a hamburger or a hot dog to match the order
of the second person. There are only two possibilities for the number of hamburgers
ordered, and they depend on whether the second person ordered a hamburger or a
hot dog. If the second person ordered a hot dog, the orders were salad, hot dog,
hamburger, salad, hot dog. If the second person ordered a hamburger, the orders
were salad, hamburger, hamburger, salad, hamburger. In the first case, the
In the second case, the
number of people who ordered a hamburger was
nR
用
禁
U
严
These are the only two
number of people who ordered a hamburger was
” must be true.
or
must be
possible cases, so the statement “
Explanation for Incorrect Answer A :
Choice (A) is not correct. The number of people who ordered a hamburger may be
must be ” is
Therefore, the statement “
but this number could also be
not true.
Explanation for Incorrect Answer B :
Choice (B) is not correct. The number of people who ordered a hamburger could not
be
Explanation for Incorrect Answer C :
Choice (C) is not correct. The number of people who ordered a hamburger may be
”
must be
Therefore, the statement “
but this number could also be
is not true.
Explanation for Incorrect Answer D :
Choice (D) is not correct. The number of people who ordered a hamburger could be
but this number could not be
file://E:\新建文件夹\e9.htm
2006-11-12
页码,8/14
The Official SAT Online Course
9
which of the following could be the value of
If
(A)
(B)
(C)
(D)
(E)
ANSWERS
AND EXPLANATIONS
Explanation for Correct Answer E :
!
途
could be the value of
er
ed
Therefore, of the choices given, only
and
are
Thus, the possible values for
then
Choice (E) is correct. If
用
Explanation for Incorrect Answer A :
业
then
However,
is
t
Choice (A) is not correct. If
商
not
于
eg
Explanation for Incorrect Answer B :
Choice (B) is not correct. If
用
nR
However,
禁
then
not
Explanation for Incorrect Answer C :
U
严
Choice (C) is not correct. If
undefined. However,
then
which is
is defined and equal to
Explanation for Incorrect Answer D :
Choice (D) is not correct. If
However,
10
then
not
and each term after the first is
The first term of a sequence is
previous term. Which of the following is an expression for the
more than the
term of the
sequence for any positive integer
(A)
(B)
(C)
file://E:\新建文件夹\e9.htm
2006-11-12
页码,9/14
The Official SAT Online Course
(D)
(E)
ANSWERS
AND EXPLANATIONS
Explanation for Correct Answer B :
Choice (B) is correct. From the given description of the sequence, the
This last formula can
term is
the
and for any positive integer
term is
term is
the
term is
the
also be written as
Another way to see this is to look at the terms of the sequence. The first five terms
Only the formula in choice (B) yields these terms for
and
are
and
Explanation for Incorrect Answer A :
Choice (A) is not correct. If for any positive integer
sequence were equal to
the
term of the
!
途
then the first term of the sequence would be
ed
But the first term of the sequence is
Explanation for Incorrect Answer C :
Choice (C) is not correct. If for any positive integer
the
term of the
then the first term of the sequence would be
sequence were equal to
用
te
r
But the first term of the sequence is
业
Explanation for Incorrect Answer D :
Choice (D) is not correct. If for any positive integer
the
term of the
then the first term of the sequence would be
sequence were equal to
eg
is
商
But the first term of the sequence is
Explanation for Incorrect Answer E :
Choice (E) is not correct. If for any positive integer
于
sequence were equal to
the
term of the
then the first term of the sequence would be
用
11
nR
But the first term of the sequence is
禁
U
严
In the figure above,
at random from
shaded regions?
and
are all squares. If a point is chosen
what is the probability that the point will be from one of the
(A)
(B)
(C)
file://E:\新建文件夹\e9.htm
2006-11-12
页码,10/14
The Official SAT Online Course
(D)
(E)
ANSWERS
AND EXPLANATIONS
Explanation for Correct Answer D :
Choice (D) is correct. The probability that a point chosen at random from
will be from one of the shaded regions is equal to the ratio of the total area of the
is equal to
The area of
shaded regions to the area of
is
and the area of the shaded region
is
Thus, the area of
so
square, it follows that
is also a
Since
is a square, it follows that
Since
is
Similarly, the area of the second shaded region,
equal to
Therefore, the probability that the chosen point
equal to
will be from one of the shaded regions is
!
途
re
d
Explanation for Incorrect Answer A :
Choice (A) is not correct. This is the probability that the point chosen at random
but the point could also be
will be from the shaded region
from
from the shaded region
用
Explanation for Incorrect Answer B :
te
业
but that is not equal to the
Choice (B) is not correct. This choice is equal to
商
will be from one of the
is
probability that a point chosen at random from
shaded regions.
eg
于
Explanation for Incorrect Answer C :
Choice (C) is not correct. This is the ratio of
to
but that is not equal to
the probability that a point chosen at random from
shaded regions.
nR
用
will be from one of the
Explanation for Incorrect Answer E :
禁
Choice (E) is not correct.
is the probability that a point chosen at random from
严
U
will be from one of the unshaded regions, but the question asks for the
probability that the point will be from one of the shaded regions.
12
which of the following must be true?
If
(A)
(B)
(C)
(D)
(E)
ANSWERS
AND EXPLANATIONS
Explanation for Correct Answer B :
Choice (B) is correct. Expanding the left-hand side of
Subtracting
multiplying both sides by
file://E:\新建文件夹\e9.htm
from both sides yields
gives
now
gives
2006-11-12
页码,11/14
The Official SAT Online Course
Explanation for Incorrect Answer A :
must be
Choice (A) is not correct. The value of
not
Explanation for Incorrect Answer C :
13
the
if
does not have to be
Choice (C) is not correct. The value of
equation holds given any value for
Explanation for Incorrect Answer D :
Choice (D) is not correct. It is the value of
that must be
not the value of
Explanation for Incorrect Answer E :
Choice (E) is not correct. It is the value of
that must be
not the value of
what is the value of
If
(A)
(B)
!
途
(C)
It cannot be determined from the information given.
ANSWERS
用
AND EXPLANATIONS
st
er
(E)
ed
(D)
业
Explanation for Correct Answer A :
it follows that
Choice (A) is correct. Since
Thus,
商
eg
i
Therefore,
于
Explanation for Incorrect Answer B :
用
nR
Choice (B) is not correct. If
equal to
But
禁
equals
were equal to
then
would be
and so
not
Explanation for Incorrect Answer C :
is not equal to the sum of
Choice (C) is not correct. The value of
U
严
the constant terms
and
Explanation for Incorrect Answer D :
implies that
Choice (D) is not correct.
in the product
Therefore, each factor
is positive, and so the product cannot be equal to
Explanation for Incorrect Answer E :
can be determined from the condition
Choice (E) is not correct. The value of
Therefore, the value of the product
can also be
determined.
14
What is the ratio of the diameter
of a circle to half the circumference of the circle?
(A)
(B)
(C)
(D)
file://E:\新建文件夹\e9.htm
2006-11-12
页码,12/14
The Official SAT Online Course
(E)
ANSWERS
AND EXPLANATIONS
Explanation for Correct Answer C :
Choice (C) is correct. A circle with diameter
has circumference
Therefore, the ratio of the diameter to half
the circumference of the circle is
or
which is equal to
the circumference is
Thus, half
Explanation for Incorrect Answer A :
is the ratio of the radius of the circle to the
Choice (A) is not correct.
circumference of the circle.
Explanation for Incorrect Answer B :
is the ratio of the diameter of the circle to the entire
Choice (B) is not correct.
circumference, but the question asks for the ratio of the diameter to half the
circumference.
!
途
ed
Explanation for Incorrect Answer D :
is the ratio of the circumference of the circle to the
Choice (D) is not correct.
diameter of the circle.
用
te
r
Explanation for Incorrect Answer E :
is the ratio of the circumference of the circle to the
Choice (E) is not correct.
radius of the circle.
is
商
plates. If the
boxes of paper plates, and each box contains
A carton contains
dollars, what is the cost per paper plate, in dollars, when the plates
carton costs
are bought by the carton?
于
用
nR
(A)
eg
15
业
禁
(B)
(C)
U
严
(D)
(E)
ANSWERS
AND EXPLANATIONS
Explanation for Correct Answer B :
Choice (B) is correct. Each carton contains
contains
boxes of paper plates, and each box
plates, so each carton contains
dollars and contains
plates, so each carton contains
dollars and contains
dollars.
plates, the cost per plate is
Explanation for Incorrect Answer A :
Choice (A) is not correct. Each carton contains
box contains
plates. Since each carton costs
boxes of paper plates, and each
plates. Since each carton costs
plates, the cost per plate is
dollars, not
dollars.
file://E:\新建文件夹\e9.htm
2006-11-12
页码,13/14
The Official SAT Online Course
Explanation for Incorrect Answer C :
is the number of plates per dollar, not
Choice (C) is not correct. This choice,
the number of dollars per plate.
Explanation for Incorrect Answer D :
Choice (D) is not correct. Each carton contains
boxes of paper plates, and each
plates. Since each carton costs
plates, so each carton contains
box contains
dollars and contains
Explanation for Incorrect Answer E :
Choice (E) is not correct. Each carton contains
boxes of paper plates, and each
plates. Since each carton costs
plates, so each carton contains
box contains
dollars and contains
dollars, not
plates, the cost per plate is
dollars, not
plates, the cost per plate is
16
re
d
!
途
用
te
业
to
is
商
in the figure above are equilateral. What is the ratio of
于
用
to
nR
(A)
and
eg
Triangles
(B)
禁
(C)
to
(D)
to
(E)
to
严
U
to
ANSWERS
AND EXPLANATIONS
Explanation for Correct Answer B :
perpendicular bisector of
it follows that
and
Choice (B) is correct. Since
Let
the point where
and
is the
intersect, as
shown in the figure below.
Then
file://E:\新建文件夹\e9.htm
and
are all
triangles. Thus,
2006-11-12
页码,14/14
The Official SAT Online Course
it follows that
Since
to
is
to
the ratio of
Therefore,
and
Explanation for Incorrect Answer A :
would also be
to
then the ratio of
But it is given
to
would be
to
and so the ratio of
to
would
to
But then, by the Pythagorean Theorem, the ratio of
to
be
to
were
to
ratio of
intersect. If the
and
the point where
Choice (A) is not correct. Let
is equilateral, so this cannot be true.
that
Explanation for Incorrect Answer C :
Therefore, the ratio of
!
途
Explanation for Incorrect Answer D :
Explanation for Incorrect Answer E :
is
te
r
intersect. If the
to
But then, by the Pythagorean Theorem, the ratio of
to
业
and so the ratio of
to
be
and
then the ratio of
to
were
to
ratio of
用
the point where
Choice (E) is not correct. Let
to
is
to
but the ratio of
ed
to
is
to
intersect. The ratio
and
the point where
Choice (D) is not correct. Let
of
and
to
cannot be
to
so
Thus,
having measure
intersect. Then
having measure
triangle with
is a
and
the point where
Choice (C) is not correct. Let
to
would be
to
would also be
would
to
But it is
is equilateral, so this cannot be true.
given that
商
eg
于
nR
用
禁
Copyright © 2006 The College Board. All rights reserved.
Back to Score Report
Privacy Policy
Terms of Use
Contact Us
U
严
file://E:\新建文件夹\e9.htm
2006-11-12
页码,1/11
The Official SAT Online Course
Help | Profile | My Organizer | My Bookmarks | Logout
Answers and Explanations
Back to Score Report
Test Sections
Section 1
View Answers and Explanations
Section 2
Online - Practice Test #5
Section 4
The first 10,000 United States patents, they were known as the X-patents, were
burned in a fire in 1836.
(A) they were known
1
Section 5
Section 6
(B)
Section 8
which they knew
Section 9
(C) which they know
Section 10
(D) to be known
(E)
known
ANSWERS
AND EXPLANATIONS
!
途
d
Section 3
re
Explanation for Correct Answer E :
Choice (E) is correct. It avoids the comma splice of the original by replacing the
independent clause “they were known as the X-patents” with the participial
phrase (“known as the X-patents”) that describes the subject of the independent
clause, “patents.”
te
用
业
is
商
eg
Explanation for Incorrect Answer A :
Choice (A) involves a comma splice. This sentence contains two independent
clauses joined only by a comma: “The first 10,000 United States patents were
burned in a fire in 1836” and “they were known as the X-patents.” However,
adding a coordinating conjunction after the first comma or replacing the first
comma with a semicolon will not correct the problem because one independent
clause comes between the subject (“patents”) and verb (“were burned”) of
the other independent clause. The error can be corrected by turning the
interrupting clause into a subordinate participial phrase, “known as the Xpatents,” that modifies the subject (“patents”) of the other independent clause.
于
nR
用
禁
严
U
Explanation for Incorrect Answer B :
Choice (B) creates a pronoun error. This revision corrects the comma-splice error of
the original by turning one independent clause (“they were known as the Xpatents”) into a dependent clause introduced by “which,” but the revision
creates a pronoun error. The pronoun “they” has no antecedent to which it can
logically refer.
Explanation for Incorrect Answer C :
Choice (C) creates a pronoun error. This revision corrects the comma-splice error of
the original by turning one independent clause (“they were known as the Xpatents”) into a dependent clause introduced by “which,” but the revision
creates a pronoun error. The pronoun “they” has no antecedent to which it can
logically refer.
Explanation for Incorrect Answer D :
Choice (D) involves faulty logic. Although this revision corrects the comma-splice
error of the original by turning one independent clause (“they were known as the
X-patents”) into a subordinate infinitive phrase (“to be known”), it also creates
faulty logic. The infinitive phrase “to be known” illogically suggests that the
action will occur at some point in the future rather than now, in the present.
2
Winston knew that if he practiced often enough he would one day be able to play the
piano as well as his brother’s playing.
his brother’s playing
file://E:\新建文件夹\e10.htm
2006-11-12
页码,2/11
The Official SAT Online Course
(A)
(B)
that of his brother
(C) his brother’s
(D) his brother could
(E)
what his brother did
ANSWERS
AND EXPLANATIONS
Explanation for Correct Answer D :
Choice (D) is correct. It avoids the illogical comparison and mixed construction of
the original by replacing the gerund phrase “his brother’s playing” with the
clause “his brother could.” This change also untangles the mixed construction of
the phrase “as well as” by turning “as well” into an adverb modifying “to
play” and “as his brother could play” into an adverb clause modifying the
adverb “well.”
Explanation for Incorrect Answer A :
Choice (A) involves faulty logic. The sentence is meant to compare the way that
Winston would one day be able to play the piano with the way that his brother
already could play, but instead it illogically suggests that Winston could play the
piano as well as “his brother’s playing” could.
ed
!
途
Explanation for Incorrect Answer B :
Choice (B) results in faulty logic. The pronoun “that” refers to the noun
“piano,” so this construction illogically suggests that Winston could play the
piano as well as his brother’s piano could play.
er
用
业
is
t
Explanation for Incorrect Answer C :
Choice (C) results in faulty logic. With the phrase “his brother’s” serving as the
object of the preposition “as,” the construction illogically suggests that Winston
could play the piano as well as his brother’s piano could play.
eg
商
于
Explanation for Incorrect Answer E :
Choice (E) results in faulty logic. With the phrase “what his brother did” serving
as the object of the preposition “as,” the construction illogically suggests that
Winston could play the piano as well as something that his brother did could play.
3
nR
用
禁
In both his longer and his shorter works of fiction, Gabriel García Márquez achieves
the rare feat to be accessible to the common reader while satisfying the most
demanding of sophisticated critics.
U
严
(A)
to be
(B)
for being
(C) of being
(D) that he is
(E)
that they are
ANSWERS
AND EXPLANATIONS
Explanation for Correct Answer C :
Choice (C) is correct. It avoids the improper phrasing of the original by replacing
the infinitive phrase “to be” with an idiomatic participial phrase (“of being”) to
modify the noun “feat.”
Explanation for Incorrect Answer A :
Choice (A) involves improper phrasing. The preposition “of being,” instead of the
infinitive “to be,” is the idiomatic construction to modify the noun “feat.”
Explanation for Incorrect Answer B :
file://E:\新建文件夹\e10.htm
2006-11-12
页码,3/11
The Official SAT Online Course
Choice (B) involves improper phrasing. The preposition “of,” instead of the
preposition “for,” is the idiomatic preposition to follow “feat.”
Explanation for Incorrect Answer D :
Choice (D) involves an awkward construction. Although a dependent clause
introduced by the relative pronoun “that” can modify a noun, the construction is
not idiomatic in this sentence. The prepositional phrase “of being” is the idiomatic
construction to modify the noun “feat.”
Explanation for Incorrect Answer E :
Choice (E) involves an awkward construction and a pronoun error. Although a
dependent clause introduced by the relative pronoun “that” can modify a noun,
the construction is not idiomatic in this sentence. The prepositional phrase “of
being” is the idiomatic construction to modify the noun “feat.” Also, in the
context of this sentence, the pronoun that serves as the subject of the dependent
clause (“they”) should refer to the singular subject of the main clause,
“Marquez,” not to the object of the preposition, the plural noun “works.”
In 1977, Jann Wenner, the founder of Rolling Stone, moved the magazine’s offices
from San Francisco to New York City, there he began developing a slicker, more
commercial style of magazine and began cultivating relationships with major
advertisers.
4
(A)
there
(B)
then
!
途
d
(C) where
用
which is when
eg
is
te
(E)
re
(D) so
ANSWERS
业
AND EXPLANATIONS
Explanation for Correct Answer C :
Choice (C) is correct. It avoids the comma-splice error of the original by using the
relative pronoun “where” to introduce a subordinate clause that describes what
happened when the magazine moved its offices to New York City.
商
于
nR
用
Explanation for Incorrect Answer A :
Choice (A) creates a comma splice. The sentence presents two complete ideas, but
the ideas are joined using only a comma. Two independent clauses should never be
joined by only a comma. This problem can be fixed by using the relative pronoun
“where” to introduce a dependent clause that describes what happened when the
magazine moved its offices to New York City.
禁
U
严
Explanation for Incorrect Answer B :
Choice (B) creates a comma splice. The sentence presents two complete ideas, but
the ideas are joined using only a comma. Two independent clauses should never be
joined by only a comma. This problem can be fixed by using the relative pronoun
“where” to introduce a dependent clause that describes what happened when the
magazine moved its offices to New York City.
Explanation for Incorrect Answer D :
Choice (D) creates a comma splice. The sentence presents two complete ideas, but
the ideas are joined using only a comma. Two independent clauses should never be
joined by only a comma. This problem can be fixed by using the relative pronoun
“where” to introduce a dependent clause that describes what happened when the
magazine moved its offices to New York City.
Explanation for Incorrect Answer E :
Choice (E) involves awkward phrasing and ambiguous pronoun reference. While the
pronoun “which” is used correctly to introduce a dependent clause, the first part
of the sentence (“In 1977, Jann Wenner, the founder of Rolling Stone, moved the
magazine’s offices from San Francisco to New York City”) would have to be
recast so that the pronoun “when” would unambiguously refer to a time
(“1977”) rather than to a place (“New York City”).
file://E:\新建文件夹\e10.htm
2006-11-12
页码,4/11
The Official SAT Online Course
Covering about 120 square miles, the New York City borough of Queens is almost as
large as if you combine Manhattan, the Bronx, and Staten Island.
(A) if you combine Manhattan, the Bronx, and Staten Island
5
(B)
when Manhattan, the Bronx, and Staten Island are combined
(C) Manhattan, the Bronx, and Staten Island combined
(D) if Manhattan were to combine with the Bronx and Staten Island
(E)
combining Manhattan, the Bronx, and Staten Island
ANSWERS
AND EXPLANATIONS
Explanation for Correct Answer C :
Choice (C) is correct. It avoids the awkward phrasing and unnecessary shift in
person of the original by replacing the subordinate clause introduced by “if” with
a noun phrase, “Manhattan, the Bronx, and Staten Island combined,” which
makes the comparison logical by comparing a noun (“Queens”) with other nouns.
This revision also avoids the unnecessary shift to the second person by deleting
“you.”
!
途
ed
Explanation for Incorrect Answer A :
Choice (A) involves awkward phrasing and an unnecessary shift in person. In the
comparative construction “as … as,” the first “as” is an adverb that modifies
the adjective (“large”) that follows it, and the second “as” is a preposition,
which requires an object. An object of a preposition must be a noun. The clause “if
… Island” acts as an adverb, not a noun, so it cannot serve as the object of a
preposition. In addition, the use of the second-person pronoun “you” is an
unnecessary shift in person.
eg
is
te
r
用
业
Explanation for Incorrect Answer B :
Choice (B) involves awkward phrasing. In the comparative construction “as …
as,” the first “as” is an adverb that modifies the adjective (“large”) that
follows it, and the second “as” is a preposition, which requires an object. An
object of a preposition must be a noun. The clause “when … combined” acts as
an adverb, not a noun, so it cannot serve as the object of a preposition.
商
于
Explanation for Incorrect Answer D :
Choice (D) involves awkward phrasing. In the comparative construction “as …
as,” the first “as” is an adverb that modifies the adjective (“large”) that
follows it, and the second “as” is a preposition, which requires an object. An
object of a preposition must be a noun. The clause “if … Island” acts as an
adverb, not a noun, so it cannot serve as the object of a preposition.
nR
用
禁
Explanation for Incorrect Answer E :
Choice (E) makes an illogical comparison. In the comparative construction “as …
as,” the first “as” is an adverb that modifies the adjective (“large”) that
follows it, and the second “as” is a preposition, which requires an object. An
object of a preposition must be a noun. Although the gerund “combining” is a
noun, it results in an illogical comparison: Queens, a borough, is compared to
“combining,” not to the other boroughs.
U
严
6
While cotton is still one of Georgia’s chief cash crops, the number of acres devoted
to its cultivation grows smaller every year.
(A)
devoted to its cultivation grows
(B)
it devotes to the cultivation of it grow
(C) they devoted to its cultivation can grow
(D) devoted to its cultivation growing
(E)
they devoted to the cultivation of it are growing
ANSWERS
AND EXPLANATIONS
Explanation for Correct Answer A :
Choice (A) is correct. It avoids the errors of the other options by using concise
phrasing (“its cultivation”) in place of awkward wording, by removing
file://E:\新建文件夹\e10.htm
2006-11-12
页码,5/11
The Official SAT Online Course
unnecessary pronouns (“it” and “they”), and by providing a singular verb
“grows” to agree with the singular subject “the number.”
Explanation for Incorrect Answer B :
Choice (B) involves awkward phrasing and subject-verb disagreement. The awkward
phrase “it devotes to the cultivation of it” can be reduced to “devoted to its
cultivation.” The plural verb “grow” does not agree with the singular subject
“the number.” The singular verb “grows” should be used instead.
Explanation for Incorrect Answer C :
Choice (C) involves illogical pronoun use and an error in verb tense. There is
nothing in the sentence to which the plural pronoun “they” can logically refer,
and the use of the past tense (“devoted”) is not consistent with the use of the
present tense elsewhere in the sentence (“is”).
Explanation for Incorrect Answer D :
Choice (D) results in a sentence fragment. As is, the sentence contains two
dependent clauses. To fix this problem, a main verb is needed (“the number . . . is
growing”).
Explanation for Incorrect Answer E :
Choice (E) involves illogical pronoun use, awkward phrasing, and subject-verb
disagreement. There is nothing in the sentence to which the plural pronoun
“they” can logically refer, the awkward phrase “devoted to the cultivation of it”
can be reduced to “devoted to its cultivation,” and the plural verb “are” does
not agree in number with the singular subject “the number.”
is
te
re
d
!
途
用
7
Unlike flying squirrels, which may leave their young in a tree cavity while foraging,
the babies of flying lemurs are usually carried with them.
业
(A)
the babies of flying lemurs are usually carried
(B)
the flying lemur’s babies are usually carried
商
eg
(C) the flying lemur is usually carrying its babies
于
(D) flying lemurs usually carry their babies
(E)
flying lemurs’ babies are usually carried
nR
用
ANSWERS
禁
AND EXPLANATIONS
Explanation for Correct Answer D :
Choice (D) is correct. It avoids the illogical comparison of the original by replacing
the subject of the main clause (“babies”) with “flying lemurs” so that “flying
squirrels” and “flying lemurs” are compared.
U
严
Explanation for Incorrect Answer A :
Choice (A) involves an illogical comparison. Because the subject of the independent
clause is “babies,” adult “flying squirrels” are illogically compared to
“babies” rather than to “flying squirrels.”
Explanation for Incorrect Answer B :
Choice (B) involves an illogical comparison. Because the subject of the independent
clause, “babies,” is modified by the singular possessive “flying lemur’s,”
adult “flying squirrels” are illogically compared to “babies” rather than to
“flying lemurs.”
Explanation for Incorrect Answer C :
Choice (C) creates an unparallel comparison and a verb-tense error. The singular
subject of the independent clause, “flying lemur,” is compared to the plural noun
“flying squirrels.” Also, the simple present tense (“carries”), not the present
progressive tense (“is carrying”), should be used in statements of general truth
such as this one.
Explanation for Incorrect Answer E :
Choice (E) involves an illogical comparison. Because the subject of the independent
clause, “babies,” is modified by the plural possessive “flying lemurs’,” adult
file://E:\新建文件夹\e10.htm
2006-11-12
页码,6/11
The Official SAT Online Course
“flying squirrels” are illogically compared to “babies” rather than to “flying
lemurs.”
Born Charles Hardin Holley, Buddy Holly recorded some of the most distinctive and
influential songs in rock-and-roll music, which includes such classics as “That'll Be
the Day,” “Rave On,” and “Peggy Sue.”
(A) which includes
8
(B)
they include
(C) also including
(D) including
(E)
these include
ANSWERS
AND EXPLANATIONS
Explanation for Correct Answer D :
Choice (D) is correct. Its avoids the verb-tense error of the original. The tense of
the verb in the dependent clause should be consistent with the past tense of the
main verb “recorded.” Replacing the dependent clause (“which … ‘Peggy
Sue’”) with a participial phrase (“including … ‘ Peggy Sue’”) to modify the
noun “songs” fixes this inconsistency in verb tenses.
re
d
!
途
用
is
te
Explanation for Incorrect Answer A :
Choice (A) involves a verb-tense error. The tense of the verb in a dependent clause
should be consistent with the tense of the verb of the main clause. The present
tense of the verb “includes” is inconsistent with the past tense verb
“recorded.”
业
商
eg
Explanation for Incorrect Answer B :
Choice (B) creates a comma splice. The two independent clauses “Born Charles
Hardin Holley, Buddy Holly recorded some of the most distinctive and influential
songs in rock-and-roll music” and “they include such classics as ‘That'll Be the
Day,’ ‘Rave On,’ and ‘Peggy Sue’” are joined only by a comma rather
than by a semicolon or a comma and a coordinating conjunction.
于
nR
用
Explanation for Incorrect Answer C :
Choice (C) involves faulty logic and unclear modification. The adverb “also” used
to introduce the participial phrase (“also … ‘Peggy Sue’”) suggests that the
songs named in the participial phrase are in addition to other songs already named,
but no other songs have been named. Also, the noun that this participial phrase
modifies is unclear. It could modify either “songs” or “music” in the main
clause.
禁
U
严
Explanation for Incorrect Answer E :
Choice (E) creates a comma splice. The two independent clauses “Born Charles
Hardin Holley, Buddy Holly recorded some of the most distinctive and influential
songs in rock-and-roll music” and “these include such classics as ‘That'll Be the
Day,’ ‘Rave On,’ and ‘Peggy Sue’” are joined only by a comma rather
than by a semicolon or a comma and a coordinating conjunction.
9
Chipmunks do not technically hibernate, but they do remain in their dens or burrows
during cold periods.
(A)
hibernate, but they do remain
(B)
hibernate, but remaining
(C) hibernate, but they remained
(D) hibernate, they remain
(E)
hibernate, remaining
ANSWERS
AND EXPLANATIONS
Explanation for Correct Answer A :
file://E:\新建文件夹\e10.htm
2006-11-12
页码,7/11
The Official SAT Online Course
Choice (A) is correct. The two independent clauses “Chipmunks do not technically
hibernate” and “they do remain in their dens or burrows during cold periods”
are correctly connected with a comma and coordinating conjunction (“, but”), and
the conjunction “but” clearly expresses the contrast between the ideas of the two
clauses. Also, the two clauses are parallel in structure, and their verbs are
consistent in tense.
Explanation for Incorrect Answer B :
Choice (B) involves a lack of parallelism. The two grammatical units connected by a
coordinating conjunction (“but”) should be parallel, but in this construction, an
independent clause (“Chipmunks do not technically hibernate”) is connected with
a coordinating conjunction to a participial phrase (“remaining in their dens or
burrows during cold periods”).
Explanation for Incorrect Answer C :
Choice (C) involves an improper verb tense. The two independent clauses in this
sentence (“Chipmunks … hibernate” and “they …periods”) are statements of
general truth. Statements of general truth should be expressed in the present
tense. The past-tense verb of the second clause, “remained,” should be the
present-tense verb “remain.”
Explanation for Incorrect Answer D :
Choice (D) creates a comma splice and involves faulty logic. The two independent
clauses “Chipmunks do not technically hibernate” and “they remain in their
dens or burrows during cold periods” are connected only by a comma rather than
by a semicolon or a comma and a coordinating conjunction. Also, without the
conjunction “but” to express the contrast between the ideas of the two clauses,
the two clauses seem to contradict each other. The first clause states that
chipmunks do not hibernate, and the second clause suggests that they do.
ed
!
途
te
r
用
Explanation for Incorrect Answer E :
Choice (E) creates faulty logic. Although this sentence is grammatically correct, it is
not logical. The independent clause “Chipmunks do not technically hibernate” and
the participial phrase “remaining in their dens or burrows during cold periods'
contradict other." This faulty logic can be corrected by connecting the two ideas
with a contrasting conjunction, as in the original.
is
业
nR
eg
商
于
10
用
Cryonics — the technique used to store human bodies at extremely low temperatures
with the hope of one day reviving them — are being performed today, but their
technology is still in its infancy.
U
严
禁
(A)
are being performed today, but their technology
(B)
are being performed today, while the technology
(C) which is being performed today, using technology that
(D) is being performed today, but their technology
(E)
is being performed today, but the technology
ANSWERS
AND EXPLANATIONS
Explanation for Correct Answer E :
Choice (E) is correct. It avoids the errors in subject–verb agreement and
pronoun–antecedent agreement of the original. It replaces the plural verb “are
being performed” with the singular verb “is being performed” to agree with the
singular subject “Cryonics.” This revision also replaces the plural pronoun
“their,” for which there is no logical antecedent.
Explanation for Incorrect Answer A :
Choice (A) involves errors in subject–verb agreement and pronoun–antecedent
agreement. “Cryonics” is a singular noun and requires the singular verb “is
being performed,” not the plural verb “are being performed.” Also the pronoun
“their” can not logically refer to anything in the sentence.
Explanation for Incorrect Answer B :
Choice (B) involves subject–verb disagreement and imprecise word choice.
file://E:\新建文件夹\e10.htm
2006-11-12
页码,8/11
The Official SAT Online Course
“Cryonics” is a singular noun and requires the singular verb “is being
performed,” not the plural verb “are being performed.” Also, the use of the
subordinate conjunction “while” to introduce the clause “while the technology is
still in its infancy” does not clearly express the contrast between the idea
expressed in the dependent clause and the idea of the main clause (“Cryonics … is
being performed today”).
Explanation for Incorrect Answer C :
Choice (C) creates a sentence fragment. Although the subject–verb and pronoun–
antecedent agreement errors are corrected, this revision creates a sentence
fragment. The use of “which” turns the verb phrase “is being performed
today” into a subordinate clause, which cannot stand alone. Also the participle
“using” turns all that follows into a subordinate phrase, which cannot stand
alone. Therefore, the subject “Cryonics” has no main verb to complete the
sentence.
Explanation for Incorrect Answer D :
Choice (D) involves an error in pronoun–antecedent agreement. There is nothing in
the sentence to which the plural pronoun “their” can logically refer.
11
In science fiction stories, robots are generally depicted as working in the service of
humanity, often escaping the control of their human masters and doing them harm.
(A) often escaping the control of their human masters and doing
!
途
often escaping being controlled by their human masters, which does
d
(B)
(C) but they often escape the control of their human masters and do
用
but often escaping their controlling human masters, which does
业
AND EXPLANATIONS
is
ANSWERS
te
(E)
re
(D) but they often escaped their controlling human masters, and it does
商
eg
Explanation for Correct Answer C :
Choice (C) is correct. It avoids the errors of the other options by precisely
expressing the contrast between the idea of the main clause (“robots …
humanity”) and the idea of the subordinate phrase (“often … harm”).
于
nR
用
Explanation for Incorrect Answer A :
Choice (A) inappropriately uses a participial phrase (“often … doing”) that does
not precisely express the contrast between the two ideas in the sentence (“robots
… humanity” and “often … harm”).
禁
U
严
Explanation for Incorrect Answer B :
Choice (B) involves imprecise phrasing and faulty logic. The participial phrase
“often being controlled by their human masters” does not clearly express the
contrast between the idea of the main clause (“robots … humanity”) and the
idea of this subordinate clause. Also, the subordinate clause “which does them
harm” modifies “escaping,” and the pronoun “them” refers to “robots.”
This clause illogically suggests that the robots cause themselves harm by escaping
human control, instead of suggesting that the robots harm their human masters
when they escape human control.
Explanation for Incorrect Answer D :
Choice (D) involves errors in verb tense and word choice and faulty logic. The past
tense of the verb “escaped” is inconsistent with the present tense of the main
verb “are … depicted.” Also, the adjective “controlling,” used to modify
“human masters,” illogically changes the meaning from humans in general to
only those humans who control robots. In addition, the pronouns “it” and
“them” in the independent clause “and it does them harm” illogically suggests
that the robots cause themselves harm by escaping human control, instead of
suggesting that the robots harm their human masters when they escape human
control.
Explanation for Incorrect Answer E :
Choice (E) involves improper word choice and faulty logic. The adjective
“controlling,” used to modify “human masters,” illogically changes the
meaning from humans in general to only those humans who control robots. In
addition, the subordinate clause “which does them harm” modifies “escaping,”
and the pronoun “them” refers to “robots.” This clause illogically suggests that
file://E:\新建文件夹\e10.htm
2006-11-12
页码,9/11
The Official SAT Online Course
the robots cause themselves harm by escaping human control, instead of
suggesting that the robots harm their human masters when they escape human
control.
The first Indian woman to win the Booker Prize, Arundhati Roy’s debut novel, The
God of Small Things, received the honor in 1997.
12
(A)
The first Indian woman to win the Booker Prize, Arundhati Roy’s debut
novel, The God of Small Things, received the honor in 1997.
(B)
Arundhati Roy, the first Indian woman to win the Booker Prize for her
debut novel The God of Small Things, receiving the honor in 1997.
(C) The first Indian woman to receive the Booker Prize, Arundhati Roy won the
honor in 1997 for her debut novel, The God of Small Things.
The Booker Prize was given to Arundhati Roy’s debut novel The God of
(D) Small Things, winning her the first such honor for an Indian woman in
1997.
(E)
Arundhati Roy, winning the Booker Prize in 1997 for her debut novel The
God of Small Things, made her the first Indian woman to do so.
ANSWERS
AND EXPLANATIONS
!
途
ed
Explanation for Correct Answer C :
Choice (C) is correct. It avoids the apposition error of the original by replacing the
subject of the main clause, “novel,” with the subject “Arundhati Roy” so that
the introductory appositive, “The first Indian woman to receive the Booker
Prize,” properly renames the author, not the novel.
er
用
业
is
t
Explanation for Incorrect Answer A :
Choice (A) involves an error in apposition. In this sentence, the introductory
phrase, “The first Indian woman to receive the Booker Prize,” illogically renames
“novel,” the subject of the main clause, rather than the author, Arundhati Roy.
商
eg
Explanation for Incorrect Answer B :
Choice (B) creates a sentence fragment. Neither the infinitive “to be” nor the
participle “receiving” can serve as a main verb for the subject “Arundhati
Roy.” A main verb is needed to complete the action of the sentence.
于
nR
用
Explanation for Incorrect Answer D :
Choice (D) involves faulty logic and faulty modification. The participial phrase
“winning her the first such honor” modifies the subject (“Booker Prize”) of the
main clause. It does not make sense to say that the prize won the prize. In
addition, the prepositional phrases “for an Indian woman” and “in 1977”
combine to illogically suggest that Roy won the prize “for an Indian woman in
1977,” not for her own novel, The God of Small Things.
禁
U
严
Explanation for Incorrect Answer E :
Choice (E) creates faulty logic. In its simplest subject–verb–direct object pattern,
this sentence states “Arundhati Roy … made … her,” which illogically suggests
that the author made herself.
13
Pilots at the airline, angered at the prospect of seeing their pension plans replaced
with less generous versions, vowed to use legal means to fight it.
(A)
it
(B)
them
(C) this
(D) such a move
(E)
that from happening
ANSWERS
AND EXPLANATIONS
Explanation for Correct Answer D :
Choice (D) is correct. It avoids the pronoun error of the original by replacing the
pronoun “it,” which has no clear antecedent, with the noun phrase “such a
file://E:\新建文件夹\e10.htm
2006-11-12
页码,10/11
The Official SAT Online Course
move.”
Explanation for Incorrect Answer A :
Choice (A) involves a pronoun error. The pronoun “it” has no antecedent to which
it clearly refers. Because the pronoun is singular, “it” can refer to either of the
two singular nouns (“prospect” or “airline”).
Explanation for Incorrect Answer B :
Choice (B) involves a pronoun error. The plural pronoun “them” has no
antecedent to which it clearly refers. Because it is plural, “them” could refer to
any of the plural nouns in the first part of the sentence: “pilots,” “plans,” or
“versions.”
Explanation for Incorrect Answer C :
Choice (C) involves a pronoun error. The pronoun “this” has no antecedent to
which it clearly refers. Because the pronoun is singular, “this” can refer to either
of the two singular nouns (“prospect” or “airline”).
Explanation for Incorrect Answer E :
Choice (E) involves a pronoun error and awkward phrasing. The pronoun “that”
has no antecedent to which it clearly refers. Because the pronoun is singular,
“that” can refer to either of the two singular nouns (“prospect” or “airline”).
In addition to its pronoun problem, the phrase “fight that from happening” is not
idiomatic. The idiomatic phrase is “keep that from happening,” but this correction
still does not correct the pronoun problem.
te
re
d
!
途
用
14
This basic document is stating the liberties guaranteed to the English people, the
Magna Carta, signed in 1215 by England’s King John, proclaims rights that have
formed the foundation of the constitutions of every English-speaking nation.
业
This basic document is stating
(B)
This basic document states
is
(A)
商
eg
(C) A basic document, it states
于
(D) A basic document that states
A basic document, it stated
用
nR
(E)
ANSWERS
禁
AND EXPLANATIONS
Explanation for Correct Answer D :
Choice (D) is correct. It avoids the comma-splice error of the other options by
turning the first independent clause, “This basic document is stating the liberties”
into an appositive. An appositive is a subordinate noun phrase that renames a
noun. In this revision, “A basic document” is the appositive that renames
“Magna Carta,” and the dependent clause “that states the liberties” modifies
“a basic document.”
U
严
Explanation for Incorrect Answer A :
Choice (A) involves a comma splice. This long sentence contains two independent
clauses (“This basic document … the English people” and “the Magna Carta,
signed … nation”) joined by only a comma. The error can be corrected by turning
the first independent clause into an appositive that renames the subject of the
second independent clause, “the Magna Carta.” In addition, the progressive
tense of the verb “is stating” should be changed to the simple present “states”
to be consistent with the present tense of the independent clause (“proclaims”).
Explanation for Incorrect Answer B :
Choice (B) involves a comma splice. This long sentence contains two independent
clauses (“This basic document … the English people” and “the Magna Carta,
signed … nation”) joined by only a comma. The error can be corrected by turning
the first independent clause into an appositive that renames the subject of the
second independent clause, “the Magna Carta.”
Explanation for Incorrect Answer C :
Choice (C) involves a comma splice. This long sentence contains two independent
file://E:\新建文件夹\e10.htm
2006-11-12
页码,11/11
The Official SAT Online Course
clauses (“A basic document, it … the English people” and “the Magna Carta,
signed … nation”) joined by only a comma. The error can be corrected by turning
the first independent clause into an appositive that renames the subject of the
second independent clause, “the Magna Carta.”
Explanation for Incorrect Answer E :
Choice (E) involves a comma splice. This long sentence contains two independent
clauses (“A basic document, it … the English people” and “the Magna Carta,
signed … nation”) joined by only a comma. The error can be corrected by turning
the first independent clause into an appositive that renames the subject of the
second independent clause, “the Magna Carta.”
Back to Score Report
Privacy Policy
Copyright © 2006 The College Board. All rights reserved.
Terms of Use
Contact Us
ed
!
途
st
er
用
业
eg
i
商
于
nR
用
禁
U
严
file://E:\新建文件夹\e10.htm
2006-11-12